X



トップページ数学
1002コメント443KB
分からない問題はここに書いてね459
レス数が1000を超えています。これ以上書き込みはできません。
0003132人目の素数さん2020/04/01(水) 16:28:45.97ID:MvKFIRgS
円x^2+(y-1)^2=1に外接し,x軸にも接する円の中心をPとするとき,点Pの軌跡を求めよ。
ただし,円の中心Pがy軸上にあるときは除くものとする。

おそらく楕円になりそうですが、焦点が外接点なのか、x軸との接点なのかわけわかりません。
自分にとっては解き方教えてもらわないと絶対無理だろの問題です。
解き方を教えていただけないでしょうか?
0004132人目の素数さん2020/04/01(水) 17:43:32.38ID:/dxHEpu+
>>3
放物線じゃね?

円Pの半径をrとすると、円Pは外接するんだから、Pと(0,1)との距離はr+1
そこから式を立てたらいい
0005132人目の素数さん2020/04/01(水) 18:00:35.88ID:MvKFIRgS
中心間の距離を使ってどのような式を立てればよいんでしょうか?
0006132人目の素数さん2020/04/01(水) 18:23:45.92ID:Z9DRJTdr
放物線C:y=x^2上の点Pにおける接線の上に、PX=1をみたす点Xをとる。ただしXのx座標はPのx座標より大きいとする。
PがC上を動くとき、Xが動いてできる曲線を求めよ。
0007132人目の素数さん2020/04/02(木) 05:42:45.20ID:nU8XEOiy
三角形の内接円と傍接円の共通接線4本のうち3本は三角形の辺ですが
残り一本は外接円の(傍接円の逆の位置の)頂点での接線に平行なことを示せ
0009132人目の素数さん2020/04/02(木) 11:03:55.19ID:5muIiata
ああ、わかった。
△ABCの∠Aの二等分線とBCの交点をP、
外接円のAにおける接線と直接BCの交点をQとする。
AB<ACとしてよい。
∠QAB=x、∠PAB=∠PAC=yとする。
∠WAP=x+y。
接弦定理により∠ACB=x。
∴APQ=x+y。
よって辺BCを直接APについて反転させた直線lと直線APのなす角もx+y。
∴l//AQ。
一方で直線BCは内接円と辺BCに接する傍接円の両方に接し、直線APはこの2円の中心を通るからlはもまたこの2円に接する。□
0010132人目の素数さん2020/04/02(木) 20:45:46.51ID:/ibIj00g
>>3
円の中心を P(x,y)、半径をr>0 とすれば
 x^2 + (y-1)^2 = (r+1)^2,
 |y| = r,
これより
 P(x,y) = (±2√r, r)
よって
 y = xx/4,  (x≠0)
0011132人目の素数さん2020/04/02(木) 21:14:03.67ID:/ibIj00g
>>6
P(p,p^2) におけるCの接線は
 y = 2p(x-p) + pp,
 X(p + 1/√(1+4pp), pp + 2p/√(1+4pp)) = (x,y)
 
 xx-y = 1/(1+4pp),

 p = x - √(xx-y),
を入れて
 (xx-y){1 + 4[x-√(xx-y)]^2} = 1,
変な問題だな。
0012132人目の素数さん2020/04/03(金) 11:15:51.08ID:Giqrz/Jd
>>7
内接円と傍接円Aの辺以外の共通接線 // 傍接円Bと傍接円Cの辺以外の共通接線
でもあった
0013132人目の素数さん2020/04/03(金) 14:34:43.44ID:CZycMBRW
V を有限次元ベクトル空間とする。
V' を V の双対空間とする。
U^0 := {φ ∈ V' | φ(u) = 0 for all u ∈ U} とする。

U と W を V の部分空間とし、 W^0 ⊂ U^0 とする。

U ⊂ W を証明せよ。
0014132人目の素数さん2020/04/03(金) 16:47:41.35ID:ykv6CEBk
任意のU要素 u を持ってくる.
適当な直和分解: V=W+W’ に対して u= w + w’.
任意の φ ∈ W^0(⊂U^0) に対して φ(w’)=φ(u)-φ(w) = 0
よって w’=0 (そうでなければ φ(w’)=1 となる φ(∈W^0)が構成できる)
ゆえに u=w ∈ W, 即ち U ∈ W である.
0016132人目の素数さん2020/04/03(金) 17:52:54.33ID:CZycMBRW
>>14

ありがとうございました。
0017132人目の素数さん2020/04/03(金) 22:10:32.47ID:JgK+ktFN
0---3
長さ3の数直線があり、一方の端の座標を0、他方を3とする
数直線上に2点X,Yをランダムに配置する
X,Yの座標をそれぞれx,yとしこのときx,yの距離が1以下になる確率を求めよ

*(条件から0≤x≤3,0≤y≤3のうち |x-y|≤1を満たす面積を考えれば幾何的に解けますがこの方法は置いておき)
この問いを連続型の確率分布とみて解く場合、どのように解けばよいでしょうか
次の確率密度関数のようなものが成り立(ちそう)だと思いましたが、
この後どうすればよいか or 根本から間違っているのでしょうか、ご教授下さい
f(x)= { (1+x)/3 (0≤x≤1)
2/3 (1≤x≤2)
(4-x)/3 (2≤x≤3)
0 (x≤0, 3≤x)
0018132人目の素数さん2020/04/03(金) 23:01:27.95ID:1+NoQgUm
fはX=xの下でYがxの1近くになる確率だからfにX=xとなる確率密度=1/3を掛けて積分する

∫[0,3]P(X=x)*P(max(0,x-1)<Y<min(x+1,3)│X=x)dx
=∫[0,1]P(X=x)*P(0<Y<x+1│X=x)dx
+∫[1,2]P(X=x)*P(x-1<Y<x+1│X=x)dx
+∫[2,3]P(X=x)*P(x-1<Y<3│X=x)dx
=∫[0,1]1/3*(x+1)/3dx+∫[1,2]1/3*2/3dx+∫[2,3]1/3*(4-x)/3dx
=1/9{(2^2-1^2)/2+2(2-1)+(4(3-2)-(3^2-2^2)/2}=5/9
0019132人目の素数さん2020/04/03(金) 23:03:58.32ID:ykv6CEBk
> *(条件から0≤x≤3,0≤y≤3のうち |x-y|≤1を満たす面積を考えれば幾何的に解けますがこの方法は置いておき)

置いておくも何もこの↑発想が 連続型の確率分布を前提としたものです.
[0,3] の線分上に一様ランダムに置かれるものとすれば
f(x,y) = 1/9 (0≤x≤3,0≤y≤3)
f(x,y) = 0 (それ以外)
となるでしょう. ( ∬dxdy f(x,y) = 1 )
〜を満たす領域S上での積分 ∬[(x,y)∈S] dxdy f(x,y) が 〜が起きる確率と解釈されます.
その結果として「幾何学的な "面積比率" を計算すればいいよ」という事になるわけです.
0020132人目の素数さん2020/04/03(金) 23:27:09.76ID:1+NoQgUm
幾何的に解くのは、二次元の一様分布F(x,y)=1/3^2を考えて、xとyが条件を満たす所を積分し
高さが1/3^2、面積が3*3-2*2の柱体の体積を求めるのと同じことだよね

f(x)を使うのは、xを固定してF(x,y)を切ってyが条件を満たす確率を求めてから、
xで累次積分するわけで同じことをやっている
0021132人目の素数さん2020/04/04(土) 00:25:45.42ID:PDQQaHtx
000〜999の中の数字の総数は?
0022132人目の素数さん2020/04/04(土) 00:34:17.47ID:OxSzzHQy
>>21
3000
0023132人目の素数さん2020/04/04(土) 00:55:22.18ID:PDQQaHtx
>>22
3×1000=3000ですよね
けど3×1000になる意味が分からないんです
0024132人目の素数さん2020/04/04(土) 02:34:27.35ID:X//y3pNZ
数学に詳しい人に聞きたいです。

命数の垓の次って?か??のどちらですか?
読み方は(じょ、し、ぢょ、ちょ)のどれですか。

Google検索結果等でも意見が分かれたままなのではっきりしてほしいです。
0025132人目の素数さん2020/04/04(土) 07:50:53.14ID:N7V4+fNK
>>23
000〜999っていうのは000、001、002、……、998、999という1000個の数のことじゃないの?
1個あたり3つの数字が使われていてそれが1000個あるんだから3000
0026132人目の素数さん2020/04/04(土) 09:33:24.66ID:zR6GpX6K
ど底辺の私に教えて欲しいのですが、

10個の景品を求めて150人で抽選します。
抽選には家族3人で参加します。
家族のうち誰か1人でも当たれば良いとして
何分の一くらいの確率になりますか?

5分の1ですか?
0027132人目の素数さん2020/04/04(土) 10:21:11.37ID:S2S4Ftgc
黒碁石が3個と白碁石が147個入っているツボからランダムに10個取ったときに
黒がx個である確率はC[3,x]C[147,10-x]/C[150,10]だから
x=0の確率はC[147,10]/C[150,10]=Π[k=3,12](150-k)/Π[k=0,9](150-k)
=Π[k=10,12](150-k)/Π[k=0,2](150-k)=140*139*138/(150*149*148)
0030 【小吉】 2020/04/04(土) 10:57:15.70ID:xmNOPA8p
>>28
>>3第1象限に円を描いていくと、
。оΟノ↑このように中心はx軸から遠ざかり、第2象限でもy軸と線対称に同様な図形が描けるから、放物線になる。
y=ax^2とおくと、
x^2+(y-1)^2=1と合同な外接円をx軸とも接するとき中心は(2,1)だから、
これを代入し1=a・2^2
1=4a
a=1/4
∴y=x^2/4
0031 【男の娘】 2020/04/04(土) 11:08:25.06ID:xmNOPA8p
>>30
その玉は、
その小さな玉は、
こっちがx軸上を原点からどんなけ離れようとも、
Pがどんなけx軸から離れようとも、
点(0,1)に居ながらにしてクルッと首だけ180°見渡して絶対真うしろにはまわりこませねえ。
せやで楕円にはならない。
0032 【大凶】 2020/04/04(土) 11:19:10.18ID:xmNOPA8p
>>31
なんどもその玉の写メを撮ろうとしたけど、遠巻きに撮るかモザイクがやっと。
2年間つかず離れず、結局名前はわからないまま。
いつかまた逢える日を楽しみにしてる。
原点でな。
0033132人目の素数さん2020/04/04(土) 12:03:26.31ID:oC6v+ILG
>>25
1個あたり3つの数字というのはどういう事ですか?
0〜999で1000個の数があって、000〜999の場合、0が000になるのは分かりますが、999は999のままじゃないですか?
0034132人目の素数さん2020/04/04(土) 12:20:24.72ID:S2S4Ftgc
>>29
簡単に考えていいなら、石を一つ取って戻しを10回繰り返したと考えると楽
全てが白でない確率は1-(147/150)^10=1-(1-1/50)^10≒1-(1-10/50)=1/5

あるいは、たった一人でチャレンジする場合の当たる確率は10/150=1/15で低いので
三人でチャレンジする場合は複数人が当たる確率は低いと見て無視して単に三倍して1/5
0035132人目の素数さん2020/04/04(土) 12:55:35.86ID:hLQ36is2
>>24

10^24 は 秭(禾弟?)または 𥝱(禾予?)読みは「じょ」

なお、SI単位系では yotta と呼ぶらしい。ヨタ話ですが・・・・
0036132人目の素数さん2020/04/04(土) 13:06:41.71ID:zR6GpX6K
>>34
分かりやすくありがとうございます。
確率論なんかの難しいことは全くわからないのですが
考え方は間違ってなさそうで良かったです。
0038132人目の素数さん2020/04/04(土) 17:08:30.33ID:/dVTYSYf
>>26, >> 37
確率: P{3人とも外れる}
 = [外れ140個から3つ選ぶパターン総数] / [全150個から3つ選ぶパターン総数]
 = (140*139*138) / (150*149*148)
P{3人の誰かが当たる}
 = 1 - P{3人とも外れる}
 = 1 - (140*139*138) / (150*149*148) = 5186 / 27565
 = 0.18813...
0039132人目の素数さん2020/04/04(土) 17:39:37.42ID:/dVTYSYf
別の考え方
P{3人の誰かが当たる}
 = P{3人のうち誰か1人が当たる} + P{3人のうち誰か2人が当たる} + P{3人のうち3人が当たる}
 = C{3,1} P{3人のうち指定済の1人が当たり, 2人が外れる}
    + C{3,2} P{3人のうち指定済の2人が当たり, 1人が外れる}
    + C{3,3} P{3人のうち指定済の3人が当たる}
 = ( 3* 10*140*139 + 3* 10*9*140 + 1* 10*9*8 ) / (150*149*148)
 = 5186 / 27565
 = 0.18813...
0041132人目の素数さん2020/04/04(土) 20:07:08.38ID:mbv9hr8e
円Aの内部に円Bがあり両円の中心を結んだ直線と円Aの交点をN_0とする。
N_0を通る円Bの接線(2接線のどちらか)と円Aの交点をN_1とする。
N_1を通る円Bの接線と円Aの交点(のうちN_1じゃない方を)をN_2とする。以下同様に接線を引き続けたときに
N_0に戻ってくるためには両円の半径の比rと中心間の距離dがどのような条件を満たせばよいか?
0042132人目の素数さん2020/04/04(土) 21:14:36.81ID:N7V4+fNK
>>33
元の問題の「数字の総数」が何を意味しているのかをはっきりさせてくれないと答えようがない
>>25は000には0という数字が3つ使われているという意味で回答している
000で1つの数字、001も002も999もそれぞれに1つの数字ととらえるのであれば000〜999には1000個の数字がある
0043132人目の素数さん2020/04/04(土) 23:47:00.34ID:PDQQaHtx
>>42
すみません。納得しました。
ありがとうございました。
0044132人目の素数さん2020/04/05(日) 01:20:49.38ID:O2isFBZQ
黒点が等間隔で一辺に各頂点を含んで7個ずつ置かれている正三角形があり、内部にも等間隔で黒点が並んでいる
この時、正三角形の内部、返上から黒点を3つ選んだ時、正三角形は全部で何個できるか?
8C3でダメなのは何でですか?
0045132人目の素数さん2020/04/05(日) 01:30:53.05ID:O2isFBZQ
すみません
8C3じゃなく、8C4ではなぜダメなのか、です
0047132人目の素数さん2020/04/05(日) 05:26:57.67ID:P4P+PQ/3
分からない問題というより質問なんですが、
虚数iってZFC公理系からどう厳密に構成するんでしょうか

現代数学のほとんどはZFC公理系から作れると聞いたのですが、iの作り方についてはいくらググっても調べられませんでした
0048132人目の素数さん2020/04/05(日) 06:17:15.66ID:+Vk/p+LH
R^2に適当に演算入れたときの(0,1)とか
多項式環の剰余環R[x]/(x^2+1)におけるxの像とか
0049132人目の素数さん2020/04/05(日) 06:27:03.26ID:P4P+PQ/3
>>48
すみません 下は勉強不足で分からないですが、上は確かに複素数の演算規則さえ与えればR^2と思えるということですか
なるほどありがとうございます
0050132人目の素数さん2020/04/05(日) 08:13:50.07ID:WyYvb5xI
>>44
問の設定が合ってるか自信ないが↓このように計算してみた.

{含まれる正三角形の個数}
= #size1+#size1’ +#size2+#size2’ +#size3+#size3’ +#size4 +#size5 +#size6
= (6*7+5*6 +5*6+3*4 +4*5+1*2 +3*4 +2*3 + 1*2) / 2
= 78
0051132人目の素数さん2020/04/05(日) 11:17:37.25ID:iq2DMm8O
>>46
ABC予想で証明できる?
0052132人目の素数さん2020/04/05(日) 12:00:56.17ID:D9Ap+iGK
正の数aの平方根のうち非負のものを√aと表す。
この定義に基づいて、0<x<yならば√x<√yを証明せよ。
0053132人目の素数さん2020/04/05(日) 12:02:45.61ID:ld7dxKAs
>>50
回答ありがとうございます
各点を結ぶので、斜めの形の正三角形も考慮する必要があります
0054132人目の素数さん2020/04/05(日) 13:13:37.91ID:WyYvb5xI
>>53
斜めは気づきませんでした.
正三角形が入った籠を数えると数えやすくなるでしょう.

#{斜め正三角}
= ( 4*5 + 3*4+2*3+1*2 + 2*3 + 1*2 ) / 2 * 2 =  48

もう少しスマートな数え方があるといいのですが...
0055132人目の素数さん2020/04/05(日) 14:12:20.30ID:HwkRF7us
>>54
回答ありがとうございます
この問題、解答だと、9C4で答えを出しているのですが、8C4だとどこの正三角形が考えられないのかがイマイチ分からないです
0056132人目の素数さん2020/04/05(日) 14:13:41.19ID:Wk6Sgfev
>>52
√x = 0 と仮定すると x = (√x)^2 = 0 となり題意と矛盾する。
∴ √x > 0,
同様にして √y > 0,
辺々たして √y + √x > 0
一方、題意により y-x > 0,
∴ √y - √x = (y-x)/(√y + √x) > 0,
∴ √y > √x.
0057132人目の素数さん2020/04/05(日) 14:28:56.62ID:Wk6Sgfev
〔補題〕
m,n が自然数ならば
 m^n - n^m = 0     (m=n または{m,n}={2,4}のとき)
 (m^n-n^m)/(m-n)> 0  (m≠n かつ(m=1 または n=1 または m+n≦5)のとき)
 (m^n-n^m)/(m-n)< 0  (m≠n かつ(m≧2 かつ n≧2 かつ m+n≧7)のとき)
0058132人目の素数さん2020/04/05(日) 14:31:28.31ID:WyYvb5xI
>>55
9C4 や 8C4 どういう背景からでてきたのか気になります.
模範解答に何の解説も無いのですか?
0059132人目の素数さん2020/04/05(日) 14:36:43.94ID:XNtMeTPW
>>44
この問題、ニフティサーブのフォーラムで出されていたのを思い出します。
「正置な正三角形」という概念を導入します。
正置な正三角形とは、文字通り、一辺は水平で、この辺の上方に頂点を持つ「向き」に
置かれた正三角形です。そして、
・サイズnの正置な正三角形には、n個の正三角形が属す
が言えます。(傾いたものを含む)ある正三角形があると、その正三角形の三つの頂点全てを含む
正置な正三角形がただ一つだけ定まります。このように、一つの正置な正三角形に定まることを指して、
「属す」と表現してます。ちょっと考えてみれば、自明なことです。

従って、サイズkの正置な正三角形がいくつあるかを数え上げ、それをk倍して総和をとれば、求めたいものが求まります。
サイズ1,2,3,...,6の正置な正三角形は、それぞれ、21,15,10,6,3,1個あるので、
1*21+2*15+3*10+4*6+5*3+6*1=21+30+30+24+15+6=126

一辺の頂点の数がn+1(=サイズがn)であれば、サイズnの正置な正三角形は、C[n+1,2]個あるので、
Σ[k=1,n](n+1-k)*C[k+1,2]=C[n+3,4]
が答えとなります。
0060132人目の素数さん2020/04/05(日) 14:38:41.85ID:pXbdSZye
https://imgur.com/a/6tcbMmr

3(2)ですが、
「Bを計算せよ」とはどういうことなんでしょうか

f(x)=log(1+x)(1-x)=log(1+x)+log(1-x)

f'(x)=1/(1+x) − 1/(1-x)

f(n)(x)={((-1)^(n-1))(n-1)︕}/(1+x)^n − (n-1)︕/(1-x)^n

より

nが奇数のとき f(n)(0)=0
nが偶数のとき f(n)(0)=-2・(n-1)︕

f(x)=−2Σk=1〜∞(x^2k)/2k

でいいのですか?

(1)の結果を利用するとはどういうことを言ってるのでしょうか。
0061132人目の素数さん2020/04/05(日) 14:51:18.19ID:HwkRF7us
>>58>>59
回答ありがとうございます
下に2段分黒点を追加して、一辺9個ずつの正三角形にし、その底辺について9つの点から4個の点を選び、それをABCDとします。この時、Cの左隣の黒点をC'とし、AとC'からは右上に向かって、Dは左上に線を引きます
この時、C'とDの線がぶつかったところからC'は真左に向かって線を伸ばし、これが作られる三角形の底辺となります
またBはAC'Dを固定した時、 C'を含む残りの点から選ぶことができ、これが斜めの三角形などの個数の代わりになります(このやり方では斜めの三角形を直接表すことが出来ないので、代替している)
よって9C4となるみたいです
2段追加して考えているのですが、1段追加しただけで、8C4とすると、どこで不備が出てくるのかが分からないです
0062132人目の素数さん2020/04/05(日) 14:52:01.93ID:+Vk/p+LH
(1)の結果でxに-x^2を代入(xの範囲には注意)
それで正しい変形になってるかどうかは級数展開の一意性からわかる
0063582020/04/05(日) 14:58:41.74ID:WyYvb5xI
>>59 ありがとうございます.

Σ[k=1,n] k * #{正置正三角形 size:k}
=Σ[k=1,n] k * C[n-k+2,2]
=Σ[k=1,n](n+1-k)*C[k+1,2]
ここまでは理解できました.
=C[n+3,4]
この最後の式変形がちょっと考えて診たのですが分かりません. (常識なのでしょうか...)
どうかご教授願います.
0064132人目の素数さん2020/04/05(日) 15:09:52.76ID:XNtMeTPW
中略しただけです。
結果が、「たまたま」コンビネーションを使って簡単に書けるので、それを用いただけですが、
「たまたま」ではなく、何らかの「必然性」が背後に隠れている気はしますが、ちょっと不明です。
0065132人目の素数さん2020/04/05(日) 16:01:53.81ID:2F4ElAOS
すべての自然数を、素数と高々 k 個の素数の積である数との和で表すことのできるような、k が存在することを証明してくれ〜
0066132人目の素数さん2020/04/05(日) 16:24:00.56ID:U+ODL1ZB
ポエムにもほどがあるだろ
0067132人目の素数さん2020/04/05(日) 17:00:21.27ID:WyYvb5xI
>>64 ありがとうございます.
必然性のある関係式が得られました. (参考 [wikipedia: 二項係数])

1/(1-x)^a = Σ[m=0,∞] a(a+1)..(a+ m-1)/m! x^m
= Σ[m=0,∞] C[a-1+m, m] x^m = Σ[m=0,∞] C[a-1+m, a-1] x^m

Σ[m=0,∞] C[p+q-1+m, p+q-1] x^m = 1/(1-x)^{p+q}
= 1/(1-x)^p * 1/(1-x)^q
= Σ[m=0,∞] { Σ[k=0,m] C[p-1+k, p-1] C[q-1+m-k, q-1] } x^m
= Σ[m=0,∞] { Σ[k=1,m+1] C[p-2+k, p-1] C[q+m-k, q-1] } x^m

x^m の係数を比較して
C[p+q-1+m, p+q-1] = Σ[k=1,m+1] C[p-2+k, p-1] C[q+m-k, q-1]

m=n-1, p+q=5, q=2  ∴ p=3 の代入により
C[n+3,4] = Σ[k=1,n] C[k+1,2] (n+1-k)

初等的には >>61 のイメージから得られると予想するのですが
そもそも内容が理解できていません...
0068132人目の素数さん2020/04/05(日) 17:01:51.56ID:Wk6Sgfev
>>60
・ラグランジュの剰余
 R_n = (x^n)f^(n)(ξ)/n!
・コーシーの剰余
 R_n = x^(n-1)(x-ξ) f^(n)(ξ)/n!
などがある。ただし ξは0とxとの中間の或る値である。

どれを使うのか分かるはずだが・・・・

高木貞治:「解析概論」改訂第三版,岩波書店 (1961)
 第2章 §25, 定理28, p.61〜67

・余談
旧ソ連の物理学者I.タム(1895〜1971) は ロシア革命直後、ゲリラ隊につかまったとき
これを知っていて助かったらしい。

数セミ増刊「100人の数学者」日本評論社 (1989) p.116
0069132人目の素数さん2020/04/05(日) 17:29:57.31ID:HwkRF7us
>>67
回答ありがとうございます
説明が下手で申し訳ございません
どこか言って頂ければ補足します
よろしくお願いします
後、8C4ではなく、8C3としたらダメなのか?
が正しいです
0070132人目の素数さん2020/04/05(日) 17:39:22.02ID:WyYvb5xI
どこというか... どうイメージしたらいいのか分かりませんでした.
いえこちらの理解力が不足しているだけなのですが,
できれば軽く絵を描いてもらえると助かります.
0071132人目の素数さん2020/04/05(日) 17:50:30.70ID:2F4ElAOS
はよせい(`_´)
0072132人目の素数さん2020/04/05(日) 18:08:56.66ID:b8VDsJ1S
すみません 初めてなので下手ですが。。
AとC'とDの線でできた正三角形を基準に考えるんだと思います
間違えてたらすみません
0073132人目の素数さん2020/04/05(日) 18:09:20.40ID:D9Ap+iGK
>>56
ありがとうございます。
中学3年生はx<y⇒√x<√yを使っていますが、証明なしで使うのは良いと思いません。
しかし証明は中学3年生には難しいと思います。
0074132人目の素数さん2020/04/05(日) 18:18:20.49ID:b8VDsJ1S

0075132人目の素数さん2020/04/05(日) 20:20:09.24ID:WyYvb5xI
>>74 やっと理解できました. B の置き場に困っていたのでしょうか.
それならもう一段足せば良いでしょう.
そして C, D から斜め上に伸ばす直線の方向を1回だけ変えるのです.
サイズ k の正置正三角形に対して B の置き場が k 個できます.
図で分かると思います. ここから C[9,4] が浮かんで来ます.
0076132人目の素数さん2020/04/05(日) 20:42:46.76ID:863BIVJo
>>60
いちいち新たにlog(1-x^2)の微分の計算などせずとも、
先に作っておいたlog(1+x)の級数があるのだから-x^2で置換して利用しろってことでしょ
ラグランジュの剰余項なら、0<θ<1として、Rn(x)=(-1)^(n-1)x^n/n/(1+θx)^nだから、
xが負のとき、│Rn(x)│<(-x)^n/n/(1+x)^n=(-x/(1+x))^n/n
xが正のとき、│Rn(x)│<x^n/nだから、-1/2<x<1ならばRn(x)→0
0077132人目の素数さん2020/04/05(日) 21:41:44.50ID:pXbdSZye
>>76
ありがとうございました
0078132人目の素数さん2020/04/05(日) 23:52:28.85ID:uLmJy8de
>>75
一辺の8コずつの点で8C3と考えた時、9C4と比べて作れない三角形って具体的にどういうのがありますか?
0080132人目の素数さん2020/04/06(月) 01:14:07.56ID:yWb+5QFo
>>78
3段増ではなく 1段増で考えて 底辺の全 8 黒点から A, B, C を拾います.
Aから右上, Bから右上, Cから左上方向に直線を伸ばします.
そこから正置正三角形 (底辺が水平な△)を構成するのは以前と同じです.
全ての正置正三角形がこれで尽くされるのは明らかです.
よってその総数は C[8, 3] = 56 になります.

この場合、逆さまや斜め向きの正三角形はカウントされません.
0081132人目の素数さん2020/04/06(月) 01:42:04.94ID:OeshHNtb
>>80
実際に点を書いてやってみたんですが、8C3と9C4では見かけ上、作れる三角形に差はないように思えてしまうんですが、どこで違いが出てるのでしょうか?
理解が遅くて申し訳ないです
よろしくお願いします
0082132人目の素数さん2020/04/06(月) 02:12:36.15ID:yWb+5QFo
>>81

>>75 にて「 サイズ k の正置正三角形に対して B の置き場が k 個できます. 」
と書きました.

サイズ k の正置正三角形に内接する正三角形 を考えてみましょう.
元の△と合わせて k 個の正三角形が得られます.
逆に斜めや逆さま正三角形に外接する正置正三角形が一意に決まることは明らかです.
よってA,C の間を Bが動くことで 全ての正三角形がカウントされます.
(斜め△は煩雑なので絵には描かなかっただけです)

例えばサイズ4 の場合を見れば内接正三角形の数え方が分かると思います.
0083132人目の素数さん2020/04/06(月) 02:30:43.85ID:91qoKK0b
いきなり!ステーキのスクラッチの当たり確率ハズレ確率のことで揉めています
誰か来てください!
お願いしますm(_ _)m
今の所、下記のスレで>>33から>>118までの議論です

いきなりステーキ Part.23
http://matsuri.5ch.net/test/read.cgi/kbbq/1585980246/33-
0086132人目の素数さん2020/04/06(月) 04:07:09.05ID:H4/4+IEI
>>46
1,1,2だけっぽいね
証明には取り組んでないけどかなり難しそう
0087752020/04/06(月) 09:49:30.13ID:yWb+5QFo
図を書き直してみたら気づきました, Bの置き場を作るには 2段増設のままでもできますね.
C 発の直線だけ折り曲げればいいです.
0088132人目の素数さん2020/04/06(月) 14:14:06.72ID:N+P/EJ2f
>>82>>87
回答ありがとうございます
1段増設でも問題なさそう、と思ってしまってるのですが、2段増設で9C4と1段増設で8C4
(すみません。8C3ではなく、8C4でした。頭が混乱して、色々と訂正してしまい、申し訳ございません)
とでは作れる三角形に違いがないように見えてしまっているのですが、どこで具体的な違いがあるのでしょうか?
0089132人目の素数さん2020/04/06(月) 14:58:41.93ID:yWb+5QFo
>>88
1段増だけでは正置△を一番下に置いた時に Bの置き場が足りなくなります.
例えば
右下にサイズ1 の正置△を置きました. Bを置く余地はゼロです.
左下にサイズ3 の正置△を置きました. Bを置く余地は 2つしかありません.
このままではどうにもならないのです.

あえて意味を見出すなら
サイズ 5 (一辺に 6点) の黒点△から作れる正△の配置数が C[8,4] になります.
0090132人目の素数さん2020/04/06(月) 16:53:11.97ID:fOU0cqsP
>>44
1辺の長さがkの△向きの正三角形内には置けるが、それ未満の大きさの正三角形には向きを変えることなく置けない、かつすべての頂点が三角格子点の上に有る、
そういう条件の正三角形はちょうどk通りだけある

1辺の長さnの△向きの正三角形内に、そのようなk通りの正三角形を置ける位置は、各々C[n-k+2,2]箇所あるので、正三角形の総数は、それらのk=1からk=nまでの総和である
よって正三角形の総数は
Σ{k=1→n}kC[n-k+2,2] @
に等しい。

この式@はC[n+3,4]に等しい。

n=7 のときは C[n+3,4]=210 となる
0091132人目の素数さん2020/04/06(月) 17:02:02.94ID:fOU0cqsP
>>90
なお、正三角形の向きが△しか許されないとすると、正三角形の総数は
Σ{k=1→n}C[n-k+2,2]
となる。この式はC[n+2,3]に等しい

>>44の問題は、正三角形を置く向きに、辺の長さに比例した自由度があるためnの次元かひとつ増えると解釈できる
0092132人目の素数さん2020/04/06(月) 17:51:52.01ID:9VRACs0R
>>44で書かれているような黒点は平面 x+y+z=n 上の非負整数格子点 等として扱うことができます。
この問題の場合は、n=6に相当します。(x,y,z)=(a,b,c)を、“abc”と コンマや括弧を省略して表すと、
28個の黒点には、次のような座標が当てられます
            600
          510   501
        420   411   402
      330   321   312   303
    240   231   222   213   204
  150   141   132   123   114   105
060   051   042   033   024   015   006    (正三角形状になるように、適当にスペースを挿入して下さい)
この問題の答えは、126個で、n=6の時のC[n+3,4]に一致しますが、この、C[n+3,4] といえば、
x+y+z+w+t=n-1 の非負整数解の個数と見ることもできます。
そこで、x+y+z+w+t=n-1 の非負整数解と、黒点三つからなる正三角形を、一対一に対応できないか?
という疑問というか衝動が湧くことは、不思議なことでは無いと思いますが、恐らく、
x+y+z+w+t=n-1 の非負整数解(x,y,z,w,t)=(a,b,c,d,e)に対し、
三点 P(a+d+1,b+e,c),Q(a,b+d+1,c+e),R(a+e,b,c+d+1) で構成される正三角形を当てれば、よいと思われます。
ちなみにこの三点は、正置な正三角形 P'(a+d+e+1,b,c),Q'(a,b+d+e+1,c),R'(a,b,c+d+e+1)
に、それぞれ、(-e,e,0),(0,-e,e),(e,0,-e) を加えて(=正置な正三角形を回転させて)作ったものです。
これが正しければ、C[n+3,4]のような形で表現できることの説明にもなります。
>>67 さん。関係式ありがとうございました。やはり「必然性」ありましたね。納得です。
0093132人目の素数さん2020/04/06(月) 19:02:57.37ID:9VRACs0R
正置な正三角形だけを数えるなら、月見団子状に積まれた団子の、土台以外の団子が、
正置な正三角形と一対一に対応できます。

土台以外の団子に対し、その団子を直接または間接的に支える、「最小限の土台の団子の固まり」に
当たる三角形がそれです。

方程式 x+y+z+w=n-1 の非負整数解は、四次元空間内で、正四面体状に配置された格子点にあたります。

方程式 x+y+z+w=n-1 の解 (x,y,z,w)=(a,b,c,d)に対し、
(a+d+1,b,c),(a,b+d+1,c),(a,b,c+d+1) を当てれば、これらは必ず正置な正三角形になります。
0094132人目の素数さん2020/04/06(月) 20:57:32.17ID:RXByCzyL
はよせい(`_´)
0095132人目の素数さん2020/04/06(月) 21:14:23.92ID:Xd2raitW
1+i/10≦log(x)≦1+(i+1)/10
を満たすxの最小値をm_i、最大値をM_iとする。

(1)i=0,1,...,9について、M_i - m_iの値はすべて相異なることを示せ。

(1)iは0以上9以下の整数とする。
iがこの範囲を動くとき、M_i - m_iを最大とするiを求めよ。

(2)(1)と同様に、M_i - m_iが4番目に大きくなるようなiを求めよ。
0096132人目の素数さん2020/04/06(月) 22:29:47.49ID:QHMQIa3H
>>90
回答ありがとうございます
理解できました。BとCをBとC'のように考えていたのがダメだったという事ですね
助かりました
ありがとうございました
0097132人目の素数さん2020/04/06(月) 22:31:17.16ID:QHMQIa3H
>>89
回答ありがとうございます
理解できました。BとCをBとC'のように考えていたのがダメだったという事ですね
助かりました
ありがとうございました
0098132人目の素数さん2020/04/06(月) 22:31:53.02ID:QHMQIa3H
すみません。間違えて投稿してしまいました
0099132人目の素数さん2020/04/06(月) 22:37:00.35ID:QHMQIa3H
>>89〜93
改めまして、回答ありがとうございます
おかげ様で、理解できました

平面上の格子点として、捉えて考えるのは僕にはちょっとまだ無理そうです…

沢山の方に協力頂きまして、ありがとうございました!
0100132人目の素数さん2020/04/07(火) 00:50:04.42ID:kkcretXZ
Mathpixはwolframより使える
0102132人目の素数さん2020/04/08(水) 00:42:42.60ID:08WwaJiN
f(x)=-x(x-2)
a[0]=a, a[n+1]=f(a[n])
とする。
以下の条件をすべて満たす実数aが存在することを示せ。
・任意の非負整数nに対して0≦a[n]≦1
・0≦p<q≦1であるどのような(p,q)に対しても、不等式p<a[k]<qを満たすある非負整数kが存在する。
0103132人目の素数さん2020/04/08(水) 03:54:38.67ID:ZohoKp5e
方程式x=f(x)を解くのに、これをy=xかつy=f(x)という連立方程式と見て、
x=aという近似値から、(a,0)→(a,f(a))→(f(a),f(a))→(f(a),0)=(a[0+1],0)
と動く動点の運動と考えて逐次的に作られた近似値の列としてa[n]を考える
ただし、区間を@x<0、A0<x<1、B1<x<2、C2<xの四つの場合で振る舞いが変わる
@a<0の場合、a[0+1]-a=-a^2+2a-a=-a(a-1)<0より、@から@にどんどん進む
A0<a<1の場合、0<-a(a-2)=a[0+1]=-(a-1)^2+1<1なのでAからAに進む
B1<a<2の場合、0<-a(a-2)=a[0+1]=-(a-1)^2+1<1なのでBからAに進む
C2<aの場合、a[0+1]=-a(a-2)<0だから、Cから@に進む
なので、例えばa=(p+q)/2とすれば、A0<a<1だからこれ以降のa[n]も常にAとなり成り立つ
0104132人目の素数さん2020/04/08(水) 14:37:01.68ID:pDfrzDrp
a[0]=0,2 のとき a[n]=0(n≧1),
a[0]=1 のとき a[n]=1,

0 < a[0] < 2 のとき
 0 < a[n] < 1(n≧1)
a[n+1]- a[n]= f(a[n])- a[n]
 = a[n](2-a[n])- a[n]
 = a[n](1-a[n])
 > 0,
∴ n≧1 で a[n]は単調増加。
 p = 0.9a[1]+ 0.1a[2],
 q = 0.1a[1]+ 0.9a[2],
とおけば
 p < a[k]< q,
を満たす非負整数kは存在しない。

なお、n→∞ のとき
 a[n]= 1 - |1-a[0]|^(2^n) → 1
0105132人目の素数さん2020/04/08(水) 22:28:56.94ID:08WwaJiN
y=e^xはx→-∞でほぼx軸に平行、x→∞でほぼy軸に平行とみなせるので、傾き-1のある直線に関して左右対称とみなせると考えたのですが、「左右対称とみなせる」をどう数式で表したらいいか分かりません
0106132人目の素数さん2020/04/09(木) 01:53:26.45ID:pbzhwLWN
>>65
ハンガリーのレーニ(A.Renyi,1921〜1970) が「大きな篩い」を使って示した。(1947)

中国の陳景潤(1933〜1996)の定理は、じゅうぶん大きいすべての偶数について k=2 の定理が真であることを示す。(〜1978)

k=1の場合は ゴールドバッハの予想である。(未解決)
0108132人目の素数さん2020/04/09(木) 11:31:44.54ID:MyyKX9BA
玉木のホモトピー論のノートで勉強していてわからないことがあるので教えてください

連続写像f:X→Yに対して写像跡 E_fを
E_f={(x,ω)∈X×Map(I,,Y) | f(x)=ω(0)}として定めると
p(x,ω)=ω(1)という写像E_f→Yがfibrationになり
さらにi(x)=(x,c_f(x))という写像X→E_f(ただしc_f(x)はf(x)での定値写像からなる道)と
r(x,ω)=xという写像E_f→Xに対してiとrはホモトピー逆写像であるという定理があります(ここまでは理解できました)
その際にi*rとid_E_fの間のホモトピーHをp*H=pr_1*(p×id_I)という可換性が成り立つようにとれるという主張があるのですが
(*は合成です)
これはIで0のとき、つまりH=i*rの時にすでに成り立たないので主張としては偽だと思います
これをどこか修正して正しい内容にすることはできないのでしょうか?

というのも、その定理の系として元々のfがすでにfibrationである時に
fとp:E_f→Bとがファイバーホモトピー同値という主張があり、その証明に可換性を使っているのです
この系が成り立つのかも含めてわかる方いたら教えてください

該当箇所は下の37p(ノート内のページとしては33p)です
http://pantodon.shinshu-u.ac.jp/downloadables/notes/fibration.pdf
0110132人目の素数さん2020/04/09(木) 15:54:47.51ID:XD28pmTr
a≦b<cとして、右辺=c^c≧(b+1)^(b+1)=(b+1)*(b+1)^b>(1+1)*(b+0)^b=2b^b≧a^a+b^b=左辺
0112132人目の素数さん2020/04/09(木) 16:33:41.08ID:XD28pmTr
1^1+2^3=3^2
0114132人目の素数さん2020/04/09(木) 19:14:36.75ID:kHWiZvsi
すいません、質問です
プログラムを書いていて関数を作る必要が出てきたんですが、
数学がさっぱりで考えてもわからなかったのでこちらで聞かせてください

例えば0〜100までの間に分布した数を、
50〜90までに置き換えたい場合どういう計算式を書けばいいのでしょうか?

文章での質問で伝えられる自信がないので図を書いてみました

https://i.imgur.com/F6O9hzM.png

どなたかよろしくお願いします
0115132人目の素数さん2020/04/09(木) 19:31:08.39ID:XD28pmTr
0.4x+50
0116132人目の素数さん2020/04/09(木) 19:39:20.71ID:pbzhwLWN
1次式では
 After = 50 + 0.4・Before,

ほかに
 After = 50 + 40・(Before/100)^n,
 After = 50 + 40・f(Before/100)^n, f(0)=0,f(1)=1,単調増加
などもある。
0117132人目の素数さん2020/04/09(木) 20:00:51.22ID:kHWiZvsi
>>115
>>116
ありがとうございます
なるほど、全体の何割を占めるかを考えてから、AFTERの最小値を上乗せすればいいのですね

Beforeの最大値は仮に100として図を書きましたが、
最大値がわからないけど(少なくとも90以上)、50〜90の間に値を収めたい、
という場合だと、もはや数式を書くことは不可能になりますか?
01181172020/04/09(木) 20:02:22.68ID:kHWiZvsi
あ、AFTERの最大値が100を超える場合のことも考慮したいです、補足です
01191172020/04/09(木) 20:06:50.09ID:kHWiZvsi
あ、すいません!自己解決です!
こちら側で最大値を取得する方法を思いつきました

ですので
>>115
>>116
さんの回答で解決です
ありがとうございます
0120132人目の素数さん2020/04/09(木) 20:26:54.40ID:117kXQFH
https://youtu.be/QuZL5IKpO_U?t=1538

A を可逆な n 次行列とする。
↑の動画の解答で、 A の固有値を λ_1, …, λ_n とするとき、 A^(-1) の固有値 は 1/λ_1, …, 1/λ_n になるということを証明なしに使っています。

λ が A の固有値であるとき、 1/λ は A^(-1) の固有値になるということは簡単に分かりますが、固有値に重複がある場合に、
重複度まで一致することは自明なことでしょうか?
0122132人目の素数さん2020/04/09(木) 21:02:24.01ID:pbzhwLWN
a^b + b^a = c^c, a>1, b>1
を満たす自然数a,b,cは存在しないことを示せ。
0124132人目の素数さん2020/04/09(木) 22:08:10.93ID:nXrodH2C
>>120
B := A^{-1} とおく.
det( xI - B ) = (x-λ1')(x-λ2')...(x-λn')  {Bの特性多項式 }
= det( xBA - (x/x)B )
= |B| * x^n * det( A - (1/x) I )
= |B| * (-x)^n * det( (1/x) I - A )
= 1/|A| * (-x)^n * (1/x - λ1)(1/x - λ2)...(1/x - λn)   {Aの特性多項式 }
= 1/|A|*(λ1λ2...λn) * (x - 1/λ1)(x - 1/λ2)...(x - 1/λn)
= (x - 1/λ1)(x - 1/λ2)...(x - 1/λn)
∴ λ1' = 1/ λ1, ..., λn' = 1/ λn
0125132人目の素数さん2020/04/09(木) 22:40:07.66ID:nXrodH2C
>>123
任意の x ∈ V に対して
[射影分解の存在]
x = (P1+...+Pn)(x) = x1+...+xn ( xi := Pi (x) ∈ Ui )
[分解の一意性]
任意の分解 x = x1'+...+xn' (xi' ∈ Ui) に対し,
ImPi = Ui より Pi(yi) = xi' となる yi ∈ V が存在する.
Pi( x1'+...+xn' ) = Pi( x )
Pi( P1(y1)+...+ Pn(yn) ) = xi
0+..+0+ Pi(yi) +0+..+0 = xi {射影子の性質 PiPi = Pi は 1,2 から導出可能}
∴ xi' = xi
0126132人目の素数さん2020/04/10(金) 08:07:01.33ID:eQpuZqsr
平面上にどの3点も同一直線状にない5点が与えられたときに
5点を通る二次曲線が一つ決まりますが、楕円、双曲線、放物線のどれになるのかを
配置から幾何的に判定する方法があれば教えてください
0127132人目の素数さん2020/04/10(金) 10:14:40.07ID:qSJe5D1l
群数列の問題で
1,3,3^2,3^3,…3^k(k=1,2,3…)という数列を考える。
1,3,1,3,3^2,1,3,3^2,3^3…

初項から第n項までの和をSnとするときSn≦555を満たす最大のnを求めよ。

解答が1枚目、僕の答えが2枚目です、
3(3^m -1)はダメでしょうか?

https://i.imgur.com/Sl9ScJG.jpg

https://i.imgur.com/Wa9wsdc.jpg
0128132人目の素数さん2020/04/10(金) 13:59:49.42ID:iHWrZjYa
>>127
とりあえず メモでは
  Σ[k=1,m] 3^k = ( 3^{m} - 1 ) / 2 になってますが間違ってますね.

Σ[k=1,m] 3^k = ( 3 + 3^2 + ... + 3^m ) (3-1) / (3-1) = ( 3^{m+1} - 3 ) / 2
和公式については その導出過程を記憶して,
最終結果に自信がないときは m=1 とかで検算したらよいと思います.
0130132人目の素数さん2020/04/10(金) 18:06:23.99ID:O/jqlpQJ
xを実数とし、-1<x<1の範囲でf(x)=(-2)^xを考える。
f(x)の実部と虚部の値が一致するようなxがちょうど2つ存在することを示し、それらを小さい方からs,tとしたとき
∫[s,t] Re(f(x)) dx
の値を求めよ。
0131132人目の素数さん2020/04/10(金) 18:45:12.35ID:IAsBrfBV
f(x) = (2^x)(-1)^x
 = (2^x)exp(iπx)
 = (2^x){cos(πx) +isin(πx)}

g(x) = cos(πx) - sin(πx)
  = sin(π(x +1/2)) - sin(πx)
  = 2sin(π/4)cos(π(x +1/4))
  = (√2)cos(π(x +1/4))
g(x)=0, -1<x<1 より
 x + 1/4 = ±1/2,
 s = -3/4, t = 1/4,

∫[s,t] f(x)dx
 = 1/{log(2)+iπ} [ f(x) ](x=s,t)
 = 1/{log(2)+iπ} {f(t) - f(s)}
 = 1/{log(2)+iπ} {f(1/4) - f(-3/4)}
 = (1+i)/{log(2)+iπ} {3/2^(5/4)},
これの実数部をとる。
0133132人目の素数さん2020/04/10(金) 19:20:12.93ID:qSJe5D1l
>>128
等比数列の和の公式の初項をかけるのを忘れていました、ありがとうございます。
0136132人目の素数さん2020/04/10(金) 23:26:00.39ID:ODZxMp5A
>>124

ありがとうございました。
0137132人目の素数さん2020/04/10(金) 23:37:57.05ID:O/jqlpQJ
古典力学の仮定のもとでは、重力加速度gは無理数であることを証明せよ。
0138132人目の素数さん2020/04/11(土) 05:58:16.38ID:QxjOJ3hV
一いち十じゅう百ひゃく千せん万まん億おく兆ちょう京けい垓がい世よ穣じょう溝こう澗かん正せい載さい極ごく

垓の次は世(よ)

と表現して何か問題あるでしょうか?
問題なければこのまま提唱したいと思います。
0139132人目の素数さん2020/04/11(土) 06:07:14.90ID:auEc/TWp
>>137
無理数なのははじめてきいた。
考えたこともなかったな。
光速は有理数??
0140132人目の素数さん2020/04/11(土) 06:37:31.45ID:WabTNdUl
>>138
万 億 兆 京 垓 𥝱また秭 穣 溝 澗 正 載 極 恒河沙 阿僧祇 那由他 不可思議 無量大数
0141132人目の素数さん2020/04/11(土) 06:45:24.83ID:QxjOJ3hV
>>140
二択である未解決問題の解消方法で提唱した。
二択状態のままでは解決しない。
0145132人目の素数さん2020/04/11(土) 06:57:55.49ID:QxjOJ3hV
JIS第一第二水準漢字範囲内でよいはず。
垓は一般に周知されていますからそのままでもいいと思われます。
0146132人目の素数さん2020/04/11(土) 10:20:58.20ID:AJ7O9J83
Sheldon Axler著『Linear Algebra Done Right 3rd Edition』を読んでいます。

V を F 上の有限次元線形空間とし、 V' を V の双対空間とする。
φ_1, …, φ_n を V' の基底とする。

このとき、 V の基底で、その双対基底が φ_1, …, φ_n であるようなものが存在することを示せ。
0147132人目の素数さん2020/04/11(土) 11:06:54.11ID:jGybxHRX
>>139
光速を単位とすれば整数
0148132人目の素数さん2020/04/11(土) 11:14:24.57ID:pXenprnA
>>130
f(x)=(-2)^x = 2^x e^{iπx}  {複素平面上の対数螺旋}
= e^{ (log2 + iπ) x} = e^{ax}
グラフより
πs = -π + π/4 ∴ s= -3/4
πt = +π/4 ∴ t= +1/4

∫dx [x=s,t] f(x)
= (1/a)( f(t) - f(s) )
= (1/a)( 2^{1/4}+2^{-3/4} ) e^{iπ/4}
= (1/a)( 2^{1/4} (1+ 1/2) ) 2^{-1/2} (1 + i )
= (log2 - iπ) / ((log2)^2+π^2 ) * (3/2 )*2^{-1/4}* (1 + i )
= (3/2 )*2^{-1/4}*(log2 + π) / ( (log2)^2+π^2 ) + i 〜
= 0.4673... + i 〜

>>131 も f(t)-f(s) = (|f(t)|+|f(s)| ) e^{iπ/4} ∝ (1 + i) を利用してるようだが
これはグラフ書いたほうが分かりやすいと思う.
0149132人目の素数さん2020/04/11(土) 11:28:31.74ID:Os6hWAHD
>>126 >>129 境界が放物線になりそうだから簡単そうなのから証明抜きで調べると

平面上に凸四角形があるとき4つの頂点を通る放物線が何個あるかを考える
凸四角形が平行四辺形⇒放物線はない ⇒5点目を凸五角形になる位置どこにおいても楕円
凸四角形が台形⇒放物線は1つ⇒放物線を境界として楕円と双曲線に分かれる
それ以外     ⇒放物線は2つ⇒2つの放物線を境界として楕円と双曲線に分かれる
0150132人目の素数さん2020/04/11(土) 11:57:52.37ID:pXenprnA
>>146
Vの基底 {e_i} を任意に採り, 行列Aを以下のように定義する.
A[ij] = (φ_i・e_j) = (φ_i・e_1,  φ_i・e_2,  ...,  φ_i・e_n)
もし行列Aが非正則なら 列成分ベクトルの間に一次従属関係が成り立つ.
 Σ[j] c[j]* (φ_i・e_j) = 0 (あるjにおいて c[j]≠0)
φ_i・(Σ[j] c[j]* e_j) = φ_i・v = 0 (v≠0)
これは矛盾である. {∵ φ・v ≠ 0 となる一時形式 φ の存在}
よって逆行列 A^{-1} が存在する.

f_i := A^{-1}[ji] e_j  と定義する.
基底の取り替えになるので {f_j} がVの基底を成す事はあきらか.

φ_i・f_j = A^{-1}[kj]* (φ_i・e_k) = A^{-1}[kj]* A[ik] = δ[ij]
つまり {f_i}の双対基底は {φ_i} である.
0154132人目の素数さん2020/04/11(土) 17:53:21.57ID:jVXfLHUH
5点 (0,0) (±1,1) (±a, 2) を通る2次曲線は何でしょう?
 |a|<√2, |a|=√2, |a|>√2,
0156132人目の素数さん2020/04/12(日) 03:58:02.03ID:VviYpdPw
準線x=l(l<0), 焦点(f,0)(f>0)を仮定できないのがつらい……
0157132人目の素数さん2020/04/12(日) 04:26:11.86ID:g/DVEXjN
>>137
一般相対論のもとではどうでしょうね?

高精度の時計(光格子時計)で計測したところ、スカイツリーの展望室では地面よりも
1日(8.64×10^4 s)あたり 4.26×10^(-9) s だけ速く時間が進んだ。
くわしい研究によれば、この比は (1+gh/cc) に等しい。
 重力加速度 g = 9.79 (m/ss)
 光速度 c = 299792458 (m/s)
として標高 h (m) を求めよ。          (h = 452.6 m)
0159132人目の素数さん2020/04/12(日) 05:43:07.81ID:2pxLR1cr
そもそも証明できるものなのか
地上でも場所によって多少変化があるのを考慮すれば中間値の定理的に有理数も無理数も取りうると思うんだが
0160132人目の素数さん2020/04/12(日) 12:25:35.63ID:Q4jlLAGj
限りなく0に近い確率で有理数となるがだいたい無理数
0161132人目の素数さん2020/04/12(日) 13:47:50.80ID:g/DVEXjN
>>154

| xx yy y |
| aa  4 2 | = 2xx - (aa-2)yy + (aa-4)y = f(x,y) = 0
| 1  1 1 |

f_x = f_y = 0 より
中心 (0, (4-aa)/(4-2aa))    |a|≠√2
0162132人目の素数さん2020/04/12(日) 14:30:06.38ID:g/DVEXjN
>>139
光速は偶数で
 c = 299792458 m/s = 2・7・73・293339 m/s

 国際度量衡総会(1983)の決議

産技総研・計量標準総合センター、もしくは将棋連盟の谷川九段まで?
0163132人目の素数さん2020/04/12(日) 15:07:42.47ID:g/DVEXjN
>>126
 I.ニュートン(1642-1727)の「プリンシピア」に5点をとおる円錐曲線の作図法がある。
 方ベキの定理(の変形)を活用した。
0164132人目の素数さん2020/04/12(日) 17:52:40.99ID:yHJL3tga
↓この事実の証明を教えて下さい。


A を正則な対称行列とする。
A の正の固有値の数と正のピボットの数は等しい。
0166132人目の素数さん2020/04/12(日) 20:36:40.07ID:yHJL3tga
↓を証明してください。

A の固有値がすべて実数である ⇒ A は対称行列である。
0167132人目の素数さん2020/04/12(日) 20:37:38.75ID:yHJL3tga
>>166

明らかに成り立ちませんね。
ストラングさんは間違っていますね。
0168132人目の素数さん2020/04/12(日) 21:07:02.50ID:1XrENEBG
>>164,166, 167
数学の本に居着いていてる基地外です
相手をしないように
0170132人目の素数さん2020/04/12(日) 22:53:34.84ID:g/DVEXjN
>>166
A の固有値がすべて実数 ⇔ A はエルミート行列(実対称行列も含む)

(略証)
xの転置共役x' とyの内積を (x,y) とする。
(x, Ax) = (x, λx) = λ(x, x)
( Ax, x) = ( λx, x) = λ’(x, x)
より
Aの固有値がすべて実数 ⇔ λ = λ' ⇔ (x, Ax) = (Ax, x) ⇔ Aはエルミート
0171132人目の素数さん2020/04/13(月) 02:19:17.64ID:PNtjkUIN
佐武一郎「行列と行列式」裳華房 (1958) p.168
 IV章、§5、例1
* 「線型代数学」(1974) に改題する前のもの。

コメント
固有値が実数だの、ヴェクトル・空間が直交するだのは、
内積空間(距離空間)から来たウィルスだろう。
その元の正規行列もノルム空間から来たもんだ。
本来の線型代数と無縁のことにページを割くのは、
軒を貸して母屋を取られてるんぢゃないか?
               ぬるぽ
0172132人目の素数さん2020/04/13(月) 11:32:11.38ID:qKd76N34
数学記号カードゲームってどんなものがあるのかな?
0174132人目の素数さん2020/04/13(月) 17:45:54.28ID:5Rb5Sin6
ライフゲームを創ったコンウェイが亡くなったとのこと
0175132人目の素数さん2020/04/13(月) 18:38:31.33ID:Oetq8pjs
>>174
> ライフゲームを創ったコンウェイが亡くなったとのこと

情報ありがとう
コンウェイは武漢肺炎に殺されたのか、志村けんなんかよりずっとショックだ
これでずっと噂されていた三角形本(The Book of Triangles)が出版される可能性はゼロになっちゃったんだなあ
0176132人目の素数さん2020/04/13(月) 20:28:35.07ID:qtt1gsZV
足立幸信さんのHPやついが去年の10月から更新されてないのですが
どうされたのでしょうか
0177132人目の素数さん2020/04/14(火) 00:08:03.40ID:X4cWnq7v
(0,0),(4,0),(4,4),(0,4)を4頂点とする正方形内の各格子点に人が立っている。
これら16人のうち、2人が若者で、14人は老人である。
時刻t=0において、(0,0)に立っている人がウイルスCに感染した。各時刻t=i(i=0,1,2,...,8)において、Cは感染者の隣接格子点に立つ人にも感染する(隣接格子点の数は3または4である)。
感染確率は隣接格子点に立つ人が若者の場合1/4、老人の場合3/4である。

(1)若者が(0,0)および(4,4)に立っているとする。時刻t=8において、点(4,4)に立っている人が感染者となる確率を求めよ。

(2)若者が(1,3)および(3,1)に立っている場合はどうか。

(3)(4,4)に立つ人が感染者となる確率が最も低くなるように若者を配置したい。その方法と理由を簡単に述べよ。
0178132人目の素数さん2020/04/14(火) 01:01:10.72ID:PHHJv41k
すみません。
こういうフライパンがあるんですが、宅急便送料を知りたいので、近似でタテ、ヨコ、高さ、の合計値を出したいと思います。

外径26×高さ7.7×最大長46.5cm・底厚4.7mm以上、ハンドル取外し時:最大長33.5cm

ハンドル取り外したときのサイズが知りたいのです。
○に棒が付いた虫眼鏡のような形で棒の太さはゼロとして近似して計算しようと思いました。
つまり、外径26cmの○に、(33.5cm-26cm)=7.5cmの棒が付いている物のタテ、ヨコ、高さ、の合計値の最小を目安として計算したいのです。

よろしくお願いします。
0180132人目の素数さん2020/04/14(火) 03:43:08.30ID:ZGzdde0B
棒の向きに33.5cm、直角方向に 26cm の長方形の箱だと、
 タテ+ヨコ = 33.5 + 26 = 59.5 cm
棒の向きと45°向きの正方形の箱だと
 タテ+ヨコ = (33.5 - 26/2)√2 + 26 = 55.0 cm
なので、正方形の方が短くなる。
高さ 7.7 cm をたすと 62.7 cm
0182132人目の素数さん2020/04/14(火) 10:54:43.57ID:PHHJv41k
>>180
ありがとうございます。自分でも計算してみたのですが、
その二つの場合のほかに、
短い辺が26pで、長い辺に棒をくっつけたとき、45度よりも少し小さい角度の時に、
長い辺は、
13+√((13+7.5)^2-13^2) = 28.85cm
タテヨコ高さ62.55cmとなるのではないでしょうか?

しかし、それが最小であるというのはどうやってやるのでしょうか?
微分で傾きがどうのこうのやった記憶があるのですが、20年以上も前なので忘れてしまいました。
0183132人目の素数さん2020/04/14(火) 11:05:43.01ID:Cd9JavFt
実際に計った方が早くね?
棒の太さをゼロで考えてそんなに細かいところまで計算するのは妥当でないと思うし
0185132人目の素数さん2020/04/14(火) 14:40:31.28ID:s2XHPXPF
>>184
こういう風に広がるシミュレーションプログラムを作ってみた。

https://i.imgur.com/eatlhhh.png

t=8 で(0,0)から(4,4)に感染が広がる割合は

> (re=mean(replicate(1e4,f25())))
[1] 0.3331
> try(mean(replicate(1e5,f25())))
[1] 0.33068

となったので約1/3という結果が得られた。

数理解は賢人にお任せ。
0186132人目の素数さん2020/04/14(火) 15:00:27.36ID:s2XHPXPF
>>185
4×4のグリッドだと

対角線上の格子点が感染する確率は

> (re=mean(replicate(1e4,f25())))
[1] 0.7842

と近似解がでてきた。
0187132人目の素数さん2020/04/14(火) 15:21:47.33ID:2JuTwYG1
0={1,2,3} 1={0,4,7} 2={0,5,8} 3={0,6,9} 4={1,8,9}
5={2,7,9} 6={3,7,8} 7={1,5,6} 8={2,4,6} 9={3,4,5}

0={1,2,3} は左辺が1,2,3の{}内に0が含まれていることを表す
1={0,4,7}は1が0,4,7に含まれているetc
今は10個の数を3つの数でコンシステントに定義できましたが
n個の数をm個の数で同様に矛盾なく定義可能となるn,mを全て求めよ
0189132人目の素数さん2020/04/14(火) 18:34:26.59ID:/zaiyYnD
頂点の数nでかつ各頂点に接続する辺の数がすべてmであるような、多重辺のないグラフを作れるn,mの組み合わせを求めよ
ってことじゃね
0190132人目の素数さん2020/04/14(火) 23:02:34.60ID:nLKpvUH3
>>189
四面体: (n,m)=(4,3) , 立方体: (8,4) , ... みたいな話ですか?
それでも >>187 の記法はよく分かりません。
0192132人目の素数さん2020/04/15(水) 13:32:50.18ID:YqRVzpwQ
整数mについての方程式
arctan(m-1)+arctan(m)+arctan(m+1)=nπ
の実数解が存在する非負整数nをすべて決定し、それらのnに対する解を全て求めよ。
ここでg(x)=arctan(x)はf(x)=tan(x)の逆関数であり、その定義域は-π/2<arctan(x)<π/2である。
0193132人目の素数さん2020/04/15(水) 14:55:04.50ID:3hiRwpzE
>>192
tanの和公式を二回つかって
tan(a + b + c) = { tan(a) + tan(b+c) }/{ 1 - tan(a) tan(b+c) } = ...
 = { t(a) + t(b) + t(c) - t(a) t(b) t(c) }/{ 1 - t(a) t(b)- t(b) t(c)- t(c) t(a) }
を得る.
これより
{ (m-1) + m + (m+1) - m(m-1)(m+1) } / { 1 - m(m-1) - m(m+1) - (m-1)(m+1) } = tan(nπ)
m(4 - mm) / ( 2 - 3mm ) = 0  ∴ m(m+2)(m-2) = 0
・m = 0 の場合
  atan(-1) + atan(0) + atan(+1) = 0 より n=0 である.
・m = +2 の場合
  0 < atan(1) + atan(2) + atan(3) < (π/2)*3 より n=1 である.
・m = -2 の場合 n < 0 になるので 不可 .

よって (n,m) = (0, 0), (1, 2) これが全ての解である.
0194132人目の素数さん2020/04/15(水) 18:02:59.39ID:yFSBbZr+
「ごちそうさまですと言え。」
何様が命令しているんだ。ふざけんな!お前らのように盗聴しかできないガキに調子に
乗られたくないわ

お前は何の権限でおれに命令しているんだ。私は47才だ。
0196132人目の素数さん2020/04/15(水) 18:07:57.82ID:yFSBbZr+
「名前を逆にしたからだ!。」と聞こえてきました。
名前を逆にしたのは、政治が勝手に決めたことですし、それに民間人の私が従う法律もありません。

勘違いはいい加減にしろよ!
0197132人目の素数さん2020/04/15(水) 18:43:36.95ID:OBrsEksp
>>193
g(x) = arctan(x) は単調増加で -π/2 から π/2 まで動く。
与式もmについて単調増加で -3π/2 から 3π/2 まで動く。
∴ それがすべての解。
0198132人目の素数さん2020/04/15(水) 19:16:54.93ID:hs0U7P9m
つまり、隣辺が1:2の直角三角形と1:3の直角三角形の角をうまく組み合わせると3π/4やπ/4の角を作れるわけね
面白いね
0199132人目の素数さん2020/04/15(水) 19:19:31.97ID:aWYwPEMa
以下の連立方程式を解け。
(x+y)(x^2+y^2)=2(x^2+1)(y^2+1)
(x^2+y^2)(x^4-y^4)=2{(x^2+1)(y^2+1)}^2
0201132人目の素数さん2020/04/16(木) 00:24:29.97ID:mpjyJ+WS
仕事終わったら家に帰る、もしくはネカフェに泊まる
定期はどうやって買うんの

落合南長崎駅から新宿三丁目駅(新宿で乗り換える)
落合南長崎駅から八潮駅(都庁前と新御徒町駅で乗り換える)

パターン1
月9日は家に帰る
パターン2
仕事が遅いときだけネカフェに行く
パターン3


難しい計算だ
落合南長崎から都庁前の定期を買っても
落合南長崎駅から八潮駅の価格が変わらなさそう
0202132人目の素数さん2020/04/16(木) 00:56:42.61ID:dG0TI+mF
500mlにアルコール濃度96%の飲み物があります。

その飲み物 90ml +水 20ml =アルコール濃度78.5%
その飲み物 170ml +水 40ml =アルコール濃度77.7%

の場合、アルコール濃度60%にするには
飲み物と水をどう配合したら良いでしょうか。
0204132人目の素数さん2020/04/16(木) 02:44:23.15ID:Fekx2b8P
>>199
与式を
f(x, y) = (x+y)(xx+yy) - 2(xx+1)(yy+1) = 0,
g(x, y) = (xx+yy)(x^4-y^4) - 2{(xx+1)(yy+1)}^2
   = (xx+yy)^2 (x+y)(x-y) - 2{(xx+1)(yy+1)}^2
とおく。
0 = 2g(x, y) - {(x+y)(xx+yy) + 2(xx+1)(yy+1)}f(x, y)
 = (xx+yy)^2 (x+y){2(x-y)-(x+y)}
= (xx+yy)^2 (x+y)(x-3y),
∴ xx+yy=0 または x+y=0 または x-3y=0.

・xx+yy = 0 のとき
 f(x, y) = -2(xx+1)(yy+1) = 0 から
 (x, y) = (-1, ±i) (1, ±i) (±i, -1) (±i, 1) 8個

・x+y = 0 のとき
 f(x, y) = -2(xx+1)(yy+1) = 0 から
 (x, y) = (i, -i) (-i, i) 2個

・x-3y = 0 のとき
 f(3y, y) = 40y^3 - 2(9yy+1)(yy+1)
  = -2(y-1)(9y^3 -11yy -y-1) = 0,
 ・ y-1=0 から (x, y) = (3, 1) 1個
 ・ 9y^3 -11yy -y-1 = 0 から
 (3b, b) (3c, c) (3c~, c~) 3個
  b = {11 + 3(2870/27 - 2√1713)^(1/3) + 3(2870/27 + 2√1713)^(1/3)}/27
   = 1.36347986
  c = (11/9 - b)/2 + 0.27659068i
   = -0.07062882 + 0.27659068i
0205132人目の素数さん2020/04/16(木) 02:53:19.73ID:Fekx2b8P
>>199
x, y を実数に限れば
 xx+yy > 0, xx+1 ≧ 1, yy+1 ≧ 1 だから
 (x,y) = (3,1) (3b, b) の2個
  b = {11 + 3(2870/27 - 2√1713)^(1/3) + 3(2870/27 + 2√1713)^(1/3)}/27
   = 1.3634798606776
0208イナ ◆/7jUdUKiSM 2020/04/16(木) 14:27:26.19ID:SCwY7gJQ
>>32
>>202
78.5%っていうのは実際に計算してみると、
90×0.96÷(90+20)=86.4/110
=0.785363636……(%)
77.7%っていうのは実際に計算してみると、
170×0.96÷(170+40)=163.2/210
=0.777142857142857……(%)
のことである。問題はこれでいい。これを踏まえ、
その飲み物xmlを水30mlで希釈したらアルコール60%になったとして、
x×0.96÷(x+30)=0.6
96x=60(x+30)
36x=1800
x=50(ml)
∴その飲み物50mlを30mlの水でうすめたらいい。
0209132人目の素数さん2020/04/16(木) 14:42:04.55ID:H2DBY6ro
整数問題で自然数の問題です。

方程式   (X+1)(X-1) = Y^3

ぱっと見た感じ、解として
X=3, Y=2 がみつかります。

●X、Y を自然数とし、 かつ、 Y ≠ 2 とする。

問1
● 「Yが奇数」 である場合、

   (X+1)(X-1) = Y^3

について、

(X+1) は整数の3乗、かつ、
(X-1) は整数の3乗 である

これを証明せよ。
****************************

↑ これが良くわかりません。
Yが奇数の時、 なぜ X+1 と X-1 の両方ともが
「整数の3乗」 であると確定するんですか?

(ちなみに、Yが偶数のときは難しすぎるので無視)
0210132人目の素数さん2020/04/16(木) 15:35:23.56ID:Fekx2b8P
・Yが奇数のとき
 X+1 も X-1 も奇数で その差が2.
 ∴ 互いに素。
 {X+1, X-1} = {A^3, B^3}
 A,B は奇数  gcd(A,B)=1

・Yが偶数のとき
 X+1 も X-1 も偶数。
 (X+1)/2 と (X-1)/2 の差が1
 ∴ 互いに素。
 {X+1, X-1} = {2A^3, 4B^3}
  Aは奇数  gcd(A,B)=1,
0212132人目の素数さん2020/04/16(木) 17:02:44.60ID:dpxw/fVy
三角形ABCの内部に点Oをとり、
(1)AOとBC、BOとAC、COとABの交点を各々D、E、F
(2)FEとAD,FEとBCの交点を各々G、H
(3)OHとAB、OHとACの交点を各々I、J
とする。このときCF、DI、JGは一点で交わることを証明せよ。
(BE、IG、DJも一点で交わる) 
0213132人目の素数さん2020/04/16(木) 17:52:32.53ID:4GCIj5rW
楕円x&sup2;/4+y&sup2;=1上を動く点Pと定点(1,0)の距離の最小値を求めよ。
また,そのときの点Pの座標を求めよ。

3/4(x-4/3)&sup2;-1/3

距離の最小値?『-1/3』の謎を教えてください。答えは当たりました。

割り込みすいません。
0214132人目の素数さん2020/04/16(木) 17:57:34.08ID:4GCIj5rW
すいません。

楕円(x^2)/4+y^2=1上を動く点Pと定点(1,0)の距離の最小値を求めよ。
また,そのときの点Pの座標を求めよ。

3/4(x-4/3)^2-1/3

距離の最小値の2乗?『-1/3』の謎を教えてください。答えは当たりました。

割り込みすいません。
0215イナ ◆/7jUdUKiSM 2020/04/16(木) 20:09:03.29ID:SCwY7gJQ
>>208
>>213-214
x^2/4+y^2=1上の点P(x,y)と(1,0)の距離の2乗は、
(x-1)^2+y^2=x^2-2x+1+1-x^2
=3x^2/4-2x+2=f(x)とおくと、
f'(x)=3x/2-2=0となるのはx=4/3のときで、
y=±√{1-(1/4)(16/9)} =±√5/3
P(4/3,±√5/3)
(1,0)との距離は、
√{(4/3-1)^2+(√5/3)^2}
=√6/3
=0.816496581……
0216132人目の素数さん2020/04/16(木) 21:10:01.28ID:4GCIj5rW
微分するのは目から鱗でしたが、
平方完成しての-1/3は?
0218イナ ◆/7jUdUKiSM 2020/04/16(木) 21:29:24.47ID:SCwY7gJQ
/__/__/__/__/__人人__
/_人人__/_/_(_^_)_
/_(_ )_)_/__/_(__)ヨォ
/_(_( _)_/__/_(^o^))_
/_(^) )_/__/_(__っ┓
/_(υ__)┓__/◎┻υ◎
◎゙υ┻-◎゙_/__/__/__/__/__/キコキコ…… __/__/__/__/__/__/__/__/__/__/__/__/__/__/__/__/__/__/__/__/__/__/__/__/「困ったら微分」「困難は分割せよ」前>>215高校時代に先生が言った言葉だよ。「信じる者は救われる」
0219132人目の素数さん2020/04/16(木) 23:38:43.92ID:10nqZrEx
>>202
アルコールと水を混ぜると体積は足し算での計算より減るんじゃなかったかな?
0221132人目の素数さん2020/04/16(木) 23:58:13.21ID:GxmMJodg
>>219
水+アルコール混合による体積減少を考慮するとなると、混ぜる前の精密な計算はかなり厄介ですね。
いちおう日本のアルコール度数表示は体積パーセントとのこと...
0222132人目の素数さん2020/04/17(金) 00:20:04.71ID:jlmVdX2C
>>210
ありがとう。
まったく分からん。

>>211
ごもっともです。
だからこそ、重要なんです。

Yが奇数のときに、X、Yの解が存在しないことを
証明するための手続きなんです。

(X+1) (X-1) が両方、整数の立方であることを説明したら、
以下のように続けてフィニッシュなんです。

最小の立方体でも体積は 1 と 8 で
7 離れている。
よって距離が2しか離れていないような
自然数で構成される立方体2つの組が存在しないのは明らか。
Yが奇数のとき、これを満たす 解 (X,Y) は存在しない。
おしまい
0223132人目の素数さん2020/04/17(金) 00:24:17.73ID:jlmVdX2C
>>210
ちなみに、
「Y ≠ 2 で Yが偶数の場合、
解 X,' は存在しない」

という証明も…何かの長い論文で証明されてます。

そちらは、素人が戯れに解くような問題じゃないみたいでした。
0224132人目の素数さん2020/04/17(金) 01:43:26.46ID:dHpPBaE7
>>223
無限降下法でしょ。
数学的帰納法を理解できるなら、理解できるレベルのお話だと思うけど。
0225132人目の素数さん2020/04/17(金) 01:58:10.21ID:WTqtCTaR
正多角形のうち、以下の条件を満たすもの全体からなる集合をSとする。
(条件)『直交する2本の対角線が、少なくとも1組存在する』
例えば正方形ABCDを考えると、対角線ACとBDが直交するため、Sの要素である。

では正2n+1角形(n=2,3,...)全体からなる集合をTとするとき、S∩Tは空集合か。
結論と理由を述べよ。
02262102020/04/17(金) 02:18:01.49ID:9hIlQifL
がんばれ。そのうち分かる。

>>211 は誤り。
0227132人目の素数さん2020/04/17(金) 03:22:11.77ID:9hIlQifL
>>215 で正解だが、芸風を伝承して・・・
AP^2 = (x-1)^2 + y^2
 = (x-1)^2 + (1 - xx/4)
 = (3/4)(x - 4/3)^2 + 2/3
 ≧ 2/3,
∴ AP ≧ √(2/3),  (x, y) = (4/3, ±(√5)/3)
また最大値は
9 - AP^2 = 9 - (x-1)^2 - y^2
 = 2(2+x) + (4-xx) - (1-xx/4)
 = 2(2+x) + 3(1-xx/4)
 ≧ 0  (-2≦x≦2 より)
∴ AP ≦ 3,   (x, y) = (-2, 0)
0228132人目の素数さん2020/04/17(金) 04:18:00.65ID:9hIlQifL
>>225
 正奇数角形の場合は、どの対角線にも、それに平行な辺がある。
(略証)
 対角線の端点以外に奇数個の頂点がある。
 一方の側に偶数個、他方の側に奇数個の頂点がある。
 偶数個の側に頂点を辿ってゆけば、対角線に平行な辺に至る。(終)

したがって、Sの条件は
 『直交する2辺(の延長線)が、少なくとも1組存在する』
と同値である。
しかし、直交する2辺(の延長線)が存在する正多角形は、偶数角形に限る。
∴ S∩T は空集合。

なお、Sは正2n角形全体からなる集合に一致する。
  対角線 0 - n と 対角線 (n-k) - (n+k) が直交する。
0229132人目の素数さん2020/04/17(金) 07:37:44.52ID:FGyxpq6I
>>221
>水+アルコール混合による体積減少
の関係式ってネット検索したけど、みつけられなかった。
0230132人目の素数さん2020/04/17(金) 08:30:01.56ID:9hIlQifL
・教えて!goo
http://oshiete.goo.ne.jp/qa/1678133.html
「水+エタノール=?」

・研究紀要/東京学芸大学附属竹早中学校
http://core.ac.uk/download/pdf/33468167.pdf
の8ページの図 (W.Cordes, 2008)

・アルコール表
http://unit.aist.go.jp/nmij/
 計量標準総合センター → 計量標準・JCSS → 一番下の「アルコール表」の下の「参考」
「エタノール水溶液の濃度と密度との対照表(旧表)」
 (15℃におけるアルコール表)  alchol120530.pdf
0231イナ ◆/7jUdUKiSM 2020/04/17(金) 10:48:47.10ID:UUkt12DA
>>219竜二の「水飲ましときゃいいから、来週サツが引っ張るらしいから絶対買うなよ」のところが浮かんだ。前>>218
‖∩∩‖ □ ‖;;;;;\
((~o~)   ‖;;;;;;‖
(っγυ  。‖╂─╂‖
■`(_)_)ц~ ‖╂─╂‖
\■υυ■_∩∩、\\‖
\\\\⊂(_ _ )`⌒つ)
\\\\\\\`υ、\/|
\\\\\`.,、、、\`/ |
__\\\\彡`-`ミっ/ L
 ̄|\_\\_U,~⌒ヾ /
]| ‖ ̄ ̄ ̄ ̄U~~U / /
__| ‖ □ □ ‖ |/ /
___`‖________‖/_/
 ̄ ̄ ̄ ̄ ̄ ̄ ̄ ̄ ̄‖ /
__________________‖/
0233132人目の素数さん2020/04/17(金) 20:05:57.33ID:WTqtCTaR
aを実数とし、
S(a) = Σ[n=1,2,...] {1/(n-a) - 1/(n+a)}/2a
とおく。

(1)lim[a→+0] S(a)を求めよ。答えのみでよい。

(2)(1)の結果を用いて、オイラーの定数γと円周率πとの間にある関係を考察し、それを等式の形で表せ。
0235132人目の素数さん2020/04/18(土) 01:45:13.33ID:lQgtbWdn
任意の自然数の列a1,a2,…anがある。
この両側から適当にそれぞれ何個か取り除き、(取り除かない場合も含める)少なくとも1個残すようにすることによって、残した数の和がnの倍数であるように出来る事を証明せよ。
0236132人目の素数さん2020/04/18(土) 02:03:46.84ID:cXu1ZGgp
>>233
(1)
S(a) = Σ[n=1,∞] {1/(n-a) - 1/(n+a)}/(2a)
 = 1/(2aa) - {1/a + Σ[n=1,∞](1/(a-n) + 1/(a+n))}/(2a)
 = 1/(2aa) - {π/tan(πa)}/(2a)
 = {1 - (πa)/tan(πa)}/(2aa)
 → ππ/6  (a→0)
 = 1.644934
  
*) マクローリンで
 x/tan(x) = x・cos(x)/sin(x)
 = (1-xx/2+・・・)/(1-xx/6+・・・)
 ≒ 1 - xx/3 + ・・・・
(2)
ちょっと手が出せませんね。
0237132人目の素数さん2020/04/18(土) 02:22:08.53ID:BHjUPemc
>>235
Si=a1+‥+ai とおく。
n|Si (1≦i≦n) なるiがあればa1〜aiを残す。
そうでなければDirichlet原理からSi≡Sj(mod n) (1≦i<j≦n)なるi,jがとれる。
この時a(i+1)〜ajを残せば良い。
0238132人目の素数さん2020/04/18(土) 04:24:20.22ID:cXu1ZGgp
>>221
アルコール度(Vol%)
混合液1L中のアルコールの量を、
15℃における純アルコール (密度 0.79351 g/cc) の体積として表示したもの。

===================
アルコール度  密度
 (Vol%)   (g/cc)
-------------------
  0.0   0.99910
 60.0   0.91296
 77.7   0.86970
 78.5   0.86753
 96.0   0.81171
  100   0.79351
===================
 (OIML R 22 -1975 国際アルコール表)  >>230
0239132人目の素数さん2020/04/18(土) 05:25:16.95ID:cXu1ZGgp
>>202

その飲み物 90ml + 水 20ml = アルコール 86.4ml + 水 24.4987 ml
アルコール 78.5 Vol% = アルコール 78.5ml + 水 24.4845 ml

その飲み物 170ml + 水 40ml = アルコール 163.2ml + 水 48.49755 ml
アルコール 77.7 Vol% = アルコール 77.7ml + 水 25.3371 ml

密度(g/cc) ≒ 0.9923 - 0.0607 V - 0.0936 V^2 - 0.0407 V^3,

ここに Vはアルコール度(Vol) 0.1 < V ≦ 0.96
0240132人目の素数さん2020/04/18(土) 06:06:16.15ID:HblCQrT8
P(x),Q(x)は実数係数の多項式で、cは実数の定数とする。いま、任意の実数xに対して
x^4+P(x)≧Q(x)^2+c, x^4+Q(x)≧P(x)^2+cがともに成り立っているという。

⑴cとしてありうる最大の値を求めよ
⑵ ⑴のcに対し、ありうるP(x),Q(x)の組をすべて決定せよ
0241132人目の素数さん2020/04/18(土) 10:13:26.07ID:ycvrciG7
2以上の整数a,b,cで

a!=b^c+c^b

を満たすものは存在するか.
0247132人目の素数さん2020/04/18(土) 16:31:54.23ID:D/oIL22U
>>210
3行でさらっと解説されてるけど
じっくりとここだけを考えたら解けたわ。

(2n+1) (2n+3) = Y^3

(2n+1) と (2n+3) が立方数 (整数の3乗 x1)であることを
証明せよ
って感じで。
0248132人目の素数さん2020/04/18(土) 17:07:45.71ID:D/oIL22U
中共のオトモダチ、東ドイツのメルケルちゃん。
そして、実質、 ドイツ帝国になり下がった EUちゃん。

@ EU + 中国
= ドイツ帝国 + 中国共産党

これ、期末試験に出るから覚えとけ〜 ( '〜')
0250132人目の素数さん2020/04/18(土) 18:54:57.80ID:Ou2q39Fu
すいません、質問です
物体を動かすアニメーションを作ってて、どうも動きがロボットみたいに味気ないので
慣性の法則っていうんですか、加速、減速、加速したのち減速
の三つの関数を入れようと思ってます

で、以下でできたのですが、欲を覚えて、加減速具合をなめらかに(移動と時間の関係を比例に近づける)したり、
逆に急激にしたり出来るような変数を付け加えた計算式に改造しようと適当に数時間いじってたんですが、
どうしても出来なかったのでどなたかアドバイス頂けないでしょうか

どれくらい難易度高いのかもよくわからないのにすみません

イージングについての参考はこちら
http://nakamura001.hatenablog.com/entry/20111117/1321539246
※カーブの具合についての計算方法は上記の通り色々あるのですが、かゆいところに手が届かないので変数で変更したいです


t : 時間(進行度)
b : 開始のX座標
c : 開始と終了の値の差分(移動距離)
d : 移動にかかる合計時間

■easeInQuad(加速)
c * (t /= d) * t + b;

■easeOutQuad(減速)
-c * (t /= d) * (t - 2) + b;

■easeInOutQuad(加速したのち減速)
if ((t /= d / 2) < 1) c / 2 * t * t + b; ※時間(進行度)が半分未満ならこちらの行だけ実行、そうでなければ下の行
-c / 2 * ((--t) * (t - 2) - 1) + b;



参考:
ちなみに単なる直線の比例グラフだと以下のようになります
c * (t /= d) + b;
0251132人目の素数さん2020/04/18(土) 21:44:04.23ID:lnOWoFYO
>>250
例えばこんな感じとか
(言語: javascript)
function easeInOut(t,b,c,d, p,q,r){
let x=t/d
let y=c / (r/p + (1-r)/q)
if (x<r) {
y *= x**p /(p*r**(p-1))
} else {
y *= (r/p + (1-r)/q) -(1-x)**q /(q*(1-r)**(q-1))
}
return y + b
}

始まりと終わりの緩急を p, q で その切り替え時間(の比率) を r で指定する。
if 分岐の2関数が滑らかに接続するようにしただけ。 難易度は低い。
グラフ↓は (p,q, r) = (2, 3, 0.8) と (0.5, 1, 0.4) の場合
0253132人目の素数さん2020/04/18(土) 22:07:34.66ID:Ou2q39Fu
>>251
なるほどです!
ちょうどjavascriptで書いていたので助かりました
ありがとうございます
0254132人目の素数さん2020/04/18(土) 23:43:39.25ID:D/oIL22U
>>209
ようやく >>210さんの説明に追いついたわ。
210さんが1時間でレスした内容を理解するのにワイは2日かかった。

みんな頭いいな、旧帝大の工学部とか?

前半
yが奇数の時、
------> y^3 = (X+1)(X-1) = 1x立方 x 1x立方

自然数の立方で差が2であるようなものは存在しないので、
これを満たす自然数 x,y は存在しない。

後半 (時間かかった)
yが偶数の時、
-------> y^3 = (X+1)(X-1) = 4x(自然数の立方) x 2x(奇数の立方)
0255132人目の素数さん2020/04/18(土) 23:48:12.05ID:D/oIL22U
問い1.3

X,Y は自然数とする。

「Yが4以上の偶数である時」、

............. Y^3 = (X+1)(X-1)

を満たす自然数 X, Y は存在しないことを証明せよ。

ヒント: Y^3 = (X+1)(X-1) = 4x(自然数の立方) x 2x(奇数の立方)

↑ めっちゃムズいし、Yが奇数の時のように、簡単には解けない。
ガチで論文何ページか必要なレベル…だと思う。
0256132人目の素数さん2020/04/19(日) 05:47:40.90ID:vjR/Of8/
a,bを整数とし、f(x)=x^2+ax+bを考える。
いま正の実数xに対し、単位円周C上に点P(cos2πf(x), sin2πf(x))をとる。
またCの弧で(1/√2)≤x≤(√3/2)かつy>0を満たす部分をKとする。
xが変化するとPもC上を動くが、PがKに含まれるようなxの範囲(閉区間)は無数に存在する。それらをx座標が小さい順にI_1,I_2,...とする。

(1)区間I_nの長さL[n]をa,bで表せ。

(2)極限 lim[n→∞] Σ[m=1,2,...,n] L[m]を求めよ。
0258132人目の素数さん2020/04/19(日) 09:26:57.69ID:hLIYoBnx
>>194-196
それたぶん拾った雑音を言語野が無意識に言語化しちゃってるのでは?
いわゆる“空耳”、「幻聴」なのでは?
0259132人目の素数さん2020/04/19(日) 09:46:12.67ID:fBJM1RRB
>>258
私には何故か嫌がらせを行う人間達がまとわりついている
「しはくはむり」と二回誰だか分からない女の声が聞こえてきたが
学部卒が博士を上回る仕事をするとそうなるらしい
0263132人目の素数さん2020/04/19(日) 11:04:35.73ID:UVABXCJs
解答ありがとうございます。

もしかしたら...
「猫からの人獣共通感染症で
脳内で炎症反応が起きミクログリアに脳神経細胞が食べられ過疎てしまった後のマクログリアによる修復が追いついていない状態」
にあるのかも知れませんね。
言語野聴覚野関連領域での神経過疎化が改善しきってないのかも?ですね..

素地にASDなどがあって神経に伝わる雑音のカット機能に障害があると陥り易い症状なのかも知れませんね..

辛いですよね。
周囲からも理解されずに孤独を深めてしまいますよね…

良い治療薬が作られるかもですから、それまで少しでもお気持ちが楽に安心して過ごせる事を願ってます
0264132人目の素数さん2020/04/19(日) 11:42:51.92ID:fBJM1RRB
>>263
いいえこれは幻聴ではありません、5chで目立つ行動をすると、それを意味不明に叩く人間達が
多く発生するのです

先輩を馬鹿にしやがって、だとか社会を馬鹿にしやがってと叫び、私の部屋での
独り言を盗聴して、因縁を付けてくる人間が確かにいるのです

特に、私が行った研究が他の学者を馬鹿にしているというふうに考える奴らがいる
から、嫌がらせを言われたり、部屋の中に勝手に入られて、物を壊されるということ
が起きています
0265132人目の素数さん2020/04/19(日) 16:26:38.09ID:TSqZi+JJ
a!+abc=c!
を満たす互いに素な自然数の組(a,b,c)を全て求めよ。
0268132人目の素数さん2020/04/20(月) 10:30:07.46ID:rA0/Poiv
>>44

>>53

>>55

はじめに正△の各辺をn等分するなら、正△の総数は M(n) = (n+3)C4.
いまの場合は n=6

数学セミナー、2006年2月号のエレ解・出題1
(出題 2月号、解説 5月号、中本先生)
0269132人目の素数さん2020/04/20(月) 12:20:21.27ID:yDfwyDii
>>264
>部屋の中に勝手に入られる

自宅なら刑法第130条の規定する刑法犯に問えますよね?

>物を壊される

「器物損壊罪」刑法261条に明確に抵触しますよね

1に証拠2に証拠、3・4も証拠で5に証拠
── 論より証拠 ──

刑事告訴(起訴・公判維持)可能な強力かつ充分な証拠収集にお励み下さい。

グッドラックでございます…
0270132人目の素数さん2020/04/20(月) 12:25:44.37ID:yDfwyDii
※ご存知かとは思いますが、
刑法は「疑わしきは罰せず」などと抜かすザル法で名高い
「犯罪者に有利に〜
〜被害者に不利に」
出来ているトンデモな悪法です。
証拠・証言力でしか、対処しようが無いと思います。

それが出来なければ「幻聴・被害妄想」、統合失調症の典型的な症状との判別がつきません。
0271132人目の素数さん2020/04/20(月) 12:54:47.92ID:EPNKRbuu
この問題の解き方教えて下さい
高校数学です
sinx-sinxcosx+cosx-1/2の最大値最小値を求めよ
0272132人目の素数さん2020/04/20(月) 13:25:36.34ID:+uECHfrL
√(n+1)/{n+√(n+1)+√(n+2)} = {n+√(n+1)-√(n+2)}/(n+1)
を満たす自然数nを全て求めよ。
0273132人目の素数さん2020/04/20(月) 14:56:34.81ID:GZ1PpgU1
>>271
(与式)= sinx +cosx - (1/2)(sinx+cosx)^2
= -(1/2)q^2 + q  { q := sinx+cosx = √2.sin(x+π/4) }
= -(1/2)(q-1)^2 + 1/2  { =: f(q) }
-√2 ≦ q ≦ +√2 より
f(+1) = +1/2 (最大値)
f(-√2) = -(1/2)(3+2√2) +1/2 = -1-√2 (最小値)
0275132人目の素数さん2020/04/20(月) 15:49:46.14ID:SvjK0ZNw
数学の公式って
著作権とかロイヤリティって
発生しないよね。

論文を読むためには、
会員になったり、有料のサブスクをしないといけないのに
なんか変な感じ。
0276132人目の素数さん2020/04/20(月) 15:53:56.14ID:SvjK0ZNw
・ダウンロードコンテンツ 幾何学 (ピタゴラスの定理セット)

・ダウンロードコンテンツ 代数学 (線形代数セット)
・ダウンロードコンテンツ 微積分 (ライプニッツ、ニュートンセット)

↑ こういう風に特許をかけそうなのに
数学者って寛容だな
0277132人目の素数さん2020/04/20(月) 16:32:34.69ID:YSMPdYrN
そもそもアルゴリズムも特許なしの筈だったんだがな
数学も何時まで持つやら
0278132人目の素数さん2020/04/20(月) 16:58:17.13ID:SvjK0ZNw
>>277
総当りN^2 探索 … 100円

二分木探索 … 2000円

クイック・ソート … 5000円
0279132人目の素数さん2020/04/20(月) 20:10:26.09ID:Db3kUO+J
>>273
グラフ化と数値解を出してみた

https://i.imgur.com/1SmTY91.png

> f<-function(x) sin(x)-sin(x)*cos(x)+cos(x)-1/2
> curve(f(x),-pi,pi,bty='l',lwd=2)
> optimize(f,c(-pi,pi))
$minimum
[1] -2.356202

$objective
[1] -2.414214

> optimize(f,c(-pi,pi),maximum = TRUE)
$maximum
[1] 1.570803

$objective
[1] 0.5
0280132人目の素数さん2020/04/20(月) 20:52:41.64ID:rA0/Poiv
例) カーマーカー特許(AT&T)

USP 4,744,026 「最適資源割当のための方法および装置」(1988/May)
 射影変換とアフィン変換を組合せて線形計画問題を解く方法。
 産業連関分析なんかに使うのかな?

特許 第2033073号「最適資源割当方法」
特願昭61-501865 特公昭62-502580 特公平5-61672

研究の出口は用途の発見。
用途があれば産業にとって useful.
new & useful inventions are patentable.
0281132人目の素数さん2020/04/21(火) 01:06:14.58ID:6SMLYdGW
>>271
 (与式)= 1/2 -{1-sin(x)}{1-cos(x)}≦ 1/2(最大値)

 (与式) = (√2 +q)(2+√2 -q)/2 -√2 -1 ≧ -√2 -1(最小値)
0282イナ ◆/7jUdUKiSM 2020/04/21(火) 09:19:38.52ID:0ilKIHza
>>231式変形を思いつかずに解かなきゃみんなが解けることにはならない。微分だ。微分しないといけない。
>>271
f(x)=sinx-sinxcosx+cosx-1/2とおき、
f'(x)=cosx-cos^2x+sin^2x-sinx
=cosx+1-2cos^2x-sinx=0とすると、sinx=1+cosx-2cos^2x
cos^2x+sin^2x=1に代入し、
cos^2x+(1+cosx-2cos^2x)^2=1
cos^2x+1+cos^2x+4cos^4x+2cosx-4cos^2x-4cos^3x=1
4cos^4x-4cos^3x-2cos^2x=0
cosx=0,2cos^2x-2cosx-1=0
cosx=0,(1-√3)/2
(0≦x≦2π)
x=π/2,α
f(π/2)=sin(π/2)-sin(π/2)cos(π/2)+cos(π/2)-1/2=1-1・0+0-1/2=1/2
f(α)=sinα-sinαcosα+cosα-1/2
=(cos^2α-2cosα)(1-cosα)+cosα-1/2
=[{(1-√3)/2}^2-(1-√3)]+(1-√3)/2-1/2
={(4-2√3)/4-1+√3}-√3/2
=√3/2-√3/2
=0
最大値 1/2
最小値 0
違うか。計算間違いか?
0283イナ ◆/7jUdUKiSM 2020/04/21(火) 10:02:08.29ID:0ilKIHza
>>282訂正。最小値が負になる可能性がある。
>>271
sin(4π/3)-sin(4π/3)cos(4π/3)+cos(4π/3)-1/2
=sin(7π/6)-sin(7π/6)cos(7π/6)+cos(7π/6)-1/2
=-2.2990381……
4π/3と7π/6のあいだのxを調べると、
f(5π/4)=sin(5π/4)-sin(5π/4)cos(5π/4)+cos(5π/4)-1/2
=-2.41421356……
<f(6π/5)=-2.3……
∴x=π/2のとき最大値1/2
x=5π/4のとき最小値-1-√2
0284イナ ◆/7jUdUKiSM 2020/04/21(火) 10:30:24.08ID:0ilKIHza
>>283確認。
f'(5π/4)=cos(5π/4)+1-2{cos^2(5π/4)}-sin(5π/4)
=0
f'(π/2)=cos(π/2)+1-2{cos^2(π/2)}-sin(π/2)
=0+1-2・0-1
=0
0287132人目の素数さん2020/04/21(火) 14:58:20.40ID:7U4Xz7Q/
>>272
√(n+1)/{n+√(n+1)+√(n+2)} = {n+√(n+1)-√(n+2)}/(n+1)

(n+1)√(n+1) = {n+√(n+1) + √(n+2)}{n+√(n+1) - √(n+2)}

(n+1)√(n+1) = {n+√(n+1)}^2 - (n+2)

(n+1)√(n+1) = (n-1)(n+1) + 2n√(n+1)

(n+1) = (n-1)√(n+1) + 2n

0 = (n-1){√(n+1) + 1 }
∴ n=1
0288132人目の素数さん2020/04/21(火) 14:59:48.59ID:ksbq9frG
電卓でモードをDegにしてsin(30)とすると0.5になりました
モードをRadにしてsin(30)にすると-0.9880316241になりました

ラジアンを度に変換する式は deg=rad*(180/π) らしいので
この式を使えば-0.9880316241が0.5になるのかなっと思ったのですが結果は-56.61004209でした。

sin(30)=0.5と、変換式で出てきた-56.61004209は別物ですか?
0289イナ ◆/7jUdUKiSM 2020/04/21(火) 15:28:27.72ID:0ilKIHza
>>284文字化けが激しいですが、
>>272
√(n+1)/{n+√(n+1)+√(n+2)}={n+√(n+1)-√(n+2)}/(n+1)の分母を払って、 (n+1)√(n+1)==n+√(n+1)-√(n+2)}{n+√(n+1)+√(n+2)}
=n+1)√(n+1)=n^2+2n√(n+1)+n+11 +)
n^2+2(-1)√(n+1) 1=0
n=1,n+1+√(n+1)=0 n≠1のとき
n+1+√(n+1)≠0

ゆ∴n =1



n=1
02912852020/04/21(火) 20:28:18.28ID:7U4Xz7Q/
>>285 の件
式の一部でググったら arcsin(x) の展開式が出てきました.

{d/dx}arcsin(x) = 1/√(1-xx) = (1-(xx))^{-1/2}
= 1 + (1/2)(xx) + ... + (1/k!)(1/2)(1+1/2)...(k-1+1/2)(xx)^{k} + ...
= 1+ Σ[k=1,∞](2k-1)!!/(2^k*k!) * x^{2k}
= Σ[k=0,∞](2k)!/(2^{2k} (k!)^2) * x^{2k}
∴ arcsin(x) = Σ[k=0,∞] (2k)!/(2^{2k} (k!)^2)*1/(2k+1) * x^{2k+1}
(与式) = arcsin(1) = π/2  {OK!}
0292132人目の素数さん2020/04/21(火) 21:53:11.04ID:e4vrYWUo
学者が解決できない問題を解決した人間に、「学者気取り」と侮辱するな!
学者と言えということではないが
0293132人目の素数さん2020/04/21(火) 22:03:56.66ID:isZ/BuV5
a,b,tを正の実定数とし、平面上の2点A,BをA(a,0)、B(b,t)と定める。
y軸上を点P(0,x)が動くとき、|AP-BP|が最大および最小となるxを求めよ。
なおそのようなxが±∞となる場合は「最大値なし」のように答えよ。
0294132人目の素数さん2020/04/21(火) 22:08:39.55ID:zIkl+tBW
フェルマーの最終定理って
あくまで予想だよな。

証明したのはワイルズさんだ、
よって、ワイルズの定理って呼ぶべきだと思うんだが…

なんで未だにフェルマーの最終定理って呼ばれるての?
ワイルズさん、かわいそす… (´・ω・`)
0296132人目の素数さん2020/04/21(火) 23:02:40.98ID:7U4Xz7Q/
>>293
Pのy軸拘束条件を外すと
|AP - BP| が一定となる点の軌跡は 双曲線, 直線, 半直線2つ のどれかになる.
一目瞭然なので計算は省略.
a=b (ABがy軸に平行)の場合だけ「最大値なし」が生じる.
0298132人目の素数さん2020/04/22(水) 00:47:53.24ID:Qsry3gwp
すみません
留数の範囲でわからない問題があるので質問させてください...

複素数zの関数 (e^z-1)/(sinz)^2
の原点における留数を求めよという問題です

留数自体あまり理解が深くないので詳しいところまで書いていただけるとありがたいです...
0299132人目の素数さん2020/04/22(水) 00:51:58.59ID:Qsry3gwp
>>298
0301132人目の素数さん2020/04/22(水) 01:04:24.54ID:puzWK/mv
>>292
このレスを書いたらすぐにrejectメールがきた
どういう情報通信がなされているのでしょうか
0302132人目の素数さん2020/04/22(水) 01:33:39.44ID:zkDD5tDY
>>296
便宜上 点P(x,y) とすると

|AP-BP|が最大となるのは
 |AP-BP|= AB (三角不等式)
 y =(t/(b-a))(x-a)  (a≠b)
 のうち(x-a)(x-b) ≧0 の部分。
 x = a,       (a=b)
 のうち y(y-t) ≧ 0 の部分。

|AP-BP|が最小となるのは
 |AP-BP|= 0,
 y = {(bb-aa+tt) + 2(a-b)x}/(2t)  (t>0)
0303132人目の素数さん2020/04/22(水) 04:25:59.72ID:TsaejLc5
テレビゲーム??
>コンウェイ氏はイギリス・リバプール出身の数学者で、1970年に発表した、
>生命の誕生や死をコンピューター画面でシミュレーションするテレビゲーム「ライフゲーム」が世界的な人気を集めました。
0305132人目の素数さん2020/04/22(水) 08:33:43.17ID:qc/CyeDg
dy/dx=x+y
をx+yをuとして解き、yの一般解を求めよ
0306132人目の素数さん2020/04/22(水) 09:18:59.97ID:x8EUKVZb
>>305
u=x+y と置く.
du/dx = 1 + dy/dx = 1 + u
du/(1+u) = dx
d{log(1+u)} = d{x}
u = C*e^x - 1 {C: 積分定数}
∴ y = u - x = C*e^x - x - 1
0308132人目の素数さん2020/04/22(水) 11:03:11.46ID:b9suJfpl
>>304
懐かしい呼び方
マイコンとかもな
0309イナ ◆/7jUdUKiSM 2020/04/22(水) 11:42:30.37ID:iq1GZOqA
>>289
>>307
f(x)=x^3/3+ax^2+bxとおくと、
f(x+1)-f(x)=(x+1)^3/3+a(x+1)^2+b(x+1)-x^3/3-ax^2-bx
=x^2+x+1/3+2ax+a+b
=x(x+1)
1/3+2ax+a+b=0
a=0,b=-1/3
∴f(x)=x^3/3-x/3
確認する。
f(x+1)-f(x)=(x+1)^3/3-(x+1)/3-(x^3/3-x/3)
=x^2+x+1/3-x/3-1/3+x/3
=x^2+x
=x(x+1)
0311132人目の素数さん2020/04/22(水) 12:33:31.15ID:x8EUKVZb
一般解は
 f(x) = x^3/3-x/3 + g(x)
g(x)は g(0)=0, g(x+1)-g(x)=0 [周期 1] を満たす任意関数

微分可能条件を外せば x∈(0,1) に対して f(x)の値を任意に定めて 他のxに対しては帰納的に定義できる
{ x∈(0,1) での x^3/3-x/3 を相殺して任意値に置き換える g(x) を持って来ればよい }
0313132人目の素数さん2020/04/22(水) 16:29:52.58ID:9mqmJ5br
球の表面積について考えよう。
(以下の物語はフィクションです)

****「単純な立体」****

遠い昔、ある惑星のある国にて

国王 「三次元の立体で、最も単純な物は何か?」

Aは答えた。
「それは三角ピラミッド(三角錐) です。
4つの頂点と4つの面を持ち、この立体は、その頂点の数が最小だからです」

Aのライバルであった B は答えた。
「それは球体です。
球はそれを構成する1つ1つのポリゴン(側面)の面積が最小だからです」

Aは反論した
「頂点と側面の数が無限個あるのに、どこが単純なのだ?」
Bは答えた
「それは本質ではありません。
紙粘土があれば素手で…ほら、ご覧ください。
このように美しい物が一瞬で誰にでも作れます」

国王 「なるほど、よく分からん」

結論が出せず、国王の仲裁のもと、
両者はお互いを認めて、その2つともを
「(3次元空間においての) ある種の単位元」
として認めることに同意した。
0314132人目の素数さん2020/04/22(水) 16:35:34.51ID:9mqmJ5br
ある時、B は球の表面積が分からずに困っていた。

Aは、
「三角錐と球はどちらも単位元であり、
双子の兄弟のようなものだ。
球を観念上の三角錐と見做してはどうだろうか?」
と提案した。

B はなるほど…と思い、これを受け入れた。
球が観念上の三角錐であるならば、
それは側面を4つ持つはずで、その4つの合計が表面積だ。

しかし、前にB自身が述べたように
「球は側面が無限個であり、それを構成する1つ1つのポリゴンの面積がゼロ」 である。
B は困った。
Aは紙粘土の球を 真っ二つに割った。
そして、 「この断面を、観念上の側面の面積 としてはどうか?」と提案した。
Bは、またもや、なるほど…と思い、これを受け入れた。

球の表面積 = (観念上の)側面の面積 x 4 = pi x r^2 (x 4)

Bはこの理論をまとめて王立学会へ提出した。

学会「結論はもっともらしい、しかし、各所の論理に不備がある」
としてBの論文と自宅は燃やされた。

全てはAの罠だったのだ。
この A という男が、後のピタゴラスである。
0315132人目の素数さん2020/04/22(水) 16:41:52.77ID:UxYqi2wk
>>307
二項係数の (n)C(r) = (n-1)C(r) + (n-1)C(r-1) から (n)C(r) = Σ_{k= r~n} (k-1)C(r-1)
x(x+1) = 2 (x+1)C(2) を右辺に入れると 2(n)C(3) = Σ_{x=1~n-2} x(x+1)
これによる一般化もある
0316132人目の素数さん2020/04/22(水) 16:53:14.65ID:JdiYsh/T
無限級数
a[k] = Σ[n=0,1,...] (n^k)/n!
について、以下の問いに答えよ。
ただし0!=1である。

(1)任意の自然数kに対してある自然数b[k]が存在し、a[k]はa[k]=b[k]eの形で表されることを示せ。

(2)a[k+1]をa[k],a[k-1],...のうち必要なもので表せ。
0317132人目の素数さん2020/04/22(水) 20:01:04.21ID:rXiT9q74
U = {1, 2, …, n}
S_1, …, S_m を互いに異なる空でない U の部分集合とする。
#(S_i ∩ S_j) = const. for all i, j ∈ {1, 2, …, m} such that i ≠ j が成り立っているとする。

このとき、 n ≧ m であることを示せ。
0318132人目の素数さん2020/04/22(水) 20:13:01.37ID:dwnNPUdQ
すみません
極限を求める問題で

lim[x→∞] 2n+1/n

lim[x→∞] (2n^2-n^3)

lim[x→∞] 2n^2-5n+3/3n^2-1

をお願いしまーす
0319132人目の素数さん2020/04/22(水) 20:13:01.37ID:dwnNPUdQ
すみません
極限を求める問題で

lim[x→∞] 2n+1/n

lim[x→∞] (2n^2-n^3)

lim[x→∞] 2n^2-5n+3/3n^2-1

をお願いしまーす
0320132人目の素数さん2020/04/22(水) 20:13:01.40ID:dwnNPUdQ
すみません
極限を求める問題で

lim[x→∞] 2n+1/n

lim[x→∞] (2n^2-n^3)

lim[x→∞] 2n^2-5n+3/3n^2-1

をお願いしまーす
0321132人目の素数さん2020/04/22(水) 21:03:27.68ID:kRr7pLPO
lim[x→∞] 2n+1/n = 2n+1/n
lim[x→∞] (2n^2-n^3) = 2n^2-n^3
lim[x→∞] 2n^2-5n+3/3n^2-1 = 2n^2-5n+3/3n^2-1
0322132人目の素数さん2020/04/22(水) 21:11:15.18ID:vQYkkh7E
正解ですね
0323132人目の素数さん2020/04/22(水) 21:45:01.87ID:U2P0DW+D
板から鎖を削り出すことが可能か不可能かというのは、数学的に証明できますか?
0324イナ ◆/7jUdUKiSM 2020/04/22(水) 23:23:40.15ID:iq1GZOqA
_____∩ っ゙___前>>309
\ (-_-))  /|、\\\
\\υ⌒υ、 /|\\\\
 ̄ ̄ ̄ ̄|υ/|、\\\\
________「 ̄|\\\\\
\\\\\\\\\\\
\\\`∩∩、\\\\\
\\⊂(_ _ )`⌒つ、\\
\\\\\`υ、\\\\
\\\\\\\\\>>323板から鎖を削りだすことはできる。証明することもできる。が、忙しいんでそれはだれかに任せたい。
0326132人目の素数さん2020/04/23(木) 00:18:11.83ID:mwnzVt9h
>>316
(1)
a[k] = Σ[n=0,∞] n^k/n!
e^t = Σ[n=0,∞] t^n/n!
{t.d/dt} e^t = t.e^t =: g(1,t) e^t
{t.d/dt}^k e^t = Σ[n=0,∞] n^k.t^n/n! =: g(k,t) e^t
{t.d/dt}^{k+1} e^t = (t.{d/dt}g(k,t) + t.g(k,t)) e^t
∴ g(1,t)=t, g(k+1,t)= t.{d/dt}g(k,t) + t.g(k,t)
b[k] := g(k,1) ∈ N { k次多項式 g(k,t) ∈ R[t] の係数は全て非負整数 (∵ 帰納法) }
a[k] = g(k,1) e^1 = b[k] e
0327132人目の素数さん2020/04/23(木) 00:21:21.70ID:mwnzVt9h
>>316
(2)
Σ[j=1,k] (-1)^j.a[j] = Σ[n=1,∞] Σ[j=1,k] (-n)^j/n!
=Σ[n=1,∞]( (-n)-(-n)^{k+1} )/(n+1)!
=Σ[n=1,∞]( (1-n)-(1-n)^{k+1} )/n!
= k.a[1] + (-1)^k.a[k+1] - Σ[j=2,k]C{k+1,j} (-1)^j a[j]

a[k+1] = (-1)^{k+1} (1+k) a[1] + Σ[j=2,k] (C{k+1,j} + 1) (-1)^{k+j} a[j]
(1)の結果は使わなかった.
0328132人目の素数さん2020/04/23(木) 02:10:58.79ID:9l3JVJvY
>>313-314
勉強できない子には
こうやって覚えさせるといいよ。

または、
球体に光をあびせて、投射された影4つ分。
0329132人目の素数さん2020/04/23(木) 02:46:42.39ID:eSmNgQRh
板から板の厚さよりも細くない鎖を削り出すことが可能か不可能かというのは、数学的に証明できますか?
0330132人目の素数さん2020/04/23(木) 03:15:25.06ID:RvkAG6tR
n次関数のn個の解を実数解はx軸上に複素数解はxy平面を複素平面だと思ってプロットすると
もとのn次関数とn個の点の位置関係は何かきれいな幾何学的関係になってるのでしょうか?
03313272020/04/23(木) 09:50:12.66ID:mwnzVt9h
>>327 (2) 別解
a[k] = Σ[n=1,∞] n^k/n!
= Σ[n=1,∞] ( n^k + n^{k+1} )/(n+1)!
= Σ[n=1,∞] ( (n-1)^k + (n-1)^{k+1} )/n!
= a[k+1] + Σ[j=1, k] (C{k,j} (-1)^{k-j} + C{k+1,j} (-1)^{k+1-j}) Σ[n=1,∞]n^j/n!
= a[k+1] + Σ[j=1, k] ( C{k,j} - C{k+1,j} ) (-1)^{k-j} a[j]
= a[k+1] - Σ[j=1, k] C{k,j-1} (-1)^{k+j} a[j]

a[k+1] = (k+1) a[k] + Σ[j=1, k-1] C{k, j-1} (-1)^{k+j} a[j]
0332132人目の素数さん2020/04/23(木) 09:55:49.30ID:jD7BDxDo
平面上に、どの2点間の距離も奇数であるような4点は存在しないことを証明せよ。
0333132人目の素数さん2020/04/23(木) 09:55:59.59ID:HZRVAVG+
>>330
まず、複素数上の任意のn点について、これを解とするn次関数が存在する

上記のn次関数の各項の係数は一般に複素数である

もし、各項の係数を実数に制限するのであれば、以下の性質を満たす

・nが奇数の場合、少なくとも1点は必ず実軸上にある
・実数でない解の場合、互いに共役である2点が解となる
0334132人目の素数さん2020/04/23(木) 10:11:52.67ID:M2d54xbk
z^k の係数を a(k) とする。
1.
〔掛谷の条件〕
a(0) ~ a(n) はすべて非負とする。
0 < a(n) < a(n-1) < ・・・・・ < a(1) < a(0)
⇒ 根は単位円の外部

0 ≦ a(0) < a(1) < ・・・・・ < a(n-1) < a(n)
⇒ 根は単位円の内部

0 = a(1) = a(2) = ・・・・・ = a(n-1), |a(0)|=|a(n)|≠0
⇒ 根は単位円上

2.
f '(z) = 0 の根は f(z) = 0 のすべての根の凸包に含まれる。
0335132人目の素数さん2020/04/23(木) 11:11:34.10ID:M2d54xbk
>>316
(1)
 x^k = Σ[j=0, k] s(k, j) x(x-1)・・・・(x-j+1)
 s(k, j) は第二種スターリング数。
を使えば
 n^k = Σ[j=1, min{k,n}] s(k, j) n(n-1)・・・・(n-j+1)  (n≧1)

 (n^k)/n! = Σ[j=1, min{k,n}] s(k, j) /(n-j)!

 (与式) = Σ[j=1, k] s(k, j) Σ[n=j, ∞] 1/(n-j)!
  = Σ[j=1, k] s(k, j) e
  = b(k) e,

* s(k, j) は k個の異なる物をj組に分ける方法の数。
 s(k, 1) = 1  (k≧1)
 s(k, k) = 1,
 s(k, 0) = δ(k, 0), s(k, j) = 0 (k<j)
 s(k, j) = s(k-1, j-1) + j・s(k-1, j)
また
 b(k) = Σ[j=0, k] s(k, j) はベル数。
0336132人目の素数さん2020/04/23(木) 11:16:48.40ID:jD7BDxDo
>>317

ヒントを出しておきます: 線形代数の知識を使う。
0337132人目の素数さん2020/04/23(木) 11:35:24.08ID:M2d54xbk
生成関数
 Σ[k=j,∞] s(k,j)/k! x^k = (1/j!)(e^x - 1)^j,
 Σ[k=0,∞] b(k)/k! x^k = e^(e^x - 1),

 b(0)=b(1)=1, b(2)=2, b(3)=5, b(4)=15, b(5)=52, b(6)=203,
0338132人目の素数さん2020/04/23(木) 11:53:23.54ID:M2d54xbk
例)
5つの異なる物を何組かに分ける方法の数は b(5) = 52 種ある。
「源氏香」
0339132人目の素数さん2020/04/23(木) 12:19:31.08ID:BDCd4koF
[質問の前提]
円周率の近似値を得るために、
単位円に内接する正多角形の1辺の長さを求める。

図と漸化式は省略

正6角形:1
正12角形:√(2-√3) = (1/2)(√6-√2) -> 0.5176381
正24角形:√(2-√(2+√3)) -> 0.2610524
正48角形:√(2-√(2+√(2+√3))) -> 0.1308063
正96角形:√(2-√(2+√(2+√(2+√3)))) -> 0.0654382

ここまでが書籍の参考に書いてあったことの抜粋です。

[質問]
正12角形は途中式(1/2)(√6-√2)が書いてあり納得できましたが、
24,48,96角形のケースは書いてありませんでした。
それぞれの途中式を教えてほしいです。
0342132人目の素数さん2020/04/23(木) 12:59:46.58ID:9XIQ+7o4
>>338
同じモノだったら?
03443392020/04/23(木) 13:03:44.21ID:BDCd4koF
正24角形
√(2-√(2+√3))=√(2-(1/2)(√6+√2))

ここで詰まるんだけど
そもそも多重根号はずせないケースなの?
ならばどうやって0.2610524を導くの?
0345132人目の素数さん2020/04/23(木) 13:07:47.42ID:M2d54xbk
>>339
中心角2θに対する辺の長さ L = 2sinθ,
から
中心角θに対する辺の長さ L ' = 2sin(θ/2),
を求める。
cos の半角公式
 cos(θ/2)= √{(1+cosθ)/2},
を使う。
 L ' = √{2 - √(4 - L^2)},

これくらい自分で計算できなきゃ・・・後ry)
0346132人目の素数さん2020/04/23(木) 13:20:49.13ID:M2d54xbk
>>342
同じモノ5つなら
 (5)
 (4,1)
 (3,2)
 (3,1,1)
 (2,2,1)
 (2,1,1,1)
 (1,1,1,1,1)
 p(5) = 7 種
0347132人目の素数さん2020/04/23(木) 13:25:35.35ID:9XIQ+7o4
>>346
ありがとうです
b(n),p(n)の直接式か漸化式かあれば
教えてください
0348132人目の素数さん2020/04/23(木) 14:07:49.04ID:WhP/1iIy
>>344
多重根号が外れるケースのほうが多いとでも思ってるのかw
めでたいなー
0349132人目の素数さん2020/04/23(木) 14:15:57.50ID:M2d54xbk
p(k) の生成関数
 Σ[k=0,∞] p(k) x^k = 1/{(1-x)(1-xx)・・・・(1-x^n)・・・・}
 p(k)の級数展開が見つかった。(Rademacher, 1937)

同じk個をj組に分ける方法を q(k,j) とすれば
 p(k) = Σ[j=1,k] q(k,j)
 q(k,j) = q(k-1,j-1) + q(k-j,j)
 q(k,1) = q(k,k-1) = q(k,k) = 1,

qの生成関数
 Σ[k=j,∞] q(k,i) x^k =(x^j)/{(1-x)(1-xx)・・・・(1-x^j)}

http://oeis.org/A000110 b(k)
http://oeis.org/A000041 p(k)

数セミ増刊「数学100の問題」日本評論社 (1984) p.58
03513392020/04/23(木) 14:27:11.61ID:BDCd4koF
>>341
ありがとう。
多重根号はずせないとのことで
小数の平方根についてググルことで納得しました。

正24角形
√(2-√(2+√3))
=√(2-(1/2)(√6+√2))
=√(2-(1/2)(2.449+1.414))
=√(0.0685)
=√((6.85)(1/100))

√6<√(6.85)<√7
2.44<√(6.85)<2.64

√(6.85)≒2.62

√(0.0685)
≒0.262
0352132人目の素数さん2020/04/23(木) 14:31:43.59ID:pKj9Ka/j
ランダムな正定値行列の生成方法ってありますか?
半正定値行列であればランダムなnxn行列Aに対して(A^t)Aを計算すればいいのはわかるのですが正定値行列については調べても正しい方法がわかりませんでした
0353132人目の素数さん2020/04/23(木) 14:45:47.70ID:pKj9Ka/j
すみません解決しました
0354132人目の素数さん2020/04/23(木) 15:50:05.20ID:9l3JVJvY
f(x) = a x^n + b x^(n-1 ) + ... + z = 0

n が奇数である時、f(x) = 0 を満たすxが
1つ以上存在することを証明せよ



↑ これって図で f(x) = x^3+1 を描けば
f(x) = 0 、すなわち、x軸を横切って交点を持つから
明らかなんだろうけどさ。
証明ってどうやるの?

x軸のどこかに隙間があったり、
関数の曲線が途中でジャンプしていたり
抜け道があるかもしれないじゃん。

例えば、
f(x)= 1/x は途中でジャンプしてるし、 x軸ともy軸とも交点がないよね。
0355132人目の素数さん2020/04/23(木) 16:00:41.17ID:RvkAG6tR
>>334
> f '(z) = 0 の根は f(z) = 0 のすべての根の凸包に含まれる。
Gauss?Lucas theorem
https://en.wikipedia.org/wiki/Gauss%E2%80%93Lucas_theorem

ガウスがこの定理を発見してたのか。実係数のn次関数のグラフの極値になってる
x座標の最大(最小)の外側に(複素数)解の(実部)が必ず存在するってことはすぐに言えるのか
0357132人目の素数さん2020/04/23(木) 17:19:17.17ID:9l3JVJvY
>>356
有理数は連続してねぇじゃん?
無理数も連続してねぇじゃん?

有理数と無理数をつなげ合わせただけの物が
なぜ連続だと言えるんだ?

なぜ実数が連続していると言える?
数学者が信じているだけで連続していないかもしれないじゃん。
0358132人目の素数さん2020/04/23(木) 17:30:28.23ID:9l3JVJvY
f(x) = a x^n + b x^(n-1 ) + ... + z = 0

n が奇数である時、f(x) = 0 を満たすxが
(すなわち、x軸との交点) が1つ以上存在することを証明せよ。


回答

f(x) = x^3 + 1 の場合を考える。

r を実数とする。
y = f(x) = 0 を満たすx軸上の点 (r, 0) が
1つも存在しないと仮定する。

x軸 を 3つに区分けする。

[r より小さい有理数を並べたもの] < r < [r より大きい有理数を並べたもの]

次に r + δ を考える。 この時…

ああああああああああああああああ 訳わかんねぇ
0359132人目の素数さん2020/04/23(木) 17:34:54.64ID:9l3JVJvY
考えろ、考えろ、考えるんだ、おれ、…。
確実に答えに…本質に…近づいている…

>>358は良い線いってる
このスレのザコ助どもに答えられないなら、
おれが自分で解くしかない。

「尋ねるよりも己で調べるのが良い、
調べるよりも己で考えるのが良い、
考えるよりも己で確認するのが良い」
0360イナ ◆/7jUdUKiSM 2020/04/23(木) 18:42:16.68ID:YxsPXNvw
>>358
>>324
アルファベットの並びは変わらないからn=25
25は奇数だから題意を満たす。
f(x)=ax^25+bx^24+cx^23+dx^22+ex^21+fx^20+gx^19+hx^18+ix^17+jx^16+kx^15+lx^14+mx^13+25x^12+ox^11+px^10+qx^9+rx^8+sx^7+tx^6+ux^5+vx^4+(w+1)x^3+yx+z
はx→±∞のときf(x)→+∞だから全体として下に凸。
f'(x)=25ax^24+24bx^23+23cx^22+22dx^21+21ex^20+20fx^19+19gx^18+18hx^17+17ix^16+16jx^15+15kx^14+14lx^13+13mx^12+300x^11+11ox^10+10px^9+9qx^8+8rx^7+7sx^6+6tx^5+5ux^4+4vx^3+3(w+1)x^2+y=0
を満たすxをx=αとおくと、
f(α)<0ならf(x)=0を満たすαは2つ以上の偶数個存在する。
f(α)=0ならf(x)=0を満たすαは1つ以上の奇数個存在する。
f(α)>0ならf(x)=0を満たすαは存在しない可能性はあるが、存在する可能性もある。
0361イナ ◆/7jUdUKiSM 2020/04/23(木) 19:58:19.97ID:YxsPXNvw
>>360訂正。前々>>324
>>358
f(x)=ax^25+bx^24+cx^23+dx^22+ex^21+fx^20+gx^19+hx^18+ix^17+jx^16+kx^15+lx^14+mx^13+25x^12+ox^11+px^10+qx^9+rx^8+sx^7+tx^6+ux^5+vx^4+(w+1)x^3+yx+zと書くまでもなかった。
題意よりnは奇数で、
x→+∞のときf(x)→+∞
x→-∞のときf(x)→-∞
よってy=f(x)のグラフは少なくとも一回x軸をまたぐ。
∴f(x)=0を満たすxが少なくとも1つ以上存在する。
いったんわかると、当たり前だろうが、と思う。これだから証明は。ただの当たり前。意味がない。
0362132人目の素数さん2020/04/23(木) 23:37:31.03ID:+/ZQ6zMU
a,bは複素数の定数とする。方程式
z^4+az^3+bz^2+az+1=0
の4つの複素数解をα、β、γ、δとする。
複素数平面上の4点P(α),Q(β),R(γ),S(δ)を周および内部に含む円で半径最小のものを求めよ。
0363132人目の素数さん2020/04/24(金) 03:53:09.87ID:FdH14EWV
>>354
a≠0, b, c, ・・・・は実数とする。
p = 1 + (|b|+|c|+・・・・)/|a| とすると
 f(p)f(-p) < 0,
区間[-p,p] を繰り返し2等分する。
任意の ε>0 について
 f(x1) f(x2) < 0   |x1-x2| < ε
を満たす [x1,x2] がある。


>>355
最も外側の極値では|f(x)|が極小だから、複素数解が存在しそう・・・・
0364132人目の素数さん2020/04/24(金) 06:07:57.08ID:5n8nJPBe
三角形ABCとその外接円の外部(周を含まない)に点Dがある。
ABCDを内部(周を含む)に含む半径最小の円を作図する方法は?
0365132人目の素数さん2020/04/24(金) 07:24:59.66ID:TvMPAnae
pを実数の定数とする。
実数a,b,cについての以下の連立不等式を解け。
b≦a+pb+(p^2)c≦c
c≦(p^2)a+b+pc≦a
a≦pa+(p^2)b+c≦b
0366132人目の素数さん2020/04/24(金) 08:50:34.39ID:FdH14EWV
両端だけ書けば
b≦c,
c≦a,
a≦b,
∴ a=b=c,

中辺から
・1+p+pp = 1 つまり(p=-1 または p=0)のとき
 a=b=c
・p(p+1)≠0 のとき
 a=b=c=0.
0367哀れな素人2020/04/24(金) 09:18:43.69ID:sGb2PyV3
>>364
Dと外接円の中心を結ぶ線を引き、
それが外接円と交わる点のうち、Dから遠い方をEとする。
DEの中点を中心として直径DEの円を描けば、
何はともあれ四点A、B、C、Dはその円内に含まれる。

次にDを中心として、
A、B、Cのうち最もDから遠い点を半径とする円を描き、
それがDEと交わる点をFとする。

DFの中点を中心として直径DFの円を描けば、それが最小の円である。(?)
0368132人目の素数さん2020/04/24(金) 09:29:26.19ID:FdH14EWV
>>364
・ABCDの凸包が3角形のとき
 3辺の長さを p>q>r とする。
 pp ≧ qq+rr のとき 直径pの円
 pp ≦ qq+rr のとき 外接円

・ABCDの凸包が4角形のとき
 対角の和を A+C、B+D とすると合計360°
 一方は 180°以上。
 180°以上の側の大きい方の頂点を除いて三角形で考える。
0369哀れな素人2020/04/24(金) 10:56:03.50ID:sGb2PyV3
>>367を訂正

「次にDを中心として」以下は間違いだから省略。

最小の円はDEを直径とする円か?
0372132人目の素数さん2020/04/24(金) 13:29:14.04ID:72kz69Ny
>>364
簡単すぎてわろ主www
A,B,C のうち、
D と最も離れた点 との線分を直径とする円で確定じゃん。
0374132人目の素数さん2020/04/24(金) 13:37:59.97ID:raCghl3F
>>371
正五角形の図より 1 : 2x = 2x : 1-2x
∴ 4x^2 + 2x - 1 = 0
cos(2π/5) = x = (-1 + √5)/4
sin(π/5) = √{1-cos(2π/5) } / √2 = √{10 -2√5 } / 4
0376イナ ◆/7jUdUKiSM 2020/04/24(金) 14:27:34.01ID:A23RaIEQ
>>361
>>371
黄金比は(1+√5)/2
cos(π/5)=(1+√5)/4
sin(π/5)=√{(1-cos^2(π/5)}
=√{1-(6+2√5)/16}
=√(10-2√5)/4
=0.587785252……
0377132人目の素数さん2020/04/24(金) 16:42:35.35ID:5n8nJPBe
>>372
DとBがもっとも離れた点とする。中心DでBを通る円をE、DBを直径の両端とする円をFとする。
Eの内部かつFの外部の領域にABのどちらか一点があるときは間違いだとすぐわかる。
その場合は外部にある点とBCの外接円が半径最小
>>368
0378イナ ◆/7jUdUKiSM 2020/04/24(金) 17:24:43.13ID:A23RaIEQ
>>376
>>364
△ABCの外接円の中心Oを通る半直線DOと外接円の交点をEとし、DEの中点にコンパスの針を刺し、半径DE/2の円を描く。
0379132人目の素数さん2020/04/24(金) 17:45:25.19ID:72kz69Ny
>>377
え、余裕ぶったけど >>372 は間違ってるんか!?
ABCは同一の外周円、 円1 の上にあって、
点Dはその外側にあるっていう前提だろ?

じゃあ、Dから最も遠い点が…この場合、Bとしたら、
線分DB を直径にして円を描いたら、A,B,C すべて中に含むし、
これが答えだろ。
だって、これより小さい円 = 直径が小さい円 なんて描けないだろ。
0380132人目の素数さん2020/04/24(金) 17:48:00.01ID:72kz69Ny
ああああ、
手元にコンパスがねぇえええ!!
作図したい時に限ってコンパスが見当たらない
0381132人目の素数さん2020/04/24(金) 17:50:25.45ID:72kz69Ny
おれは手を動かした方が
考えが捗るんだ。

散歩したり、トイレやベッドで
パズルを考える事もあるけどさ、
やはり数学は手を動かすのが一番やりやすい。

… MS-Paint でやってみるか
0383132人目の素数さん2020/04/24(金) 18:13:49.05ID:7AvnGIne
A(a,b,c,d)
Aを変化させた場合の値は分るとする
Aはロスを表すもので0〜1と考えられる(場合により1を超えるだろうがabcdの値に制限があるとする)
aのみを変化させることは可能 → いくつかプロットすると曲線となった
bを変化させるときaも変化してしまう
cを変化させるときaも変化してしまう
dを変化させるときaも変化してしまう
Aの計算式の算出方法について方針など教えてください
0384132人目の素数さん2020/04/24(金) 21:37:33.84ID:5n8nJPBe
>>364 一般化した話のスライドがあった。一般化しても数学的に簡単な話かと思っていたらそうでもないみたいだ。

 最小包含円 点集合を包含する半径最小の円
https://www.jaist.ac.jp/~uehara/course/2014/i481f/pdf/ppt-7.pdf
0385132人目の素数さん2020/04/24(金) 22:50:44.80ID:gAM6gLQO
全然数学的じゃないけど円の形を保ったまま伸び縮みする輪ゴムをかぶせれば一発だな
0386132人目の素数さん2020/04/24(金) 23:00:42.00ID:ZBDsOWg7
>>379
>線分DB を直径にして円を描いたら、A,B,C すべて中に含むし、
含むと限らない
DBを半径として円を描いたらABCすべて含むけどな
0387132人目の素数さん2020/04/24(金) 23:20:10.62ID:5Kcdt8vP
f(a) = ∫[0, ∞] e^(-x)cos(ax)dx = 1/(1+a^2)
とおく。

(1)a=1/bなるbを用いて、h(1/b)を求めよ。
g(a) = ∫[0,a] f(a) da
h(a) = ∫[0,a] g(a) da

(2)∫[0, ∞] sin(x)/x dx = π/2 を示せ。
0388132人目の素数さん2020/04/25(土) 02:01:35.57ID:Hi7yRVBg
三本の太さは同じだが長さと重さの違う棒、
その1(長さa, 重さA)、
その2(長さb, 重さB)、
その3(長さd, 重さD)がありまして、
その棒をこの順番で溶接でくっつけたときに、重心がどこにあるかを調べたいのです。
棒は均一に同じ密度だとします。

どのようにしたらよいのでしょうか?積分を使うのでしょうか?
0389132人目の素数さん2020/04/25(土) 02:08:58.58ID:gWVamGeU
>>387
f(a) = ∫[0, ∞]dx e^(-x)cos(ax)
= {1/(1+ia) + 1/(1-ia)}/2
= 1/(1+a^2)

g(a) = ∫[0,a]da 1/(1+a^2) = arctan(a)
h(a) = ∫[0,a]da arctan(a)
= a.arctan(a) - ∫[0,a]da a/(1+a^2)
= a.arctan(a) - (1/2)∫[0,a^2]ds 1/(1+s)
= a.arctan(a) - (1/2) log(1+a^2)

g(a) = ∫[0,a]da f(a)
= ∫[0, ∞]dx e^(-x) ∫[0,a]da cos(ax)
= ∫[0, ∞]dx e^(-x) sin(ax)/x
= ∫[0, ∞]dx e^(-x/a) sin(x)/x
∴ ∫[0, ∞] e^(-x/a) sin(x)/x = arctan(a)
a → ∞ により ∫[0, ∞] sin(x)/x = π/2

h(1/b) 云々は意図がよくわからない
0390132人目の素数さん2020/04/25(土) 10:29:47.45ID:gWVamGeU
h(a) = ∫[0,a]da g(a)
=∫[0, ∞]dx e^(-x) ∫[0,a]da sin(ax)/x
=∫[0, ∞]dx e^(-x) {1- cos(ax)}/x^2
= a.∫[0, ∞]dx e^(-x/a) {1- cos(x)}/x^2
= 2a.∫[0, ∞]dx e^(-x/a) {sin(x/2)/x}^2
= a.∫[0, ∞]dx e^(-2x/a) {sin(x)/x}^2

∴ ∫[0, ∞]dx e^(-2x/a) {sin(x)/x}^2
 = arctan(a) - (1/2a) log(1+a^2)
(a → ∞) ∫[0, ∞]dx {sin(x)/x}^2 = π/2

問題の意図とは違うかも。
0391132人目の素数さん2020/04/25(土) 11:09:11.55ID:Cwq71twU
辺の長さ1対2対√3の三角形に外接する三角形ABCの面積Sの最大値最小値教えて下さい
0392132人目の素数さん2020/04/25(土) 11:17:12.47ID:rBuhJQzD
最大値最小値?
それ、直角三角形だから外接円は直径2だろ?
0393132人目の素数さん2020/04/25(土) 11:35:27.54ID:BqK44kXn
>>391
最大値はない。いくらでも大きくできる
最小値は元の三角形と同じとき。(√3)/2
0395132人目の素数さん2020/04/25(土) 14:37:41.80ID:NU1ShEA6
3辺の長さがBC=a,CA=b,AB=cで、最長辺がABであるような△ABCを考える。
ここで以下の(操作)を定義する。

(操作)「3辺の長さがx,y,z(0≦x≦y≦z)の三角形に対して、3辺の長さがx,y,z/2の新しい三角形を作る。」

△ABCに(操作)を施し、その新しくできた三角形に(操作)を施し、さらに…と無限に(操作)を施すことができるとき、a,b,cが満たす関係式を求めよ。

ただし3辺の長さがx,y,zの三角形に(操作)を施すことができないとは、3辺の長さがx,y,z/2の三角形が存在しないことを指す。
0396132人目の素数さん2020/04/25(土) 16:57:01.84ID:TejRT81v
三角不等式
 y-x < z < x+y,
 y-x < z/2 < x+y,
が両立することから
 2(y-x) < z < y+x,
∴ y < 3x,
さて、どうするか・・・・
0397132人目の素数さん2020/04/25(土) 18:22:48.22ID:OHswhcEN
複素数ってネーミングが酷いけどさ、
あれって英語のImaginary Number
英語圏、数学動画のコメント欄でも評判が悪いのな

以下のように改名すべきって主張されてて納得したわ。

・Real Number → Direct Number
   直行数

・Imaginary Number →Lateral Number
   側方数
0398132人目の素数さん2020/04/25(土) 20:45:19.80ID:nWc5MJiO
xの値の範囲が1≦x≦のときに、aを定数とする2次関数y=−x^2+2axの最小値は−4である。このときのaの値を求めよ。ただし、aの値の範囲は1<a<2。
0399132人目の素数さん2020/04/25(土) 21:02:00.48ID:ReTz6AXc
>>397
複素数はComplex Numberですが

知らんのか?
0400132人目の素数さん2020/04/25(土) 23:04:30.99ID:vTtrr7Ii
2x2正方行列AについてA^3=Oとなる条件を求めよ

と言うのがわかりません

A=(ab)
(cd)

として
A^2=(a+d)A-(ad-bc)E
A^3=(a+d)A^2-(ad-bc)A=Oなら

[(a+d)^2-(ad-bc)]A=(a+d)(ad-bc)E


ここからがわかりません

とりあえずAはEの実数倍であることは必要であってますか?
0403132人目の素数さん2020/04/26(日) 00:13:38.96ID:Ktrnazai
>>401
なるほディウス!

つまり
a+d=0またはad-bc=0、かつ、
(a+d)^2=ad-bc、

またはA=EorO

ってことですか
0404132人目の素数さん2020/04/26(日) 00:17:33.55ID:/lkqvJ2S
>>400
ケイリー・ハミルトンの定理とか懐かしい
どこまでの条件が欲しいのかわからないけど、もし成分レベルの条件が欲しいなら、
仮にAに逆行列が存在したらどうなるか考えてみればわかると思う
0406132人目の素数さん2020/04/26(日) 00:37:00.99ID:MdR4o85a
>>403
[(a+d)^2-(ad-bc)]A=(a+d)(ad-bc)E でAがEの定数倍なら
A=kE と置けば k={(a+d)(ad-bc)}/{(a+d)^2-(ad-bc) ってことだぞ}
0407132人目の素数さん2020/04/26(日) 01:51:38.93ID:XJpZpSSS
A^3=O
⇔a+d=0 かつ ad-bc=0

←はハミルトンケーリーより自明。
→A^3=Oよりad-bc=0は明らか。
∴ A^2-(a+d)A=O。
両辺にAをかけて(a+d)A^2=O。
∴ (a+d)^2A=O。(∵ A^2=(a+d)A)
∴ (a+d)^2a=0, (a+d)^2d=0。
∴ (a+d)^3=0。
0408132人目の素数さん2020/04/26(日) 10:26:55.46ID:JKzB//mA
>>395
どなたかこれをお願いします
かなり昔の早稲田大学の理系入試に類題があったと思いますが見つけられず、教えを請いに参りました。
0409132人目の素数さん2020/04/26(日) 10:37:01.73ID:/lkqvJ2S
>>407
ネタばらしされちゃったから補足しておくと、成分を使わずに書けば
tr(A) = det(A) = 0
が A^3 = O であるためのの必要十分条件ってこと
当然 A の逆行列は存在しない
あと、後半の証明は、もし a + d ≠ 0 なら A = O になって矛盾するとしたほうがスマートかな
0410132人目の素数さん2020/04/26(日) 10:49:03.13ID:/lkqvJ2S
>>408
その(操作)っていつでも可能じゃないの?
問題の条件が足りない気がする
0411132人目の素数さん2020/04/26(日) 11:01:52.03ID:lNbbygqz
>>403

 (a+d)(ad-bc) = 0,
すなわち
 a+d=0 または ad-bc=0,
これと
 [(a+d)^2 - (ad-bc)] = 0
から
 a+d = ad-bc = 0   ・・・ (*)
∴ A^2 = O も成り立ちますね。

なお、A=kE の場合も
 A^3 = (k^3)E,
 k = 0,
 A = O,
となり、(*) に含まれます。

ところで・・・・
A = k [1, x]  = k [1, 1] [x, -1]
  [x, xx]   [x, x] [x, -1]
のようになっており、左右入れ替えて掛ければ
 [x, -1] [1, 1] = O
 [x, -1] [x, x]
つまりAの正体は零因子なので
 A^2 = O
となるのです。
0412132人目の素数さん2020/04/26(日) 11:11:34.70ID:lNbbygqz
↑ の訂正
A = 2k [x, -1]  = k [1, 1] [x, -1]
   [xx, -x]   [x, x] [x, -1]
でした。スマソ
0413132人目の素数さん2020/04/26(日) 11:36:23.93ID:lNbbygqz
もちろん、
 A^3 = O  ⇔  A^2 = O
が成り立つのは2次行列に限った場合です。

反例:シフト行列
  [ 0, 1, 0 ]
R = [ 0, 0, 1 ]
  [ 0, 0, 0 ]

   [ 0, 0, 1 ]
R^2 = [ 0, 0, 0 ]
   [ 0, 0, 0 ]

R^3 = O
0414132人目の素数さん2020/04/26(日) 13:34:18.84ID:lNbbygqz
>>398
 yが最小となるのは、xの範囲のどちらかの端だが・・・・
 x=1のときは y=-1-2a, これともう一方の値のうちの小さい方が4
0415132人目の素数さん2020/04/26(日) 13:47:24.72ID:lNbbygqz
>>410
y > 3x のときは
 y - x > (x+y)/2 > z/2,
となるから{x, y, z/2}は不可能らしいよ。

あと 2(y-x) < z も要るかな。
0416132人目の素数さん2020/04/26(日) 13:59:26.48ID:/lkqvJ2S
>>415
ん?よくわからない
z / 2 と x, y について何か追加の条件があるの?
単に一番長い辺を1つ選んで、その辺の中点と頂点を結んで新しい三角形を作るのを繰り返すだけだと思ってた
0417132人目の素数さん2020/04/26(日) 14:44:45.19ID:/lkqvJ2S
任意の三角形で可能なら
>三角形の成立条件(存在条件):三辺の長さが a,b,c である三角形が存在する必要十分条件は,
>a+b>c かつ b+c>a かつ c+a>b
https://mathtrain.jp/seiritu

あと c が一番大きい前提の条件 a ≦ c かつ b ≦ c でも書いておけばいいのかな
よくわからないけど
0420132人目の素数さん2020/04/26(日) 14:59:14.35ID:/lkqvJ2S
成立する具体例ってある?実は存在しないってオチ?
それとも1回できれば無限にできるとか?
0421132人目の素数さん2020/04/26(日) 15:03:01.40ID:JKzB//mA
>>416
問題文読み間違えてる
例えば3辺の長さが(1,100,100.4)の三角形に対してこの操作を行うと、(1,50,50.2)の数の組ができるけど、これは三角形の3辺にならない
0422132人目の素数さん2020/04/26(日) 15:06:55.88ID:JKzB//mA
>>420
早稲田の問題は一回できれば無限回できるというオチだったと記憶していますが、この問題よりも条件に制限があったと思います
成立する例は正三角形とかですね
(2,2,2)→(2,2,1)、これは三角形の成立条件を満たす
0424132人目の素数さん2020/04/26(日) 15:34:31.11ID:JKzB//mA
>>423
一回は(操作)可能ですよ
無限回可能かというと、そうではないですね
0426132人目の素数さん2020/04/26(日) 15:42:21.62ID:JKzB//mA
(5,5,6)→(5,5,3)→(5/2,5,3)→(5/2,5/2,3)
これで1ループなので無限回操作可能ですね
0428132人目の素数さん2020/04/26(日) 15:50:25.65ID:lNbbygqz
z<2x なら、うまくローテーションしそう・・・ (十分条件)
0431132人目の素数さん2020/04/26(日) 16:55:28.04ID:/lkqvJ2S
>>428
0 < x ≦ y ≦ z のとき、
z ≦ 2x かつ y < x + z/2 かつ 2x < y + z
が十分条件の1つになることがわかった
このとき、
(x,y,z)→(x,y,z/2)→(x,y/2,z/2)→(x/2,y/2,z/2)
でループになる

(証明)
(1): (x,y,z)→(x,y,z/2)
仮定より、
x < y + z/2
y < x + z/2
z/2 < x + y
となるので(操作)を施すことができる
このとき、
z/2 ≦ x ≦ y

(2): (x,y,z/2)→(x,y/2,z/2)
仮定より、
x < y/2 + z/2
y/2 < x + z/2
z/2 < x + y/2
となるので(操作)を施すことができる
このとき、
y/2 ≦ z/2 ≦ x

(3): (x,y/2,z/2)→(x/2,y/2,z/2)
仮定より、
x/2 < y/2 + z/2
y/2 < x/2 + z/2
z/2 < x/2 + y/2
となるので(操作)を施すことができる
このとき、
x/2 ≦ y/2 ≦ z/2

ここで、(3)で得られた(x/2,y/2,z/2)は(x,y,z)と相似であるので、
以上(1), (2), (3)により、(x,y,z)がなす三角形は、無限に(操作)を施すことができる
(証明終了)

これ以外に無ければいいんだけど…
0432132人目の素数さん2020/04/26(日) 17:17:52.15ID:/lkqvJ2S
>>431
この条件を使ったら、二等辺三角形を含まないループ
(8,10,12)→(8,10,6)→(8,5,6)→(4,5,6)
が作れた
0433132人目の素数さん2020/04/26(日) 17:26:08.71ID:/lkqvJ2S
>>431
この3条件のうち、
y < x + z/2
は必要条件だから、
x ≦ z/2 ≦ y

x ≦ y ≦ z/2
の場合で分けて考えれば、残りも証明できるかも
0435132人目の素数さん2020/04/26(日) 18:45:33.87ID:/lkqvJ2S
>>434
>x ≦ y ≦ z/2 はありえない

それは証明できる?

>言いたいのは
>z/2 ≦ x ≦ y
>ではないの?

>>431の証明でわかるのは、
z ≦ 2x かつ y < x + z/2 かつ 2x < y + z
が十分条件の1つになることと、
z/2 ≦ x ≦ y のときは「y < x + z/2 かつ 2x < y + z」が必要になるってことだけ
残りのパターンも考えないと必要十分にならない
0436132人目の素数さん2020/04/26(日) 19:41:39.99ID:krNAjsP6
a+b+c≦20を満たす、自然数(a,b,c)の組のうち、aが奇数のものは半数以上あることを示せ
0437132人目の素数さん2020/04/26(日) 20:17:45.16ID:PUf+kZPC
>>435
>>x ≦ y ≦ z/2 はありえない
>それは証明できる?

三角不等式と与条件より
z/2 < (x+y)/2 ≦ (y+y)/2 = y
0438132人目の素数さん2020/04/26(日) 21:02:17.81ID:aW2mSzpS
(19-1,2)+(19-3,2)+...+(19-17,2)
=18*17/2+16*15/2+14*13/2+12*11/2+10*9/2+8*7/2+6*5/2+4*3/2+2*1/2
=N(aが奇数)

(19-2,2)+(19-4,2)+...+(19-16,2)
=17*16/2+15*14/2+13*12/2+11*10/2+9*8/2+7*6/2+5*4/2+3*2/2
=N(aが偶数)

N(aが奇数)とN(aが偶数)の各項を大きい方から見比べて、N(aが奇数)>N(aが偶数)
0439132人目の素数さん2020/04/26(日) 21:08:49.38ID:bPfdGFxG
大学でε-δ論法をやってるんですが

x→0でのsinx/x、(1+x)^(1/x)

これらの極限はεδでどうやって示せるんですか?
0440132人目の素数さん2020/04/26(日) 21:24:37.54ID:/lkqvJ2S
>>437
ありがとう
それで z/2 < y がわかるから、あとは
x < z/2 < y
のケースを考えればいいね
0441132人目の素数さん2020/04/26(日) 22:41:19.02ID:lNbbygqz
>>439

[例1]  lim[x→+0](1+x)^(1/x)= e.
e は nが自然数なるとき lim[n→∞](1+1/n)^n として定義されたのであったが、
連続的変数xに関しても標記の等式が成り立つのである。
それを証明するために n ≦ 1/x < n+1 (nは自然数) とすれば
 {1 + 1/(n+1)}^n <(1+x)^(1/x)<(1 + 1/n)^(n+1),

任意のε>0 に対して一つの正数Nを取って、n≧N なるとき
 e - ε <{1 + 1/(n+1)}^n,(1 + 1/n)^(n+1)< e + ε
ならしめ得る。 然らば 0 < x ≦ 1/N なるとき
 e - ε <(1+x)^(1/x) < e + ε,
すなわち |(1+x)^(1/x)- e| < ε.
∴ lim[x→+0](1+x)^(1/x)= e.

[例2]  lim[x→0]sin(x)/x = 1.
(前略)
 1 > sin(x)/x > cos(x)  (1)
さて 0 < sin(x)< x から lim[x→0]sin(x)= 0.
∴ 0 ≦ 1 - cos(x)^2 = sin(x)^2 < x^2 を用いて lim[x→0]cos(x)= 1.
∴ (1)から標記の関係を得る。      (証終)

高木:「解析概論」改訂第三版、岩波書店(1961)
  p.20-22 第1章 §9. 連続的変数に関する極限
0443132人目の素数さん2020/04/26(日) 22:48:18.29ID:Dq8acRgI
↑ おまえには無い

↓ おまえも無い
0444132人目の素数さん2020/04/26(日) 22:49:09.81ID:Dq8acRgI
うわ、矢印間違えた…
0446132人目の素数さん2020/04/26(日) 23:40:42.99ID:lNbbygqz
>>436
a+b+c≦n を満たす自然数(a,b,c)の組
 aを決めると b+c = n-a から
 (b,c) の組合わせは n-a-1 とおり。

・aが奇数(1,3,・・・・)のとき
 n-a-1 = n-2, n-4, ・・・・ (≧1)
 足し合わせると[ ((n-1)/2)^2 ]とおり。

・aが偶数(2,4,・・・・)のとき
 n-a-1 = n-3, n-5, ・・・・ (≧1)
 足し合わせると[ ((n-2)/2)^2 ]とおり。

∴ 奇数の方が偶数より n/2 ほど多い。
0449132人目の素数さん2020/04/27(月) 00:06:57.40ID:R+jFPCfx
>>445
己の生に意味がないことに気づいた。

だから、己の生に意味を与えるために
仕事や結婚をするのでしょう。
0450132人目の素数さん2020/04/27(月) 01:45:37.73ID:NmPFwCj5
>>448
k=3 から k=n まで足し合わせれば

a:奇数
 Σ[k=3,n] [ ((k-1)/2)^2 ]=[ (n+1)(n-1)(2n-3)/24 ]

a:偶数
 Σ[k=3,n] [ ((k-2)/2)^2 ]=[ n(n-2)(2n-5)/24 ]

全 体
 Σ[k=3,n] (k-1)(k-2)/2 = Σ[k=3,n] {k(k-1)(k-2) - (k-1)(k-2)(k-3)}/6
  = n(n-1)(n-2)/6,

n=20 の場合
 a:奇数 615,  a:偶数 525,  合計 1140
0451132人目の素数さん2020/04/27(月) 08:24:46.10ID:om55qmtg
↓ってどういうこと?
比例関係の場合xとyの比が同じだから一方がa倍になったらもう一方もa倍になるんじゃないの?


693 ツール・ド・名無しさん sage 2020/04/26(日) 19:26:02.18 ID:kTEMn+SY
比例ってy=kxだからxがa倍になったらyもa倍になるのでは?

720 ツール・ド・名無しさん sage 2020/04/26(日) 23:18:32.51 ID:8KMvg+fa
>>693
yがa倍の時xは(a+k)倍やろ
こんだけユニークIDで義務教育からやり直さないといけないやつがいるって日本の未来は暗いな
0452132人目の素数さん2020/04/27(月) 08:58:54.44ID:NmPFwCj5
>>450
 全部の組合せ

a+b+c = n のとき
 (a-1)+(b-1)+(c-1)= n-3,
n-3 を3つの非負整数に分ける。
n-1 個から仕切りを2つ選ぶ。
  C(n-1,2) = (n-1)(n-2)/2,

a+b+c ≦ n のとき
 a+b+c+d = n+1
 (a-1)+(b-1)+(c-1)+(d-1)= n-3,
n-3 を4つの非負整数に分ける。
n個から仕切りを3つ選ぶ.。
 C(n,3) = n(n-1)(n-2)/6,
0453132人目の素数さん2020/04/27(月) 08:59:47.25ID:R+jFPCfx
>>451
その通りであっている。
例の >>720は意味不明なことを言っているから無視しろ、
そいつの頭の中では別の関数 y = f(x) = (a+k)x の話をしているんだろう…。

y = f(x) = kx
において、 xがa倍になればyもa倍になる。
いわゆる、 f(2x) = 2 f(x) が成立する扱いやすい関数だぁね。

*********************************
一般のxの1次関数

y = f(x) = kx + D
でDがゼロの場合に見かける式ッス。
0455132人目の素数さん2020/04/27(月) 12:55:38.41ID:NmPFwCj5
>>413
A が 2x2行列のとき
 A^n = O ⇔ A^2 = O
(略証)
 A^n = O,
から
 det(A)= 0,

2次のハミルトン・ケーリーから
 A^2 = tr(A) A - det(A) E = tr(A) A

∴ O = A^n = {tr(A)}^(n-1) A,
∴ tr(A)= 0,
∴ A^2 = O,      (終)
0456132人目の素数さん2020/04/27(月) 13:46:46.30ID:NmPFwCj5
↕ 3次以上では成り立たない。
n≧3 とする。
n次の右シフト行列R(主対角線の右上だけ1, 他は0)
左シフト行列L(主対角線の左下だけ1, 他は0)
に対しては A, A^2, ・・・・, A^(n-1)≠ O, A^n = O.
0457132人目の素数さん2020/04/27(月) 16:16:45.16ID:NmPFwCj5
>>431
 z ≦ 2x は十分条件で不要、2x < y+z は自明ですね。

三角不等式
 y-x < z < x+y,
は成立っていると仮定します。
 x - z/2 ≦ x - y/2 ≦ y/2,
 z/2 - x ≦(x+y)/2 - x = (y-x)/2 ≦ y/2,
より
 |x - z/2| ≦ y/2,
が成り立ちますから、残る問題は
 y < x + z/2,
のみです。

・y ≧ x + z/2 の場合
 (x, y, z/2)が三角形にならない。→ 不可

・y < x + z/2 の場合
三角不等式
 |x - z/2| ≦ y/2 < y < x + z/2,
が成り立ち、無限に(操作)を施すことができる。
(x, y, z) → (x, y, z/2) → (x, y/2, z/2) → (x/2, y/2, z/2) →

答 2|a-b| < c < a+b.
0458132人目の素数さん2020/04/27(月) 16:42:00.02ID:KIlC6vob
>>457
>2x < y+z は自明ですね。

本当だ、気が付かなかった

>z ≦ 2x は十分条件で不要

それは違くない?
(x, y/2, z/2) → (x/2, y/2, z/2)
が成立するには、
z/2 ≦ x
が必要では?
0459132人目の素数さん2020/04/27(月) 16:46:29.72ID:KIlC6vob
>>457
>>458
>2x < y+z は自明ですね。
>
>本当だ、気が付かなかった

あれ、ちょっと待って
等号不成立(2x ≠ y+z)は言える?
0460132人目の素数さん2020/04/28(火) 00:57:13.31ID:NzvESDop
>>458
(5, 11, 14)→(5, 11, 7)→(5, 11/2, 7)→(5, 11/2, 7/2)→(5, 11/4, 7/2)
の初めの2つは z>2x です。
(1回目/3回目、2回目/4回目で同じ数を半減しています。)
3つ目で z<2x となった後は、ローテーションします。

>>459
たしかに。仰るとおり。正三角形は除外するのかも。
0461132人目の素数さん2020/04/28(火) 01:13:26.35ID:Oig1Nv2X
>>460
>(5, 11, 14)→(5, 11, 7)→(5, 11/2, 7)→(5, 11/2, 7/2)→(5, 11/4, 7/2)
>の初めの2つは z>2x です。
>(1回目/3回目、2回目/4回目で同じ数を半減しています。)
>3つ目で z<2x となった後は、ローテーションします。

なるほど
この後に、(5, 11/4, 7/2) → (5/2, 11/4, 7/2) となるから、これが(5, 11/2, 7)と相似になるのか
それなら z > 2x の場合も同様に証明できるかな?

>たしかに。仰るとおり。正三角形は除外するのかも。

あ、そうか
正三角形は無理なんだった
2x < y+z
は必要条件みたいだね
0462132人目の素数さん2020/04/28(火) 12:22:12.91ID:FTahPpkN
1辺の長さが1の正方形ABCDの辺AB,AD上に、それぞれAP=x,AQ=x(0<x<1)となる点P,Qをとる。
正方形から△APQを取り除いた五角形S(五角形PBCDQ)の周上に2点X,Yをとる。

(1)線分XYを直径とする円とSの共有点の個数の最小値はいくつか。

(2)同様に最大値はいくつか。またそれを与えるX,Yの位置を1つ述べよ。
0463イナ ◆/7jUdUKiSM 2020/04/28(火) 18:01:51.15ID:Q5cWNrtc
>>378
>>462
(1)最小値3
∵BDが直径でも外接円はCを通る。
(2)BC上にX,DQ間にYをとり、
x<(1-x)/2すなわちx<1/3のとき、
最大値10
x=1/8とすると、
BX=DY=1/8
正方形の中心Oに外接円の中心をあわせると、
外接円の半径rはr=OX=OY
PQの中点をMとすると、
OM<OX<OPのとき外接円は正方形と10の交点ができ、
OM=(3/8+1/16)√2
=7√2/16
=0.618718434……
ピタゴラスの定理より、
OP=√{(3/8)^2+(1/2)^2}
=√(9/64+16/64)
=5/8
=0.625
7√2/16<OX<5/8
x=1/8=0.125だから、
1/8<XとなるXとして、
たとえばX=0.133……=2/15を選ぶと、
OX=√{(1/2-2/15)^+(1/2)^2}
=√{(121+225)/900}
=√346/30
=0.620035841……
OM<OX<OPに収まる。
0464132人目の素数さん2020/04/28(火) 19:48:06.85ID:NzvESDop
>>462
 円周とSの周の共有点ですかね。(つまり、内部を除く。)
 共有点はつねに有限個としていいですか。
0465132人目の素数さん2020/04/28(火) 21:23:03.29ID:hh5HZHuQ
lim[n→∞]n√(1+1/n)^2-2(1+1/n)cos(π/n)+1
の答えは何になりますか?
教えて下さい
√以下は全て√の中に入ってます
0466132人目の素数さん2020/04/28(火) 22:22:38.59ID:vd3js1Pr
>>465
(1+1/n)^2-2(1+1/n)cos(π/n)+1
= 1+2/n +1/n^2 - 2(1+1/n)(1 - π^2/2n^2 + o(1/n^2) ) + 1
= (1+ π^2)/n^2 + o(1/n^2)

n √{...} = √{1+ π^2 + n^2 * o(1/n^2)} → √{1+ π^2} (n→∞)
0468 【だん吉】 2020/04/29(水) 00:53:35.79ID:pHutbusZ
>>463補足。
>>462(2)の末尾。
∴Xの位置の1つが示された。
Yの位置はOについてXと点対称だから、
DQ間のDから2/15の位置にとればいい。
0469132人目の素数さん2020/04/29(水) 10:03:05.49ID:RTVe42Je
ホモロジーの加法性公理って和が有限個の時は他の公理から導けるんでしょうか?
ご教示おねがいします
0472132人目の素数さん2020/04/29(水) 14:00:41.55ID:RTVe42Je
>>471
まさにちょうどこの講義ノートを見て、導けるのか思ってと質問したところです
質問してから気づいたのですが当該の問題6.8の主張はそもそも間違っていますよね?
S^1を開集合の和で書いたら成り立たないですし
証明のどこがおかしいのかはまだ理解できていませんが
0473132人目の素数さん2020/04/29(水) 14:04:17.16ID:Mk0K+WWV
>>469
X∪Yをdisjoint unionとする。
i:H(X,φ)→H(X∪Y,φ)
p':H(X∪Y,φ)→H(X∪Y,Y)
a:H(X,φ)→H(X∪Y,Y)
とする。p'i=aは切除同型だからa^(-1)がとれる。
そこでp=a^(-1)p'とおく。
同様にして
j:H(Y,φ)→H(X∪Y,φ)
q':H(X∪Y,φ)→H(X∪Y,X)
b:H(X,φ)→H(X∪Y,X)
q=b^(-1)q'
とおく。
この時
pi=1x、qj=1y、pj=qi=0
ker q=im i
であるから
H(X∪Y)はH(X)とH(Y)の直和である。
0475132人目の素数さん2020/04/29(水) 22:12:49.40ID:5ystJe2X
東大工学系の院試の問題が100%解ける程度では数学が得意とは言えませんか?
0476132人目の素数さん2020/04/29(水) 22:24:57.82ID:YfQbj77o
どういう集団の中での話なのかで違ってくるだろう
世間一般のレベルで言えば得意と言っていいよ
0477132人目の素数さん2020/04/30(木) 09:27:54.58ID:iXXkUTqo
>>475
得意って言っていいと思うよ、
少なくとも受験数学は得意で、
十分に学力は高いと言える。
数学者としての才能については知らん。

そういえば、学者として実績を出す人って
地方の旧帝大が多いよな。
あれくらいの学力の高さが…
もっとも学者に向いているんだろうか…。
0478132人目の素数さん2020/04/30(木) 09:32:33.73ID:lj0AFPzq
基礎的な研究ってそれに対するモチベーションがあることのほうが重要なんだろうね
地頭は旧帝程度でも十分
もちろんもっと賢い人にそのモチベーションがあればもっとすごい業績を上げる可能性はあるだろうけど
0479132人目の素数さん2020/04/30(木) 11:09:23.20ID:BomaO0ub
xy平面の各格子点に人が立っており、時刻0において(0,0)に立っている人(以下単に(0,0)と書く)はCウイルスに感染している。
各時刻n(=1,2,...)において、Cウイルスの感染者に隣接する格子点に立っている人は、確率pでCウイルスに感染する。

(1)時刻2において(1,1)が感染し、時刻3において(1,0)が初めて感染する確率をpで表せ。

(2)時刻3において(1,0)が初めて感染する確率をpで表せ。

(3)各時刻n,n+1,n+2において(n,0)が感染している確率a[n]を、それぞれpとnで表せ。

(4)a,bは非負整数でa≧bとする。時刻aにおいて(b,0)が感染している確率を、pとnで表せ。

(5)a,bは非負整数でn≧a+bである。時刻nにおいて(a,b)が感染している確率を、pとnで表せ。
0481132人目の素数さん2020/05/01(金) 02:50:10.74ID:EID36hmC
関数方程式
f(y'')+f(y')=f(y)
の定数でない解を1つ求めよ。
ここでy=f(x)であり、したがってf(y)=f(f(x))である。
0482132人目の素数さん2020/05/01(金) 07:36:10.43ID:ck1SUGCf
ひとつ見つけるだけならf(x)=xとして微分方程式解けばいいだけじゃね
0484132人目の素数さん2020/05/01(金) 08:39:37.55ID:7sZbP7cd
こんなのが解けたとして、
将来なにのやくに立つのさ?
0486132人目の素数さん2020/05/01(金) 09:39:58.77ID:0/Nf7kiT
ナブラの導入をやっているのですが、偏微分をやった次の
「Fを、成分が関数であるベクトル値関数F=(Fx,Fy,Fz)として
∇・F=∂Fx/∂x+∂Fy/∂y+∂Fz/∂z


という下りがよく分かりません
「成分が関数であるベクトル値関数F」というのはどういうことですか?
0488132人目の素数さん2020/05/01(金) 10:57:56.45ID:kSfPXdSD
ヴェクトル F が変数(x,y,z)と共に変わるとき、ヴェクトル値関数とよぶ。

ヴェクトル値関数Fが 変数(x,y,z)の属する空間と同じ3次元空間に属する場合には
F =(Fx, Fy, Fz)と表わされ、∇などが定義される。
0490132人目の素数さん2020/05/01(金) 12:22:48.57ID:hNadpfzg
関数方程式
y+f(y)+f(y')=ye^(-x)
の一般解を求めよ。
ただしy=f(x)である。
0491132人目の素数さん2020/05/01(金) 13:28:06.39ID:7sZbP7cd
>>484-485
回答が適当すぎる。
>>484 は良い質問なのに…。

そんなんだから、数学や工学は専門バカだと言われるんだよ。
俺の知っている科学者は、文学や詩歌などにも精通している。
ここの専門バカとは違う。
0495132人目の素数さん2020/05/01(金) 14:44:26.84ID:7sZbP7cd
局アナのスポーツ選手や官房長官への質問よりは
良い質問だと思う。
自画自賛ですいませんね、うへへ…。
0496132人目の素数さん2020/05/01(金) 18:20:22.17ID:L6BRmC25
>>486
関数:独立変数の値に応じて関数値が決まる
ベクトル値関数:独立変数の値に応じてベクトルが決まる
0497132人目の素数さん2020/05/01(金) 19:01:50.84ID:7sZbP7cd
「π じゃなくて
τ = 2 π を満たす τを標準にすべき」

って外人がキレ散らかしてる動画でワロタw

お前らはどう思う?
0498132人目の素数さん2020/05/01(金) 19:04:10.18ID:7sZbP7cd
現実に回転について考える時、
きっちり 1回転 をすれば元の状態に戻る。
よって、この1回転を回転での基本単位とする。 (360度であり 2π [rad] )。

ここで不自然なのは、1回転が 2π [rad] であること。
1回転が基本単位ならば、 τ [rad] (τ = 2π) として、
これを標準の記法として採用するのが明らかに合理的。

円周の長さ 2πr も 半径 r の 2π倍
→ 半径 r の τ 倍 と書ける。

sin x などの波も 1区間を 2π じゃなくて、 τ と書ける。
(1区間 なのに 2π って…なんで いちいち、 2倍しないといけないんだよ)

πはオワコン、τこそが最強!
ヤード、ポンド法からメートル法に切り替えるような感じ。
0499132人目の素数さん2020/05/01(金) 19:13:09.33ID:vc8wXdUG
ものすごくどっちでもいー
0500132人目の素数さん2020/05/01(金) 19:36:59.65ID:7sZbP7cd
テキストも論文もどのページも 2π だらけでウンザリ

これをτに統一したら、 インク代が 数% 減らせるわ

だいたい、直径と円周の長さで定義されるとかいうのが不自然すぎるわ、π。

半径r (円の中心と円の上の点の距離) と 円周の長さで
定義されたτの方が自然だろ。
0501132人目の素数さん2020/05/01(金) 20:18:03.23ID:8ge44aj8
>>500
ざっと1割くらいが2πって書いてあると言うことか。
なんでそんなの読んでるの?
0502132人目の素数さん2020/05/01(金) 21:01:46.43ID:TiGoQfo8
半径が重要と認識されたのは後からのことで、古代では直径が基本だった
そして今さら書き換えられない
0504132人目の素数さん2020/05/01(金) 21:09:32.39ID:OpQxHUgD
πだったところがτ/2になるわけだけど、そんなにインク量減るかな?
0505132人目の素数さん2020/05/01(金) 21:30:42.52ID:xwH/JTK3
π暗記厨がそろばんも得意だとして2πをそらんじる速度は何パーセント減くらいになるだろう?
ほぼ変わらんのかな
0506132人目の素数さん2020/05/01(金) 21:42:25.36ID:TiGoQfo8
>>503
ラジアンは扇の長さで定義してるからつかないよ
0507132人目の素数さん2020/05/02(土) 00:43:42.66ID:s1ev2h+J
>>505
それ面白いな!
できたら
π^2も頼むわ
0508132人目の素数さん2020/05/02(土) 03:00:32.43ID:sUcCey6n
実数a,bはa≠bかつab≠0を満たす。実数xについての関数
f(x)=ax(x≧0),bx(x<0)
を考える。
このとき実数値をとる関数g(x)で、f(x)との積f(x)g(x)が区間(-∞,∞)で微分可能となるようなものを全て決定せよ。
0511132人目の素数さん2020/05/02(土) 03:16:57.60ID:RsW8vkfg
おっと
g(√|t|)とg(-√|t|)が共に可積分だ。
同値条件が一意に定まるわけない。
0513132人目の素数さん2020/05/02(土) 04:17:58.23ID:cc3iOZ6v
x≠0 では f(x)も g(x)も微分可能だから
 f(x) g(x) も微分可能。

x=0 では
 lim[x→+0]f '(x)= a,
 lim[x→-0] f '(x)= b,
∴ |f(x)|≦ Max(|a|, |b|)|x|= c|x|,
 g(0)= 0
ならば
 |g(x)|< M|x| (←微分可能)
 |f(x)g(x)|< Mc xx,
 {f(x) g(x)}' = 0.
0516132人目の素数さん2020/05/02(土) 12:39:31.14ID:7rFsNYIK
e^(iπ) = -1

πはオワコンなので、ここでは新しく
τ = 2π を採用する。
すると、

e^(iτ) = 1

美しさがアップ!!
0517132人目の素数さん2020/05/02(土) 15:06:59.79ID:py6kZ/lG
円周率の定義について
森毅が書いていた
0518132人目の素数さん2020/05/02(土) 17:20:35.31ID:9OGDAERP
>>497
俺ら下民が円周率使うのって精々円筒状のもんの円周測るときだけだから
円周を直接測るか直径測って3.14掛けるだけだしなあ
2倍して掛けるのめんどくせーわ
半径とか他の部分で仕事する上級国民様は両方覚えたらいいんじゃね?って感じだけど
上級国民様も二つ覚えるのはなんだかんだウザイから2πですませてるんだろうな
手書きしたらτとtとかTは紛らわしくなるせいかも
0519132人目の素数さん2020/05/02(土) 18:53:10.27ID:Wl01iXlk
単位円C:x^2+y^2=1があり、C上に長さLの弧(0<L<2π)を1つとり固定する。
またaを実数とし、実数xについての関数f(x)=2π(x^2+ax)を定める。

いま、点(cosf(x),sinf(x))がLに含まれるような正の実数xの範囲を考えたい。

(1)このようなxの範囲は、重なりのない無限個の閉区間[s_1,t_1],[s_2,t_2],...の和集合Kとして表される。
s_1とs_2をaで表せ。ただしs_1はKに属する最小の実数であり、s_1<s_2<...である。

(2)Kの長さL(K)を以下のように定義する。
『L(K) = lim[j→∞] Σ[i=1,...,j] (t_i - s_i)』
L(K)が収束することを示し、その値をLで表せ。
0521132人目の素数さん2020/05/02(土) 20:39:39.86ID:hxKz/47B
数学初心者で用語の使い方等適切でないかもしれませんが質問させてください。
全ての実数関数の集合(こういうものが考えられるのか分かりませんが)を考えたとき、
それらの関数からそれらの関数の定数を無くした関数への写像を考えて、その終集合の関数にその微分を対応させる写像を考えたときに、この写像は単射ですか?単射ならどういうふうに証明するのか教えてください。
0522132人目の素数さん2020/05/02(土) 20:46:42.11ID:A33YzmSX
要は原始関数が定数を除いて一意に定まるかという話ですね
0523132人目の素数さん2020/05/02(土) 20:52:53.14ID://ZYrmNR
厳密な数学の言葉でのべたいなら、商集合に関する写像の取り扱いを勉強すればええんとちゃう
微分可能な関数集合について、定数差の違いで商とった集合について微分するて作用はwell-definedになるし
0526132人目の素数さん2020/05/02(土) 21:17:35.29ID:hxKz/47B
>>522
あっ、そうですね!どうやって示すのでしょうか?
>>523
同値類に微分を対応させる写像ということですか?
0527132人目の素数さん2020/05/02(土) 22:25:59.89ID://ZYrmNR
>>526
それで良いんじゃねと思う
0528132人目の素数さん2020/05/02(土) 22:35:40.53ID:hxKz/47B
原始関数 一意性で調べたら出てきました。お二方ありがとうございました。
0529132人目の素数さん2020/05/03(日) 05:10:35.75ID:zBmTMjFD
サイコロを振ってその出目を当てる遊びをしているとして
当てられる確率は通常は1/6だと思うのですが、「1を絶対に選ばない」と決めている人がいるとします。
この人が出目を当てられる確率も1/6なのでしょうか?
0530132人目の素数さん2020/05/03(日) 05:23:56.85ID:jm0BDyF/
三角形ABCの内接円とABとACの接点をD、E、内心をIとしAIとDEの交点をFとする
内接円上の(AF上にない)任意の点GとA、Fの3点を通る円は内接円と直交することを示せ。
0531132人目の素数さん2020/05/03(日) 06:04:30.91ID:cPIDocM4
>>529
そうです。
どういう理由付けの選択をしても1/6です(7とか答えない限り)
0532132人目の素数さん2020/05/03(日) 06:38:58.31ID:oVWU3N4J
n=0,1,2,...に対して、格子点(n,0)および(n,1)全体からなる集合をSとする。
時刻tにおいてSの要素の格子点(a,b)がCウイルスに感染しているとき、時刻t+1において隣接する格子点も確率pで感染する。ただしx座標が減少する方向へは感染しないものとする。

いま、時刻0で(0,0)がCウイルスに感染している。

(1)t≧2とする。時刻tにおいて(1,1)が感染している確率をpで表せ。

(2)時刻tにおいて(m,0)が感染している確率をP(t,m,0)、(m,1)が感染している確率をP(t,m,1)とする。
以下の2つの極限の大小を比較せよ。
lim[t→∞] P(t,2020,0)
lim[t→∞] P(t,2020,1)

(3)(100,0)は感染しない点であるとする。この仮定のもとで(2)の大小比較をせよ。
0533132人目の素数さん2020/05/03(日) 09:47:55.81ID:lzXtYV1z
>>532
感染した点が自然治癒するかどうかの情報が無いが、とりあえず決して治癒しないものとして考える。

時刻 t=1 において格子点(1,0)が感染していない確率は 1-p であるが
時刻 t=1 から時刻 t=1.1 の間に隣の点へ感染するかの判定は無限回(連続濃度回)行われており
p≠0 であれば、大数の法則から時刻 t=1 から時刻 t=1.1 の間に1度も感染しない確率は0である。
1度でも感染すれば、自然治癒がないので時刻t=1.1においては感染していることになる。
したがってP(1.1,1,0)=1 である。同様に P(1.2,1,1)=1 であるから t≧2 において常に P(t,1,1)=1 である。

以上のことから
(1) p=0 のとき 0 ,p≠0 のとき 1
(2)および(3) p=0のときは両方0、p≠0のときは両方1であるから、いずれにせよ大小は等しい
0534132人目の素数さん2020/05/03(日) 12:51:22.76ID:wydbx/i3
a,bを実数の定数として、x^2+axy+y^2=bの表す図形をSとする
b≠0のとき、Sが1直線または2直線になるためのa,bの条件を求めよ
0535132人目の素数さん2020/05/03(日) 13:23:00.31ID:04epL35S
3月の宿題で(1)のみ正解の数弱@shukudai_sujaku

昨年度の大学への数学(大数)での勝率は、

学コンBコースが 1/1 = 100% ,

宿題が 3/10 = 30% でした!

宿題の勝率が低すぎると思うので、

これからは一層精進していきたいです!

https://twitter.com/shukudai_sujaku
https://twitter.com/5chan_nel (5ch newer account)
0536132人目の素数さん2020/05/03(日) 13:36:15.08ID:WmACQ9m2
>>534
x^2+ayx+y^2-b=0
2x=-ay±√{(ay)^2-4(y^2-b)}
=-ay±√{(a^2-4)y^2+4b}

i)
(a^2-4)y^2+4bがyに依らない実定数になる場合
a=±2かつb≧0
このとき直線になる

ii)
(a^2-4)y^2+4bが(py-t)^2の形になる場合
p^2=(a^2-4),2pt=0,t^2=4b
@p=0の場合
a=±2でi)に同じ
At=0の場合
b=0より
2x=-ay±√{(a^2-4)y^2}=-{a±√(a^2-4)}y
|a|≧2かつa±√(a^2-4)≠0が必要だが後者は常に満たされるので
b=0かつ|a|≧2

したがって
(a=±2かつb≧0)または(b=0かつ|a|≧2)
0537132人目の素数さん2020/05/03(日) 13:39:29.57ID:gDS/wFg0
>>525

円の面積は πr^2 より τr^2/ 2 の方が自然だろ。

円周 τr を積分して求められるんだから
r の積分で係数に 1/2 がついている方が分かりやすい。

球の体積だって r^2 を積分するから 1/3 が出るじゃん。
0538132人目の素数さん2020/05/03(日) 14:29:54.21ID:wydbx/i3
>>536
一直線を表すとき、(a^2-4)y^2+4b=0となり、
y^2=4b/a^2-4を考えないのは、なぜなんですか?
0539132人目の素数さん2020/05/03(日) 15:00:24.56ID:MAAXTS1W
円周=τr
表面積=τr^2
円の面積=1/2τr^2
球の体積=1/6τr^3

確かにτを使う方が規則性が見えていいね
0540132人目の素数さん2020/05/03(日) 15:01:50.67ID:MAAXTS1W
前世違うことを書いてたので忘れて下せえ
0541132人目の素数さん2020/05/03(日) 15:40:31.12ID:25w/KAgu
規則性て・・・
n次元単位球の体積はπ^(n/2)/(n/2)!だぞ・・・
0542132人目の素数さん2020/05/03(日) 15:54:42.82ID:j+z0AFt4
学術の巨大掲示板群 - アルファ・ラボ
ttp://x0000.net

数学 物理学 化学 生物学 天文学 地理地学
IT 電子 工学 言語学 国語 方言 など

PS 連続と離散を統一した!
ttp://x0000.net/topic.aspx?id=3709-0
0543132人目の素数さん2020/05/03(日) 16:23:48.03ID:gDS/wFg0
>>539
だよなぁ?

積分をキッズに教えるときに分かりやすい。

例. 切り株の年輪の長さ と その断面積

A. πの場合、
円周 r *2 π → 円の面積 π r^2

B. τを用いた場合
円周 (τ) * r → 円の面積 (1/2) * (τ) r^2

円の面積の係数に (1/2) が出ており、
一般の積分の規則のデモンストレーションになっている。

r^n --> 1/(n+1) * r^(n+1)
0544132人目の素数さん2020/05/03(日) 16:41:07.31ID:gDS/wFg0
球体(球)の表面積と球体体積

●例. タマネギの皮 と 玉ねぎの体積について

まず、球(タマネギ) を「それを包み込むシリンダ」に入れる。

球の中心点から球の表面上の1つの点へ線を伸ばしていき、
貫通して、シリンダのロール部分へ投射されるとして…
これを全ての球面上の点でおこなうと、 ちょうど、ロール部分の全てへ投射される。

よって、球の表面積は 「それを包み込むシリンダのロールの面積」 と等しい

シリンダからロールを剥がし、まっすぐに伸ばして長方形にすると

高さ 2 r * 2π r = 4 * π r^2 (これは、球が地面に作る影の円… の面積4つぶん)

4 π r^2 , これを τで表すと 2τ r^2

係数が 4から2に変わっただけだが、
4 = 2x2 と因数が2つ、 これ が 2 だけと因数1つになったのでスッキリする。


●球の表面積がわかったので、次に球の体積を求める

A. πの場合、
4 * πr^2 → 球体 4/3 * πr^3

B. τを用いた場合
(1/2)* τr^2 → 円の面積 (2/3)*τr^3

4/3 より 2/3 の方がスッキリする。
0545132人目の素数さん2020/05/03(日) 16:42:47.41ID:gDS/wFg0
πよりもτの方が
明らかに合理的、かつ、教育的。

異論は、それなりに、認める。
0546132人目の素数さん2020/05/03(日) 17:14:36.67ID:25w/KAgu
だからさ・・・
n次元単位球の体積はπ^(n/2)/(n/2)!だよ・・・
0548132人目の素数さん2020/05/03(日) 18:02:13.06ID:7kK8LrHZ
>>547
計算機科学的ににπの整数n倍or逆整数1/n倍かつ計算量理想であるネイピア値2.718281828…に最も近い
3.141592653…が最も計算機負担が軽いと勝手に思ってる。

いや俺が考えている根拠は別に有るんだが円周率との話の連続性は無いんだ。何でネイピア数かと言うと
理論計算機科学的に最も高効率な整数進数はネイピア数に最も近い整数3で
これが実数適用可能なハウスドルフ次元だとネイピア数2.718281828…次元が理想ではあるが
しかし計算機工学的に実現も運用も最も容易で、3の次にネイピア数に近い整数2が選ばれているなんて
プロじゃなきゃ其の本当の所を理解しきれない所から、勝手に
『数学五大定数の一つ円周率はネイピア数に近いほど良い』と言ってみたんだ。無論、論理飛躍。
0550132人目の素数さん2020/05/03(日) 19:40:30.56ID:25w/KAgu
>>549
もちろん半整数の階乗はガンマ関数で定義されてると思って書いてるよ
0551132人目の素数さん2020/05/03(日) 19:41:51.62ID:mG0FVK1E
πよりも "椅子" 、eよりも "テーブル" の方が合理的なんじゃないかな?
異論は認めないので、よろしく。
0552132人目の素数さん2020/05/03(日) 19:45:03.68ID:25w/KAgu
ヒルベルト乙
0554132人目の素数さん2020/05/03(日) 21:48:50.28ID:wydbx/i3
誰か538の質問に答えてくれませんか?
yがxの従属関数だから、という理由だとは思うんですが、なんか納得できません
0555132人目の素数さん2020/05/03(日) 22:22:50.41ID:qRR9aAsj
点Oを中心とする半径1の円CとOを中心とする正方形Tが異なる8点で交わっている。CとTの一方に含まれ、他方に含まれない部分の面積の総和をSとする。Sが最小となるとき、正方形Tの一辺の長さを求めよ。
お願いします
0556132人目の素数さん2020/05/03(日) 22:31:45.45ID:25w/KAgu
>>554
その通り
aとbは定数なんだから>>538のように勝手に
y^2=(aとbの式)←つまり定数
なんて置けないよ
もし置いてしまったらyの値が決まってしまうだけじゃなく
元々の式からxの値も決まってしまうからね
(もしyの値を決めてもxの値が不定になるような時ならいいけど、この場合そうなってないからダメ)
0557132人目の素数さん2020/05/03(日) 22:52:18.14ID:wydbx/i3
>>556
ありがとうございます。
納得できました。
0558イナ ◆/7jUdUKiSM 2020/05/03(日) 23:34:16.56ID:kS4ZQJRT
>>480
>>555
Oから正方形Tの1辺までの最短距離をtとおくと、
正方形の面積が単位円の面積が等しくなるとき、正方形および単位円の境界線の内外での過不足がちょうど相殺されてSは最小になるから、
4t^2=π
t^2=π/4(パイパーヨン)
t=√π/2
正方形の1辺は、
2t=√π(るーとぱい)
0559132人目の素数さん2020/05/03(日) 23:49:37.02ID:25w/KAgu
マジ?
自分は計算したら一辺が4/√5になった
0560sage2020/05/04(月) 00:01:23.91ID:0INxFrMF
>>555
4/√5
0561132人目の素数さん2020/05/04(月) 00:39:50.61ID:0INxFrMF
>>555
求める長さを x (√2<x<2) とすると、S=xx-2x√(4-xx)+8arccos(x/2)-π である。
dS/dx = 2x-4√(4-xx) = 0 を解いて x = 4/√5
0562132人目の素数さん2020/05/04(月) 02:39:29.99ID:jDRWX2Ph
3月の宿題で(1)のみ正解の数弱@shukudai_sujaku

昨年度の大学への数学(大数)での勝率は、

学コンBコースが 1/1 = 100% ,

宿題が 3/10 = 30% でした!

宿題の勝率が低すぎると思うので、

これからは一層精進していきたいです!

https://twitter.com/shukudai_sujaku
https://twitter.com/5chan_nel (5ch newer account)
0563132人目の素数さん2020/05/04(月) 03:10:49.60ID:Dc16gyp2
∫[1,∞] exp(-x)ln(x) dx
は収束する。その値を求めよ。
値だけでなく、解答に至る過程も簡潔に記述すること。
0564132人目の素数さん2020/05/04(月) 03:39:10.10ID:sAooM0TB
部分積分により
∫[1,∞]exp(-x)・log(x)dx
 =[ -exp(-x)・log(x) ]+ ∫[1,∞]exp(-x)/x dx
 =[ -exp(-x)・log(x) + Ei(-x) ](x=1,∞)
 = - Ei(-1)
 = 0.21938393439・・・・
0565132人目の素数さん2020/05/04(月) 03:40:59.59ID:sAooM0TB
円Cの内部かつ正方形Tの外部である部分の面積は
 S1 = -x√(4-xx)+ 4arccos(x/2),
円Cの外部かつ正方形Tの外部である部分の面積は
 S2 = xx -x√(4-xx)+ 4arccos(x/2)+ xx,
それらの和は
 S = S1 + S2,
それらの差は正方形Tの面積と円Cの面積の差
 S2 - S1 = xx - π,

x = 4/√5 = 1.788854382 のとき
 S1 = 0.254590436
 S2 = 0.312997782
 S = S1 + S2 = 0.567588218
 S2 - S1 = 16/5 - π = 0.058407346

x > 1.77777777 = 16/9
0566132人目の素数さん2020/05/04(月) 04:03:52.42ID:sAooM0TB
ところで、
 S2 - S1 = xx - π = 0,
とおけば円積問題となる。これが成り立つのは
 x = √π = 1.77245385・・・・
のときで
 S1 = S2 = 0.28445855


エジプトのリンド・パピルス(1650 BC)には有名な円積問題に関連して
 √π ≒ 16/9
と看做したことに相当する解法が述べられているという。

"直径からその 1/9 を取り去り、残りの部分の上に正方形を作る”

数セミ増刊「数の世界」日本評論社(1982) p.77
0567イナ ◆/7jUdUKiSM 2020/05/04(月) 05:33:22.53ID:yAlzGnAp
>>558
そっか、円の膨らみのぶんだけ正方形をおっきしていいってことか。
面積S出して微分か。
;;;;;∩∩;/;;;;;;;;;;;
;;⊂(-_-;);⌒つ;;;;;;;
 ̄ ̄ ̄ ̄`υ ̄ ̄ ̄];;;;
~~~~~~~~~~~~~~~~~~~
0568132人目の素数さん2020/05/04(月) 07:26:38.72ID:WVGfXWfV
>>553
もうええわ
πでもτでも好きにしろ
おれが勝手にτ を使うから
お前らは使うなよ?

くたばれ、円周率厨ども
0572イナ ◆/7jUdUKiSM 2020/05/04(月) 09:15:42.62ID:yAlzGnAp
>>567俺がもっともわかりやすい答案を作らないといけない。俺がやらなきゃだれがやる。
>>555
点Oを中心としたxy平面に正方形Tを正対させ、正方形Tの1辺までの最短距離をtとおき、
正方形の面積は4t^2
点(0,t)、点O(0,0)、単位円と正方形が交差する点(√(1-t^2),t)を結ぶ中心角をθとすると、
t=cosθ
S/8=π/8-(1/2)t√(1-t^2)-π{(π/4-θ)/2π}+(1/2)t{t-√(1-t^2)}-π{(π/4-θ)/2π}
S=π-4t√(1-t^2)-(π/2-2θ)+4t{t-√(1-t^2)}-(π-4θ)
=-4t√(1-t^2)-π/2+2θ+4t^2-4t√(1-t^2)+4θ
=4t^2-8t√(1-t^2)-π/2+6θ
S'=0よりt= ,S=
0573132人目の素数さん2020/05/04(月) 10:03:03.84ID:fx1r5+mF
>>500
それだと、円の面積はτr^2/4になるからなぁ。
球の表面積も2τr^2だし。

それなら、分数係数より整数係数のほうがまだしもじゃね?
0574132人目の素数さん2020/05/04(月) 10:04:48.22ID:fx1r5+mF
>>573
すまん円の面積はτr^2/2の間違い。
0575132人目の素数さん2020/05/04(月) 10:43:10.32ID:VvHr4vpb
>>555ですがみなさんありがとうございます
自分でやってみたら4/√5になりました
0576132人目の素数さん2020/05/04(月) 14:53:10.06ID:Xax2Zw3g
平面上に円Cと円Dが固定されており、DはCの内部に含まれる(したがってCとDは共有点を持たない)。
Dの中心をOとするとき、Dの周上を2点P,Qが∠POQ=90°を保ったまま動く。
P,QでのDの接線をそれぞれl,mとするとき、lがCによって切り取られる線分の長さをl_Cとし、同様にm_Cも定める。
l_C+m_Cの最大値と最小値を与えるP,Qの位置はどこか。
0577イナ ◆/7jUdUKiSM 2020/05/04(月) 15:48:57.24ID:yAlzGnAp
>>572
>>575どうやったらなったの?
Sをtとθで表してS'=0でいいよね?
0578132人目の素数さん2020/05/04(月) 18:07:58.14ID:Wcxwivfs
aを実数の定数とする時、θの方程式
「方程式sinθ+cosθ-a=0の解が存在する」と
「円x^2+ y^2=1と直線y+x-a=0が共有点を持つ」が同値になるのは、
x^2+y^2=1がx=sinθ,y=cosθと同値で、
直線y+x-a=0にx=sinθ,y=cosθに代入した形になっているからで合ってますか?
よろしくお願いします。
0579132人目の素数さん2020/05/04(月) 20:22:23.59ID:WVGfXWfV
問い.1

n を自然数とする

x = 24n + 1

x が 5の倍になる時を除いて、
x は 必ず素数 または 半素数 となる

↑ これを証明せよ (出典: ワイ大学ワイ学部 2020 後期)
0580A欄既卒 ◆iD93.8lby6 2020/05/04(月) 20:31:11.31ID:WVGfXWfV
>>579 を明日の昼までに解いてこい

解けなかったザコ助は
数学板を出入り禁止や ( '〜')
0581sage2020/05/04(月) 20:32:16.06ID:0INxFrMF
>>579 偽問
1729 = 24×72+1 = 7×13×19
0583132人目の素数さん2020/05/04(月) 20:41:22.76ID:J15J0e0p

なぜあからさまな構ってちゃんの相手をしてしまうのか
0584132人目の素数さん2020/05/04(月) 20:48:54.75ID:W/Y0XOK9
反例としても使えるタクシー数すげーww
0586132人目の素数さん2020/05/04(月) 21:44:15.75ID:h8sRGmLo
>>578
〉aを実数の定数とする時、θの方程式
「方程式sinθ+cosθ-a=0の解が存在する」と
「円x^2+ y^2=1と直線y+x-a=0が共有点を持つ」が同値になるのは、

〉直線y+x-a=0にx=sinθ,y=cosθに代入した形になっているからで合ってますか?

違う。y=sinθ+2cosθ, x=-cosθを代入してもy+x-a=0はsinθ+cosθ-a=0になるでしょ?
ちゃんと片方の仮定からスタートして同値変形をしていかないと示したことにはならない
0587132人目の素数さん2020/05/04(月) 22:38:25.07ID:5fT1c3ml
> 585
お前が目指しとるのは前立腺を刺激し合う同性だろ、PINK板へ去れ
0588132人目の素数さん2020/05/05(火) 00:21:21.10ID:RZ3z9pji
Aを3次の実正方行列、a,b,cをそれぞれAの相異なる列ベクトルとする。
このとき、det(A)≦|a||b||c|を示せ。ただし|x|はxのノルムである。
0589132人目の素数さん2020/05/05(火) 00:34:22.73ID:H2fT6dc1
>>579
x =(6m+1)^4 = 24m(54m^3 + 36m^2 + 9m + 1)+ 1,
 m ≠ -1 (mod 5)

x =(24m+1)^3 = 72m(192m^2 + 24m + 1)+ 1
 m ≠ 1 (mod 5)

>>581
 (6m+1)(12m+1)(18m+1)= 72{18m^3 + 5m^2 + m(m+1)/2} + 1,
 m ≡ 0,1 (mod 5)
0590132人目の素数さん2020/05/05(火) 00:44:13.37ID:ANqRnX9L
>>588
これは個人的興味なのですが
この行列の行ベクトルをs,t,uとするとき、det(A)≦√(|a||b||c||s||t||u|)も成り立つのでしょうか?
0591132人目の素数さん2020/05/05(火) 01:00:31.39ID:H2fT6dc1
>>576
円Cの半径をR、円Dの半径をr、中心間の距離をd とする。
C: (x-d)^2 + yy = RR,
D: xx + yy = rr,
題意により
 d + r < R,
 P(r・cosθ, r・sinθ)
 Q(-r・sinθ, r・cosθ)
とおく。
 円Cの中心(d,0)とlの距離 |r - d・cosθ|
 円Cの中心(d,0)とmの距離 |r - d・sinθ|
したがって
 l_C = 2√{RR -(r - d・cosθ)^2},
 m_C = 2√{RR -(r - d・sinθ)^2},

l_C + m_C の最大は θ=45゚ のとき。
  4√{RR -(r - d/√2)^2},

最小は θ=225゚のとき。(円Cと円Dがは隔たっている場合)
  その両脇の角(円Cと円Dが接近している場合)
0592132人目の素数さん2020/05/05(火) 01:04:03.67ID:T3NHJuuv
>>588
|detA|はOA=a,OB=b,OC=cとなるOABCを取り、この4点を頂点に含む平行六面体の体積に等しく、それは|a||b||c|より小さい
0593132人目の素数さん2020/05/05(火) 02:17:19.69ID:ixImTe6Q
>>586
どうして同値になるのか分からないです。
教えてもらってよろしいですか?
0594132人目の素数さん2020/05/05(火) 03:41:50.19ID:H2fT6dc1
>>588
 A =(a,b,c)
 a~ =(a1,a2,a3)
 b~ =(b1,b2,b3)
 c~ =(c1,c2,c3)
 ~は転置を表わす。
とすと、サラスの公式で
 det(A)=(a2・b3-a3・b2)c1 +(a3・b1-a1・b3)c2 +(a1・b2-a2・b1)c3
  =(a×b)・c
  = S(n・c)
  = S |c|cosθ
  =(a,b,cのなす平行六面体の体積)

 (a×b)~ =(a2・b3-a3・b2, a3・b1-a1・b3, a1・b2-a2・b1)= S n,
S はaとbが作る平行4辺形の面積、S =|a×b|≦|a||b|
n はその法線方向の単位ヴェクトル。
θ はnとcのなす角。
 S =|a×b|=|a||b|sinφ ≦|a||b|,
 (n・c)= |c|cosθ ≦|c|,
∴ det(A)=(a×b)・c = S(n・c)= S|c|cosθ ≦S|c|
  =|a×b||c| ≦|a||b||c|
0596132人目の素数さん2020/05/05(火) 10:08:29.59ID:b2IqdVzK
3月の宿題で(1)のみ正解の数弱@shukudai_sujaku

昨年度の大学への数学(大数)での勝率は、

学コンBコースが 1/1 = 100% ,

宿題が 3/10 = 30% でした!

宿題の勝率が低すぎると思うので、

これからは一層精進していきたいです!

https://twitter.com/shukudai_sujaku
https://twitter.com/5chan_nel (5ch newer account)
0598132人目の素数さん2020/05/05(火) 11:28:53.63ID:xX2V8Dbl
>>588
A = (a[1],a[2],...,a[n])
Gram-Schmidt法で列ベクトルを直交化
a[1]' := a[1]
a[k]' := a[k] - (a[k]・φ[1])φ[1] - (a[k]・φ[2])φ[2] - ... - (a[k]・φ[k-1])φ[k-1]
φ[k] := a[k]' / |a[k]'|
|a[k]'|^2 = |a[k]|^2 - 2 Σ[j=1,k-1] (a[k]・φ[j])^2 + Σ[j=1,k-1] (a[k]・φ[j])^2
= |a[k]|^2 - Σ[j=1,k-1] (a[k]・φ[j])^2
≦ |a[k]|^2 ∴ |a[k]'| ≦ |a[k]|

行列式の性質より
det(A) = det(a[1],a[2],...,a[n]) = det(a[1]',a[2]',...,a[n]') = det(A')

|det(A)|^2 = det(A'^t A') = Π[k=1,n] (a[k]'・a[k]') = Π[k=1,n] |a[k]'|^2
∴ |det(A)| ≦ |a[1]| |a[2]| ... |a[n]|

行ベクトルに対しても同様に
|det(A)| ≦ |b[1]||b[2]|...|b[n]|

よって
|det(A)| ≦ √{|a[1]||a[2]|...|a[n]||b[1]||b[2]|...|b[n]|}
0599132人目の素数さん2020/05/05(火) 12:11:59.99ID:4rzFrDuj
>>579
この問題はね、
予想が真であるか偽であるかも含めて、
それをどういうアプローチで証明するかを見たかったんよ。

>>581 みたいに馬鹿でもチョンでも出来る力業で
解いてほしくなかった。
エクセルや計算機を回して
反証が出るまで走らせただけじゃん…。
その解き方と過程に、何の価値があるというのか?
0601132人目の素数さん2020/05/05(火) 13:52:34.10ID:dF3SDVX2
>>599

> >>579
> この問題はね、
> 予想が真であるか偽であるかも含めて、
> それをどういうアプローチで証明するかを見たかったんよ。
>
> >>581 みたいに馬鹿でもチョンでも出来る力業で
> 解いてほしくなかった。
> エクセルや計算機を回して
> 反証が出るまで走らせただけじゃん…。
> その解き方と過程に、何の価値があるというのか?
そんな解き方で解ける程度の問題ってことでしょ。
0602132人目の素数さん2020/05/05(火) 13:55:59.05ID:rIr45OhO
馬鹿でもチョンでも思い付く嘘問に、何の価値があるというのか?
0603132人目の素数さん2020/05/05(火) 14:17:21.80ID:B6ZQqn9P
だからタクシー数を知ってたらすぐ分かるって話では?

1729=12^3+1^3=10^3+9^3

最初の等式からすぐに24n+1型であることもすぐ分かるし
x^3+y^3の因数分解の公式から素因数13と19を持つこともすぐ分かり、残りの素因数が10以下だから反例として確定する
0604132人目の素数さん2020/05/05(火) 14:18:07.06ID:1Wh/dMU5
学術の巨大掲示板群 - アルファ・ラボ
ttp://x0000.net

数学 物理学 化学 生物学 天文学 地理地学
IT 電子 工学 言語学 国語 方言 など

PS 連続と離散を統一した!
ttp://x0000.net/topic.aspx?id=3709-0
0605132人目の素数さん2020/05/05(火) 14:48:17.21ID:nu2fWXVa
この世に曲線などないんだよ。
あるのは直線のみ。
どんな曲線も直線の集まりに過ぎないんだよ。
そこを認めないから円周率に終わりがないんだよ。
0606132人目の素数さん2020/05/05(火) 14:55:45.61ID:1xcgG1Mk
放物線y^2=4pxについて,次の問いに答えよ。
(1)直線y=mx+nが,この放物線の接線であるためのm,nの条件を求めよ。

答えmn=p

(2)この放物線に引いた2本の接線が垂直に交わるとき,交点Pの軌跡を求めよ

(2)の方針に「2本の接線の方程式をy=mx+n,y=m'x+n'とするとき,
(1)の結果と2直線の垂直条件から,n,m',n'はすべてmを用いて表される。」
とあったんですが、mの2次方程式と解と係数の関係の他にも解き方はあるようなんですがどうなんでしょうか?
0608132人目の素数さん2020/05/05(火) 16:42:47.28ID:Vg/wAO0p
e^(x^3)のマクローリン展開はどうやって求めるのですか?

答は下記のyにx^3を代入するだけの式になりますか?

e^y = 1+y+(1/2)y^2 +(1/3!)y^3+…
0609132人目の素数さん2020/05/05(火) 16:43:46.70ID:227hHAl/
>>608
うん
0610132人目の素数さん2020/05/05(火) 17:35:54.09ID:XJcON/Dn
制約条件の下での極値について、例えば
f(x,y,z)=xyz、条件:x+y+z=10
の極値を求めよという問題で、普通に未定乗数法で極大を計算することはできるんですが、このときにλ=xy=yz=zxと条件からx≠0としてz=y,x=10-2yをfに代入したら
F(y):=f(10-2y,y,y)=y^2(10-2y)
となり、これはy=0で極小となります
しかし(10,0,0)はfの極小値ではないです

このように、f(x,y,z)の制約条件下の極値と制約条件をfに代入した1変数関数Fの極値が一致しないのは何故ですか?
0611132人目の素数さん2020/05/05(火) 17:47:26.25ID:B6ZQqn9P
>>610
λの条件をちゃんとみると
xy=yzからy≠0のときx=yもわかる
するとx=y=z=10/3

y=0のときはz=0でx=10となる(これも極値)

途中yを自由変数のように代入することは出来ないのでは?
0612132人目の素数さん2020/05/05(火) 17:57:57.19ID:wfN4eAQ9
>>608
こういう大学レベルの話、基礎解析とか線形代数とか
フーリエ変換とかイプデロ論法とか?
空気読まずに話をするやつってなんなの?
0613132人目の素数さん2020/05/05(火) 18:06:44.59ID:XJcON/Dn
>>611
>y=0のときはz=0でx=10となる(これも極値)
あれ?これ極値になる?
0<x<1としてもyとzを適当に動かせば正にも負にもなるような……

>途中yを自由変数のように代入することは出来ないのでは?
ですね、ありがとうございます
0615132人目の素数さん2020/05/05(火) 18:19:09.05ID:XJcON/Dn
>>613
>0<x<1としてもyとzを適当に動かせば正にも負にもなるような……

ここのx,y,zはh,k,l(in R)のミス
(10+h)+k+h=10⇔h+k+l=0を満たす実数
0616132人目の素数さん2020/05/05(火) 18:21:56.39ID:BDY0rhyh
>>612
ここは高校数学スレとは違う
自分に理解できない話題が出たからといって、そうやって駄々を捏ねるのは良くない
0617132人目の素数さん2020/05/05(火) 18:48:34.93ID:B6ZQqn9P
>>615
fのグラフは4次元にあるので見づらいですが、fやfを条件式上に制限したグラフ図形はy=0(もしくはx=0やz=0)という直線上でf=0という停留値をとる形になってると思います
階段を横に歩くようなイメージです
縦に歩けば上下に変化します
0620132人目の素数さん2020/05/05(火) 19:36:17.15ID:H2fT6dc1
x+y+z=10 をみたす平面Πで考えます。
f = xyz = 0 となるのは
・xy平面(z=0)とΠの交線
・yz平面(x=0)とΠの交線
・zx平面(y=0)とΠの交線
の3直線ですね。
中央の正三角形の内部で f>0 で、交線を越す度に fの符号が変わります。
2本の交線が交わった所では、時計回りに +,-,+,- となります。
これは極値ではなくて、峠点(鞍点)です。
街道方向で上に凸(f_uu<0)、稜線方向で下に凸(f_vv>0)です。
変数が2つ以上あるときに現れることがあります。

序でにいえば、これらをまとめて停留点(f_u = f_v = 0)といい、
ヘッシアンで分類されます。
 (f_uu)(f_vv)-(f_uv)^2 > 0 → 極値
 (f_uu)(f_vv)-(f_uv)^2 < 0 → 峠点
ここで(u,v)は面内座標で本問では
 x ={20 +(√6)u +(3√2)v}/6,
 y ={20 +(√6)u -(3√2)v}/6,
 z ={20 -(2√6)u}/6,
0621132人目の素数さん2020/05/05(火) 19:55:47.16ID:wfN4eAQ9
>>579 はワイの思いつきだけどさ、
不備があったのは謝る。

ただ、これにもうちょっと工夫して
条件をゆるめて改良したら何かに使えそう、
国立大の二次試験の問題とか。
0624132人目の素数さん2020/05/05(火) 21:40:21.19ID:mTkWDZlh
>>616
分からん問題はここに書いていいんだね
もし分からん問題が中学レベルでそれが中学レベルであることすら分からなくても
空気読まずに書いていいかなあ(今のところないけど)
0625132人目の素数さん2020/05/05(火) 22:08:23.93ID:ANqRnX9L
平面上に円Cと、Cの外部にある異なる2点P,Qが与えられている。

(1)Cの直径ABでPA=QBとなるものを、定規とコンパスのみで作図する手順を示せ。

(2)Cの半径を1とする。Cの弦XYで長さt(0<t<2)かつPX=QYとなるものを定規とコンパスで作図したい。このようなことは可能か。以下の【選択肢】から正しいものを選べ。
【選択肢】
(a)tの値によらず可能である。
(b)tが特定の範囲内にあるか、または特定の値の場合にのみ可能である。
(c)条件をみたすtは存在するが、そのようなどのtも定規とコンパスでは作図できない。
(d)tがどのような値であっても不可能である。
0626132人目の素数さん2020/05/05(火) 23:15:45.89ID:Vg/wAO0p
下記問題の後半「値を求めよ」
の解法はどのようにすればいいのですか。
https://imgur.com/a/TE3Nuyv
0627132人目の素数さん2020/05/05(火) 23:24:34.51ID:B6ZQqn9P
>>626
単純にn→∞として
cos(π/2^(n+1))→cos(0)=1だから
右辺の頭の1/2を移項して、2では?
0628132人目の素数さん2020/05/05(火) 23:29:46.82ID:wfN4eAQ9
1問目がヒントになってるじゃん。

cos の方を積分して微分して
しゃーってやったらいけるやん。
06316262020/05/05(火) 23:55:12.75ID:Vg/wAO0p
>>627
n→∞とは書いてないです。

>>628
詳しくお願いします。
0633132人目の素数さん2020/05/06(水) 00:06:51.13ID:f7XA6HdU
>>626
(上)
n=1 のとき
 cos(π/4)=(1/2)√2 ,
n>1 のときは半角公式
 cos(θ/2)=(1/2)√{2(1+cosθ)},
から。
(下)
 x = √(2+x),
から
 x = 2,   (必要条件)
0634132人目の素数さん2020/05/06(水) 00:23:53.18ID:oqa8Xuuv
>>633
n→∞のときの極限をxとすれば
x = √(2+x), となるが
n→∞とは書いてないです。
0635132人目の素数さん2020/05/06(水) 00:52:28.55ID:hVWkN8c/
何がわからないのかわからないというのはこういう状態のことを言うのか
0637132人目の素数さん2020/05/06(水) 01:43:38.65ID:f7XA6HdU
全部掛ければ

cos(π/4)cos(π/8)cos(π/16)・・・・ cos(π/2^n)
 = sin(π/2)/{2^(n-1)・sin(π/2^n)}
 = 1/{2^(n-1)・sin(π/2^n)}
 → 2/π,         (n→∞)

これが ヴィエタ(Viete, 1540〜1603)の公式で、
史上初の πを表わす数式 とされていました。

・Petr Beckmann:「πの歴史」ちくま学芸文庫 (2006)
  田尾陽一/清水韶光 訳 344p.1320円
http://www.chikumashobo.co.jp/product/9784480089854/
http://chikuwablog.cocolog-nifty.com/blog/2019/10/post-cd9a93.html
0638132人目の素数さん2020/05/06(水) 01:46:41.69ID:f7XA6HdU
しかし、ラィプニッツ(Leibniz, 1646〜1716)級数 或いは グレゴリー(Gregory, 1638〜1675)級数と呼ばれる
 π/4 = 1 -1/3 +1/5 -1/7 +1/9 -・・・・
が,この2人より前にインド・サンガマグラーマの数学者マーダヴァ(Madhava, 1340頃〜1415頃)によって発見されてたことが分かりました。

・加藤和也・黒川信重・斎藤 毅:岩波講座 現代数学の基礎「数論1」, p106〜

てぇこたぁ「πの歴史」の記述は誤り?
0639132人目の素数さん2020/05/06(水) 01:52:58.84ID:W1iQIMkL
初項 a_1=√2 、漸化式 a_(n+1)=√(2+a_n) で定義される数列を{a_n}とすると
>>626上の式の右辺は (1/2)a_n で表されるわけだが
解答者は lim_(n→∞)a_n の値の求め方を返答しており、
質問者は>>626下の式の値が lim_(n→∞)a_n となることに疑問がある。
という状況か。

質問者の疑問は式の意味(出題の意図)をどう解釈するか?という部分にあたるから唯一の正しい解釈というのは存在しないが
解決すべき課題を(それなりに妥当な)どのような数学モデルに解釈して解決していくかというセンスは
数学を学び、活用していくには重要なことであり、この問題の出題意図にはそういった部分も含まれていると考えられる。

おそらく>>626下の問題は上記のlim_(n→∞)a_nの値として求めるのが想定されている解答だと思われるが
もし、これよりも妥当だと思われる解釈で解答を作成できればそれが正解と認められるかもしれない。
>>626下の式の解釈として上記のlim_(n→∞)a_nの値とするよりも妥当な解釈があるのなら具体的に述べてみろってこった。
そんなものが存在するようには思えないが、可能性は否定できない。
0640132人目の素数さん2020/05/06(水) 05:38:05.67ID:8EZ+0UWu
放物線y^2=4pxについて,次の問いに答えよ。
(1)直線y=mx+nが,この放物線の接線であるためのm,nの条件を求めよ。

答えmn=p

(2)この放物線に引いた2本の接線が垂直に交わるとき,交点Pの軌跡を求めよ

(2)の方針に「2本の接線の方程式をy=mx+n,y=m'x+n'とするとき,
(1)の結果と2直線の垂直条件から,n,m',n'はすべてmを用いて表される。」
とあったんですが、mの2次方程式と解と係数の関係の他にも解き方はあるようなんですがどうなんでしょうか?
0641132人目の素数さん2020/05/06(水) 10:21:26.43ID:2vnYqcGe
>>640
連立式: y=mx+p/m,y=(-1/m)x-pm より
(x,y) = (-p, p/m-pm)
よって x=-p の直線がPの軌跡となる.  {∵ p/m - pm が動く範囲は (-∞,+∞)}
06426262020/05/06(水) 10:28:48.45ID:oqa8Xuuv
>>639
ありがとうございます。
まさにそのような回答を期待していました。
多分、問題の意味は「n→∞のとき・・・の値を求めよ」
ということだろうと思ってはいましたが、確信は持てま
せんでした。

問題の書き方は下記の形式で書いてあります。
2×2×2×・・・×2
--------n個---------

しかし、この場合n→∞として
2×2×2×・・・×2=∞とは
しないで2^n と答えるはずです。

問題の場合、n→∞としないと答えようが
ないというのは、大学の数学をある程度やった
者しか分かりません。これから大学数学をやろ
うとする者には、もしかしたらnが有限のとき
cosを使わずnの式で表せるのかなと思ったり
するわけです。
0643132人目の素数さん2020/05/06(水) 10:32:41.84ID:L6b20Al4
>>635
おれも大学の基礎解析とか
途中で分からなくなったから気持ちは分かる。

全く物事を理解していない状態だと、
己が何を理解していないのかすら認識できない。
よって、先生へ質問することすら出来ず、単位を落とす。

高校生の頃は東工大の二次試験の数学とか
そこそこ、解けてたから数学得意だと勘違いしてたけど
ワイは井の中の蛙のクソでした。
0644132人目の素数さん2020/05/06(水) 10:34:50.37ID:L6b20Al4
まぁ、ワイは恥を知らない方の
無恥なんですけどね、なんつって (^^)
えへへ…
0645132人目の素数さん2020/05/06(水) 12:06:27.84ID:hVWkN8c/
>>642
>もしかしたらnが有限のとき
>cosを使わずnの式で表せるのかなと思ったり
>するわけです。

どういうこと?
あの多重根号の式を多重根号を使わずに表したいってこと?
例えば、 n = 3 のときの
√(2 + √( 2 + √2))
の値をこれ以外の方法で表したいってこと?
何がしたいのかわからん

>問題の場合、n→∞としないと答えようが
>ないというのは、大学の数学をある程度やった
>者しか分かりません。

極限って高校でも数学Vでやるんじゃなかったっけ?
0649132人目の素数さん2020/05/06(水) 16:00:51.91ID:RG00+xls
θ1、θ2がどちらも独立に一様分布に従うとき θ1 - θ2の分布は三角分布になる というのはどうやって証明できるんでしょうか?
0650132人目の素数さん2020/05/06(水) 16:01:57.53ID:RG00+xls
>>648
正解がでてからも誤答連投する芸風も許容する寛大なスレだと思っております。
0652132人目の素数さん2020/05/06(水) 19:00:13.21ID:C7GBw+NK
ホモロジー論の本で以下のような記述があったのですがこれは正しいのでしょうか
正しいなら証明が知りたいです、よろしくお願いします

位相空間の包含写像 A⊂Xから誘導される
特異チェイン複体の包含写像 S(A)⊂S(X)についてS(A)はS(X)の直和因子である
0653132人目の素数さん2020/05/06(水) 19:48:27.67ID:1eDWV5m3
S(X)は特異写像たちを基底とする自由加群で、その中でS(A)は単体の像がAに含まれる特異写像たちだけを基底として集めてくるだけだから当然直和因子では?
0654132人目の素数さん2020/05/06(水) 20:01:08.20ID:C7GBw+NK
>>653
ああ言われてみればそのとおりですね
なぜこんなとこで詰まっていたのか…orz
ありがとうございます
0656132人目の素数さん2020/05/07(木) 03:53:32.24ID:SMpkBNpt
平面上に円Cと、Cの外部にある異なる2点P,Qが、それぞれ固定されている。

(1)Cの直径ABでPA=QBとなるものを、定規とコンパスのみで作図する手順を示せ。

(2)Cの半径を1とする。Cの弦XYで長さt(0<t<2)かつPX=QYとなるものを1つ、定規とコンパスで作図したい。
このようなことは可能か。以下の【選択肢】から正しいものを選べ。

【選択肢】
(a)tの値によらず作図可能である。
(b)tが特定の範囲内の値であるか、または特定の値の場合に限り作図可能である。
(c)tがどのような値であっても作図できない。
0657132人目の素数さん2020/05/07(木) 07:09:59.96ID:jPdQ7uxV
2以上の自然数nに対してS[n]=1+2+...+n
と定義する。
いまNが「悪い自然数」であるとは、S[N]-kが素数となる自然数k(1≦k≦N)が存在しないことを指す。
悪い自然数は存在するか。存在するならば、悪い自然数を1つ求めよ(最小の悪い自然数を求める必要はない)。
0658132人目の素数さん2020/05/07(木) 10:14:51.97ID:WvOjlRP7
>>656
(1)条件不足ではない?存在しないものを作図のしようがないし…
それとも直径ABが存在するための条件を求めることも含めて解答せよということ?

(2)一般に、作図可能な線分の長さは有理数係数の2次方程式を有限回用いて表せる数に限られる。
0<t<2の範囲に作図可能でない数は無数にあるので(a)は誤り。
同様に、特定の範囲内のすべてのtについて作図可能ということもあり得ない。
「特定の値に限って作図可能」と「どのような値でも作図できない」はどちらもあり得るが
どちらが正しいかはC,P,Qの位置関係による。
よって正しいのは(b)または(c)
0659132人目の素数さん2020/05/07(木) 11:55:01.31ID:OQGYpYA4
昨日知恵袋で質問しても解決しなかったのでお願いします

f(x_1,x_2)=x_1 ^ 2 + 3 * x_2 ^ 2
これを最小にするx_1, x_2を求めよ
ただし、x_1 + x_2 = 1
0663132人目の素数さん2020/05/07(木) 15:17:57.12ID:WvOjlRP7
>>662
△APQと直線BRについてメネラウスの定理を適用するんや。
全部外分になるからちとわかりにくいが、例えば点Aから出発して反時計回りに回ると
(AB/BP)*(PR/RQ)*(QC/CA)=1 となるで。あとはなんとでもなるやろ。
0665132人目の素数さん2020/05/07(木) 16:20:33.15ID:7ip8Cm16
>>662
ムズすぎて剥げた。

>>663
こういう幾何学の問題って
解けるかどうか (ユニークな解を持つかどうか)を
パッと見で、判別する方法とかあるのん?
0666132人目の素数さん2020/05/07(木) 16:25:04.31ID:eXMgxv1Q
見た瞬間メネラウスだから例題みたいなもんやろ
(ユニークな解)とか言ってる場合ちゃうで!中学の勉強をちゃんとしぃ
0667132人目の素数さん2020/05/07(木) 16:34:55.04ID:YxHohFI2
この世に曲線なんて存在しないんだよ。
だから円は厳密には多角形なんだよ。
これを円として扱うもんだから円周率が無限に続いてしまうんだよ。
円周率の桁が増えるほど、円を構成する角が増える関係なんだよ。
0668132人目の素数さん2020/05/07(木) 17:04:45.69ID:7ip8Cm16
>>666
ただ1つの解 っていう意味ね。
ちなみに、中学は30年前に卒業しますた。 ( '〜')b
0669132人目の素数さん2020/05/07(木) 17:22:19.69ID:3tbk6xje
>>657
解けたわけじゃないが、適当にプログラム書いて検証してみた
100万以下の「悪い自然数」はないっぽい
(違ったらすまん)

ところで、もし「悪い自然数」が存在しなければ、
S[N] と S[N+1] の間に必ず素数が存在することになるんだが、
そんなことってあり得るのか?
0670132人目の素数さん2020/05/07(木) 17:58:26.04ID:caB4p9fh
大きい自然数x付近での素数出現確率は1/logx位だから、S[N]付近では2logN個の中に1個程度
S[N-1]からS[N]までのN個の中に素数が存在する確率はNが大きくなるほど(N>>2logN)となってほぼ確実になっていくような気が…
だから悪い自然数が小さいところで見つからなければ厳しそう
0671132人目の素数さん2020/05/07(木) 18:01:55.35ID:8wL8ZgC6
>>669
RH
⇒∀ε>0∀x>>0∃p x<p<x+x^(1/2+ε)

だけどコレがε=0で成り立つかに近いからな。
答えられるとしたら反例がある場合だけど答えが「悪い自然数はない」だとかなりRHに近い。
(RHより真につよい香りがする)
百万まで探して見つからないなら5chで解けるような問題じゃない希ガス。
0672132人目の素数さん2020/05/07(木) 18:07:01.22ID:7ip8Cm16
「nと2n の間に素数が存在する」
って証明されてるよな。

あれを言い換えただけじゃね?
0673132人目の素数さん2020/05/07(木) 18:13:33.92ID:7ip8Cm16
例えば、
S(5) = 15
S(6) = 21

n = S(5) ー 5 = 15-5 = 10
2n = 20

これで 10〜20 までを守備範囲に含んでいる。



そもそも、[n 〜 2n ] に必ず素数が1つ存在するのは証明済み。

a = S(N) - N とおく、
この値を基準にすると、
a 〜 2a に素数が必ず存在する。
これはちょうど、
S(N) と S(N+1) の間のすべての自然数のセット。

だから、同じことを言葉を換えて言っているだけだな。
0674132人目の素数さん2020/05/07(木) 18:29:21.75ID:7ip8Cm16
あの…、俺の説明で
本質的な部分を勘違いしていたらごめんなさい…

みんなの使っている数式のレベルが
高すぎてついていけん
0675132人目の素数さん2020/05/07(木) 18:32:22.07ID:3tbk6xje
S[N] - N = S[N-1]
より、 N ≧ 6 のときは常に
S[N+1] < 2 S[N-1] = 2 (S[N] - N)
となるんだから、ベルトラン=チェビシェフの定理は使えなくね?
0677132人目の素数さん2020/05/07(木) 18:53:19.06ID:8wL8ZgC6
∀n ∃p s(n)-n<p<s(n) ‥‥@
∀ε>0 ∀x>>0 ∃p x<p<x+x^(1/2+ε) ‥‥A

とすると@→AでAとRHはかなり近い(RH⇒A、逆も正しいという話も聞いたことがあるけど眉唾)
ちなみに

∀ε>0 ∀x>>0 ∃p x<p<x+x^(a+ε)

の形で知られている最良の結果は確かa=5.25。
0678132人目の素数さん2020/05/07(木) 18:59:37.45ID:3tbk6xje
>>657はリーマン予想並みかそれ以上の難問かもしれないってこと?
未解決問題なの?
問題の出所が知りたい
0679132人目の素数さん2020/05/07(木) 19:50:04.65ID:LAwb6m8j
>>678
ありがとう。nと2nの間に素数がある云々から考えついて、全て合成数になることを予想した。普通に示せるだろうと予想していたが、行き詰まってここで聞いてみた
整数論をやってる人の意見が聞きたいな、工学部だからせいぜい大学院入試程度しかできないから
0680132人目の素数さん2020/05/07(木) 23:27:58.38ID:3tbk6xje
>>679
>>657の人?
自分で問題を考えて、「悪い自然数」は存在するだろうって予想したってこと?
すごいな
俺も最初は存在を示せるんじゃないかと思ったんだけど行き詰った
例えば、もし S[N] が N! だったら S[N] - 2, … , S[N] - N が全部合成数になるんだけど、実際の S[N] は N! に比べて小さすぎるから上手くいかない
だからプログラムを書いて検証したんだけど、100万まで悪い自然数が無かったから、おかしいなと思ったんだよね

そういえば、 S[N] は三角数だけど、もし悪い自然数 N が存在すれば、その S[N] の三角形のどの段を抜いても長方形に変形できることになる
はたしてそれは可能だろうか?
0681132人目の素数さん2020/05/07(木) 23:59:51.04ID:8wL8ZgC6
だから無理だって
もし必ず素数が存在する事が示せるなら
十分大きなxに対し

x-x^(0.51) <p< x

を満たす素数pが存在する事が示せてしまうけど今この形で最良のものはR. C. BAKER, G. HARMAN and J. PINTZ の定理

x-x^(0.525) <p< x

ググったら出てくる
0682132人目の素数さん2020/05/08(金) 00:44:23.80ID:IU3jiVQk
>>681
すると、あなたの主張は「悪い自然数」は存在する?存在しない?
どちらも難しすぎて証明できない?
0683132人目の素数さん2020/05/08(金) 00:50:21.84ID:gGrwQusG
>>659
f(x, y)= xx + 3yy
 =(3/4)(x+y)^2 +(1/4)(x-3y)^2
 = 3/4 +(1/4)(x-3y)^2   (← x+y=1)
 ≧ 3/4
 = f(3/4, 1/4).
0684132人目の素数さん2020/05/08(金) 02:01:09.00ID:k2HXTAi9
>>682
存在するような気はするけど100万まで調べてないならないのかも知れない。
結論
難しい。
ここで考えるような問題ではない。
0685132人目の素数さん2020/05/08(金) 09:50:17.32ID:k0ECE/UQ
存在するような気はしないでしょ
統計的にはNが100付近の時点で悪い自然数である確率は10万分の1くらいまで下がる
0686132人目の素数さん2020/05/08(金) 10:00:44.48ID:CMUD4s7e
ダメ元である区間に
半素数が1つも存在しないことを
証明しようと思ったけど無理そう、もうダメだ… orz
0687132人目の素数さん2020/05/08(金) 10:03:45.41ID:CMUD4s7e
フェルマーって本職は裁判官だったよな

アマチュア数学家がそれなりの実績を
残せたのってさ、いつの時代までだろう…
今じゃもう絶対、起こり得ないよね

微積分が発明されたあたりまでかな、
アマチュアが戦えたのは
0688132人目の素数さん2020/05/08(金) 10:27:30.10ID:WmDpVhCu
3月の宿題で(1)のみ正解の数弱@shukudai_sujaku

昨年度の大学への数学(大数)での勝率は、

学コンBコースが 1/1 = 100% ,

宿題が 3/10 = 30% でした!

宿題の勝率が低すぎると思うので、

これからは一層精進していきたいです!

https://twitter.com/shukudai_sujaku
https://twitter.com/5chan_nel (5ch newer account)
0690132人目の素数さん2020/05/08(金) 12:01:45.73ID:IU3jiVQk
>>684
冷静に考えてみると、小さな値で反例が見つからないってことは、
ルジャンドル予想のような難問である可能性が高いってことかな
ルジャンドル予想における平方数の間隔は
(n + 1)^2 - n^2 = 2n + 1
で、>>657の問題における三角数 S[n] の間隔は、
S[n + 1] - S[n] = n + 1
だけど、 S[n] = n (n + 1) / 2 だから、ほとんどルジャンドル予想の類似と言えるかな
0691132人目の素数さん2020/05/08(金) 12:20:15.03ID:k2HXTAi9
>>685
RH→∀e>0 ∀x>>0 ∃p x-x^(1/2+e) < p < x

でRHを仮定してすらこのeをハズす事に成功していない。
外せるのかも知れないけど、もちろん外せないかなと考えた人はいっぱいいるだろうけど、世界の天才達が誰も成功してない事を見るとやはりハズせない=成り立たない気がする。
しかし100万まで探してないならないのかも知れない。
結論
難しい
数学は難しくて解けない命題を探す学問ではない。
一見難しくてこんなもん解けるかと思いきや実は解ける問題を探すゲーム。
難しくて解けないだけの数学の問題なら死ぬほど転がってる。
0693132人目の素数さん2020/05/08(金) 13:11:37.92ID:k0ECE/UQ
>>691
よくわかってないんだけどRHはπ(x)とli(x)の誤差がx^(1/2+ε)程度を言ってるんだよね
それは素数の個数の和(積分)についての話で、区間(差分、積分前)での誤差(=つまりx付近での素数である確率の誤差)はx^(-1/2+ε)程度に抑えられることと対応するんじゃないの?
0694132人目の素数さん2020/05/08(金) 13:36:40.17ID:k2HXTAi9
>>693
わからん。
難しい。
p(x)=min{p | p≧x }
とおくとき
RH→limsup log(p(x)-x)/log x = 1/2
でこれからかんたんに
∀e>0 ∀x>>0 ∃p x< p < x+x^(1/2+e)‥‥(✳︎)
一方で∀n n is not bad なら
∃M p(x)-x<Mx^(1/2)‥‥(✳︎✳︎)
とかなりキツイ命題が証明できてしまう。
こんなすごい事は成立しない希ガスというのはあくまで勘。
勘は所詮勘。
もちろん勘も大切だけど所詮勘。
難しい。
ココでいつまでもグチグチグチグチ考えても誰も得しない。
0695132人目の素数さん2020/05/08(金) 13:48:58.01ID:k0ECE/UQ
>>694
→RHならキツイと思うけどRH→だと微妙

素朴に素数定理基準で考えれば
∀ε>0、∃N ∀x≧N s.t. x≦p<x+x^ε
が成り立つべき
0697132人目の素数さん2020/05/08(金) 14:48:44.02ID:IU3jiVQk
プログラムで計算したデータを眺めていたら、 S[n] - 2 が素数になるケースが頻繁に現れることに気が付いた
そこで、こんな予想を提出しようと思う

【予想】
「三角数 S[n] = 1 + 2 + … + n に対し、
 S[n] - 2
 の形の素数は無数に存在する」

あと、 S[n] - k (1 ≦ k ≦ n) が素数になるような最小の k は、 n に比べて小さいように見える

【参考】 n = 100万 の近くのデータ
999995: S[999995] - 7 = 499995500003 は素数
999996: S[999996] - 19 = 499996499987 は素数
999997: S[999997] - 2 = 499997500001 は素数
999998: S[999998] - 50 = 499998499951 は素数
999999: S[999999] - 31 = 499999499969 は素数
1000000: S[1000000] - 41 = 500000499959 は素数
1000001: S[1000001] - 8 = 500001499993 は素数
1000002: S[1000002] - 64 = 500002499939 は素数
1000003: S[1000003] - 5 = 500003500001 は素数
1000004: S[1000004] - 17 = 500004499993 は素数
1000005: S[1000005] - 26 = 500005499989 は素数
0698132人目の素数さん2020/05/08(金) 15:07:51.16ID:k0ECE/UQ
>>696
素朴には∃p x≦p<x+x^εが正しいし
これは計算機でそれっぽさが確かめられると思うよ
x^1/2オーダーはあくまで積分後の誤差であって、積分前の誤差までそのオーダーを採用するのは粗すぎるはず
(現状の理論的限界としては仕方ない採用かもしれない)
実際はlogxオーダーになってると考えるのが自然

>>697
> あと、 S[n] - k (1 ≦ k ≦ n) が素数になるような最小の k は、 n に比べて小さいように見える

あたりまえ
昨日も書いたけどS[N]付近での素数は2logN個に1個程度
S[1000000]付近での素数は30個に1個くらい
0699132人目の素数さん2020/05/08(金) 15:44:38.54ID:IU3jiVQk
>>698
>あたりまえ
>昨日も書いたけどS[N]付近での素数は2logN個に1個程度
>S[1000000]付近での素数は30個に1個くらい

なるほど
試しに S[n] - k を S[n] + k (1 ≦ k ≦ n) に変えて実験してみたが、特に偏りは見られなかった
ちなみに、 S[n] + 1 が素数になるケースも頻繁に現れるようなので、この形の素数も無数に存在していそう
2次体あたりの理論を使って上手いこと証明できないかな?

【参考】 n = 100万 の近くのデータ
999995: S[999995] + 13 = 499995500023 は素数
999996: S[999996] + 1 = 499996500007 は素数
999997: S[999997] + 16 = 499997500019 は素数
999998: S[999998] + 76 = 499998500077 は素数
999999: S[999999] + 41 = 499999500041 は素数
1000000: S[1000000] + 1 = 500000500001 は素数
1000001: S[1000001] + 2 = 500001500003 は素数
1000002: S[1000002] + 14 = 500002500017 は素数
1000003: S[1000003] + 15 = 500003500021 は素数
1000004: S[1000004] + 17 = 500004500027 は素数
1000005: S[1000005] + 38 = 500005500053 は素数
0700132人目の素数さん2020/05/08(金) 16:18:48.20ID:k2HXTAi9
>>698
少なくともRHが真なら無理です。
RHとは、独立に1/2より小さくならない事も示せてる希ガス。
素数の分布はxの付近に平均1/logx の割合で分布してますが、あくまで平均であり密なとこと粗なとこのムラっけがめちゃめちゃあり、そんな単純にはいきません。
本気でそんな事ないと思うなら解析数論の教科書読むしかない。
0701132人目の素数さん2020/05/08(金) 16:21:49.36ID:7uad0f2n
次のような問題を解くにはどのような本を読めばいいのでしょうか。
https://imgur.com/akT4V52
0702132人目の素数さん2020/05/08(金) 16:37:19.22ID:k0ECE/UQ
>>700
別にRHが真でも上のこととは矛盾しないし、独立に示せているなら出典教えてほしいです
0705132人目の素数さん2020/05/08(金) 17:03:40.40ID:k2HXTAi9
>>704
忘れた、というか挑戦しようと思って教科書だけ買って諦めた。
教科書今手元にないから思い出したらかく。
0706132人目の素数さん2020/05/08(金) 17:03:49.82ID:k0ECE/UQ
ちなみに素数定理で考えると

x=1000000として
xからx+x^(1/2)に素数が存在しない確率は10^(-33)くらい

さらにxが大きいところではもっと悲惨な確率になっていくよ
それを打ち破る誤差が本当にあるのか?
0707132人目の素数さん2020/05/08(金) 17:38:55.59ID:k2HXTAi9
>>706
信じられないなら証明やってみたらいいじゃん?
ちなみに知られている最良の結果は

>>694の極限値≦0.525

のはず。
この手の解析数論はRH流とSieve流の二つの大きな流れがあって0.525はSieve流の解析数論の結果。
RHが正しければ≦0.5でこれ以上は無理だったハズ。
今のところこの方面で結果を出せてるのはSieve流だけなのでSieveだけで=0.5を示して、=0.5⇒RHという方針にチャレンジしてる人もいるみたい。
(黒川先生の本には誰かが逆を証明したと書いてあったけどこの先生の情報あんまりあてにならん)
確か≦7/12は割と初等的なSieveのテクニックだけですむ。
↓このどっちにも載ってる

リーマンゼータ函数と保型波動 (共立講座 21世紀の数学 21) | 本橋 洋一

整数論―解析的整数論入門 (1970年) (近代数学新書) | 三井 孝美
0708132人目の素数さん2020/05/08(金) 17:43:04.40ID:CMUD4s7e
10^(-33) といっても 0 じゃないからな〜。

素数が無限に存在するからには
いつか何処かでそういう素数が存在している
…と思われます、

いつになるかは知りませんが
いつか誰かがその素数を発見した時に
面倒にならぬよう臆病なくらい丁寧に進める必要があるかと。
0710イナ ◆/7jUdUKiSM 2020/05/08(金) 18:29:42.23ID:0lfGvHm4
>>577こっちをちゃんと途中式やってほしい。
>>662
メネラウスの定理より、
Aを起点に、
(AB/BP)(PQ/QR)(RB/BC)(CQ/QR)=1より、
{(AP+PB)/BP}(1/6)(RB/BC)(1/1)=1
(AP/BP+1)(1/6)(RB/BC)(1/1)=1
(AP/BP+1){(BC+CR)/BC}=6
(AP/BP+1)(CR/BC+1)=6──@
Bを起点に、
(BR/RC)(CQ/QA)(AP/PB)=1より、
{(BC+CR)/RC}(1/1)(AP/PB)=1
(BC/CR+1)(AP/PB)=1
BC/CR=PB/AP-1
BC/CR=(PB-AP)/AP
@に代入し、
(AP/BP+1){CR/BC+1}=6
(AP/BP+1){AP/(PB-AP)+1}=6
(AP/BP+1){PB/(PB-AP)}=6
AP+PB=6(PB-AP)
7AP=5PB
∴AP:PB=5:7
0711132人目の素数さん2020/05/09(土) 00:43:50.08ID:dz3/aCOm
>>710
>Aを起点に、
>(AB/BP)(PQ/QR)(RB/BC)(CQ/QR)=1より、
その式自体は成り立つとして、それをメネラウスの定理とは普通言わんやろ

そもそも、>>662質問→>>663メネラウス1回ですぐ終わるシンプルな方法を解答→>>664ありがとうございました
で即座に解決しているだろうに、何をしたいんや…
0712132人目の素数さん2020/05/09(土) 00:45:47.51ID:NE54uC7L
イナって数値解析の人?
とてもじゃないが人間業とは思えんw
0713132人目の素数さん2020/05/09(土) 00:47:08.56ID:NE54uC7L
おまいら工科の人間(田島一郎)が
任意の元に本当に任意に数をぶっこむようになってから
理論がおかしくなってしまったんだ
関数をなめんなよ
0714132人目の素数さん2020/05/09(土) 00:52:15.77ID:NE54uC7L
関数とは
(1) 値域を決める(0の場合が方程式)
(2) (1)に応じて定義域が定まる

おまいらはこの逆をやっている
(1) 定義域は任意であるから任意に数をぶっこむ
(2) 値域が決まる

これは数学ではない
もちろん全射を仮定すればそれでもよいが
全射の仮定もなしに定義域を定義(笑)して値域を決めている
しかし世の中の参考書はほとんどすべて関数を後者で説明している

もう一度言う関数について
任意の定義域を任意に決めてよいなんていう法則はどこにもない
全部やり直せ
0716132人目の素数さん2020/05/09(土) 00:56:02.84ID:NE54uC7L
そもそも全射であることを示せる人間はどれくらいいるのか
D.G.ノースコットは環準同型写像を定義する前に全射を仮定している
この意味は対象と対象との間の射の存在を仮定していることに同義である

おそらく人間が全射を示すことができるのはかなり限定的な場になるだろう
実際写像fの像f(X)=Yを示すことは大変困難であった
全射の証明をきちんとできる人間はいるのだろうか
0717132人目の素数さん2020/05/09(土) 01:03:07.12ID:NE54uC7L
>>715
f:X→Y(写像)

∀x∈X,∃f(x)∈Y

この意味はわかるか?

ここで最初に定義域が「決まっている」とはどういう意味だ?
0718132人目の素数さん2020/05/09(土) 01:06:44.45ID:NE54uC7L
因みに工科の人間の都合で写像も

∀x∈X,∃1y∈Y; f(x)=y

と書かれるような始末だ

このように書かれた本は全部処分するべき
このyはなんだ?
暗黙に全射が仮定されているのでf(X)=Yのf(x)=yなのだ
これは数学ではない
0719132人目の素数さん2020/05/09(土) 01:22:25.35ID:NE54uC7L
おまえらはどうせ全射の仮定もないくせに
関数f(x)のxに数をでたらめに代入してきたからわからないんだ
この間違いは致命的だぞ
たとえば
(1)関数が在る
(2)そして方程式を立てる
(3)その判別式を計算する
(4)方程式の解を求める

これが何故できるのか絶対に説明ができない
今すぐに写像乃至関数がf(x)=yと書いてあるものを捨てろ
そしてたしかに始集合という意味で定義域は先に在るが
中身は任意だ
それだから終集合から認識が始まる
0720132人目の素数さん2020/05/09(土) 02:05:14.40ID:dz3/aCOm
>>NE54uC7L
『Inter-universal geometry と ABC予想 (応援スレ) 45』の方はともかくとして
『数学系YouTuberについて語れ。』スレで00:00から連投しているところを見るに
IDが変わる瞬間を待っていたのだとは思うのですが

スレ違いですので、以下放置で。
0721イナ ◆/7jUdUKiSM 2020/05/09(土) 02:14:55.78ID:N64unEQc
>>710修正。
>>662
メネラウスの定理より、
Aを起点に、
(AB/BP)(PQ/QR)(RB/BC)(CQ/QA)=1より、
{(AP+PB)/BP}(1/6)(RB/BC)(1/1)=1
(AP/BP+1)(1/6)(RB/BC)(1/1)=1
(AP/BP+1){(BC+CR)/BC}=6
(AP/BP+1)(CR/BC+1)=6──@
Bを起点に、
(BR/RC)(CQ/QA)(AP/PB)=1より、
{(BC+CR)/RC}(1/1)(AP/PB)=1
(BC/CR+1)(AP/PB)=1
BC/CR=PB/AP-1
BC/CR=(PB-AP)/AP
@に代入し、
(AP/BP+1){CR/BC+1}=6
(AP/BP+1){AP/(PB-AP)+1}=6
(AP/BP+1){PB/(PB-AP)}=6
AP+PB=6(PB-AP)
7AP=5PB
∴AP:PB=5:7
0722132人目の素数さん2020/05/09(土) 02:26:19.84ID:doYDXY8W
ここ数日、群論と素数の話ばかりで偏りすぎ。

しかも、建設的な話が進むこともなく、
身内で同じ話をグルグル回してるだけ。
0724132人目の素数さん2020/05/09(土) 06:14:05.96ID:SY8s45F/
>>723
ストーカー乙w
0725132人目の素数さん2020/05/09(土) 12:15:11.41ID:5uhdEFoq
キチガイですね
0727132人目の素数さん2020/05/09(土) 15:54:22.48ID:MuNpF+Pe
数学の本に下記の文が載ってました。
x−の読み方は何ですか?またこのように表現することは数学のどの分野で出てきますか?
Aが矢を20回射た。命中した回数をxとすると、命中しなかった回数はx−(xの上に横棒)。

宜しくお願いします。
0728132人目の素数さん2020/05/09(土) 17:03:36.40ID:srY4DWUK
n枚のカードがあり、各カードの裏面には1,2,...,nのうち整数が1つ書かれている。どのカードに書かれている数字も相異なる。これらのカードは中身が見えない袋に入れられている。
以下のようなゲームを行う。

(A)袋からカードを1枚無作為に取り出し、表面の数を記録する。これがk回目(1≦k≦n)の取り出しであるとき、記録した数をa[k]とおく。カードは破棄する。(B)に進む。

(B)もう1回(A)を行うか選択する。
行わない場合、それまでに記録したa[1],...,a[i]の最大値を得点とし、ゲームを終了する。
ただしa[j]=1となるj(1≦j≦i)がある場合、すなわち1が書かれたカードを引いていた場合は、代わりに0を得点とする。

【問】
得点の期待値を最大化したい。どのような基準でゲームを終了するべきか。
0729イナ ◆/7jUdUKiSM 2020/05/09(土) 17:17:06.93ID:N64unEQc
>>721
>>555
正方形Tの一辺の長さをtとすると、
cosθ=t/2
T/8=θ/2π-(t/4)√(1-t^2/4)+(t/4){t/2-√(1-t^2/4)}-(π/4-θ)/2π
T=4θ/π-2t√(1-t^2/4)+2t{t/2-√(1-t^2/4)}-4(π/4-θ)/π
=4θ/π-t√(4-t^2)+t^2-t√(4-t^2)-1+4θ/π
T'=
0730132人目の素数さん2020/05/09(土) 17:34:41.60ID:dz3/aCOm
>>728
カードの裏面には1〜nの整数が1つずつ書かれているにしても、表面にどんな数が書かれているかの情報は無いのか?
記録するのは表面の数なのだから、これでは問題が成立しないと思うのだが。
0732132人目の素数さん2020/05/09(土) 17:41:43.49ID:doYDXY8W
>>728
タイプミスか?

カードの表面は…何だ?
数字が…書かれているの?

これ、カードじゃなくて
ボールに番号が書かれてると考えていいのかな?
0733132人目の素数さん2020/05/09(土) 17:57:15.33ID:dz3/aCOm
>>727
上につける横棒は、普通「バー」と読む。xの上なら「エックスバー」
書きやすく誤読しにくい便利な記号なので様々な場面で用いられるが、例えば
論理や命題での「否定」、集合での「補集合」、確率での「余事象」には統一してこの記号がよく使われるし
統計での「平均・期待値」や複素数での「共役」などに用いられたりもする。

便利で使いやすい分様々な用途で使われるので、どのような意味で用いられているのかは
その場その場できちんと確認する必要があるだろう。

>>727の文面を見る限りでは、命中した回数xに対して(x-)=20-xとする。というだけの意味であろう。
07347272020/05/09(土) 19:29:23.68ID:8Pgo94vR
>>733
丁寧なご回答感謝致します!
誤認識の通りです!ありがとうございます!
0735イナ ◆/7jUdUKiSM 2020/05/09(土) 19:42:54.92ID:N64unEQc
>>729つづき。
>>555
t=2cosθ
cosθ=t/2
θをcosを使って表すとどうなるの? Arccosとかなしで。
θ=cos^(-t/2)?
T=4θ/π-t√(4-t^2)+t^2-t√(4-t^2)-1+4θ/π
=t^2-2t√(4-t^2)-1+8θ/π
=t^2-2t√(4-t^2)-1+8cos(-t/2)/π
T'=2t-√(4-t^2)-(-2t)/√(4-t^2)-8sin(-t/2π)=0
t√(4-t^2)-(4-t^2)+t√(4-t^2)=0
2t√(4-t^2)=4-t^2
4t^2(4-t^2)=16-8t^2+t^4
5t^4-24t^2+16=0
(5t^2-4)(t^2-4)=0
1.5<t<2
t=2√5/5≒0.89442716はおかしい。
図からt≒1.78885432のはず。
t=4/√5であってると思う。
0737132人目の素数さん2020/05/09(土) 23:46:29.66ID:tUatUVy2
Xがハウスドルフ空間の時、任意の異なるn点は必ずし開集合で分離できるのでしょうか
0738 【大凶】 2020/05/10(日) 00:10:30.30ID:Cwd/UqAb
>>735
>>555
正方形Tの一辺の長さtが、
t=4/√5のときCとTの片方に含まれてもう一方に含まれない部分の面積Sは最小になると思うけど、
Tの微分=0の途中式をアークコサインとかアークタンジェントとかコセカントとかなしで示してください。
0739132人目の素数さん2020/05/10(日) 00:53:17.20ID:0ota7gzc
>>737
面倒なので3点x,y,zで考える(n点でも同様)
まずx,yを開集合U,Vで分離する
次にxとz,yとzそれぞれ分離したときのzの開近傍をW_x,W_yとする
W:=W_x∩W_yはzの開近傍であってU,Vと交わらない

もっと簡単な方法があるかもしれない
0740132人目の素数さん2020/05/10(日) 00:57:19.22ID:0ota7gzc
あ、詰めが甘かった
U,Vもそのままじゃなくてzと分離したx,yのそれぞれの開近傍との共通部分をとらないとダメだ
0741132人目の素数さん2020/05/10(日) 00:58:10.31ID:PD03dghb
>>737
i<j に対して開集合Uij Vijを
Uij ∩ Vij = φ、xi∈Uij、xj∈Vij
と選べる。
そこでWjを
Wi= (∩[k] Uik)∩( ∩[l] Vki)
で定めれば
xi∈Wi、Wi ∩ Wj=φ
0742132人目の素数さん2020/05/10(日) 01:21:03.14ID:4M7s5hfJ
>>739
>>741
ありがとうございます
0743132人目の素数さん2020/05/10(日) 03:03:46.38ID:6jFxdgqG
kを実数とする。実数xについての関数f(x)をf(x)=sin(x)/xと定める。

lim[k→0] ∫[0,∞] f(x) *e^(-kx) dx
および
lim[k→0] ∫[0,∞] f(sin(x)) *e^(-kx)dx
を求めよ。
0744132人目の素数さん2020/05/10(日) 04:22:59.82ID:nFzazPfr
p1,p2 を連続する素数として、
√(8*p2+1)-√(8*p1+1)
の最大値を求めよ。

もし、これが2未満であることが証明されたら、>>657の「悪い自然数は無い」が示される事になるはず。
0745132人目の素数さん2020/05/10(日) 07:39:50.63ID:URsi1s0v
確率を教えて欲しいです。
どなたかよろしくお願いいたします。

13個の抽選販売品に対して45名の応募があります。私の家族からは3名応募しております。

この3名のうち一人でも当選する確率は何%でしょうか?
0746132人目の素数さん2020/05/10(日) 09:28:54.04ID:ohXezbgm
>>745
当たる確率は13/45 、逆に外れる確率は32/45
3人とも当たらない確率は(32/45)^3
これを1から引けば、少なくとも1人は当たる確率が出る
1-(32/45)^3
=1-(32768/91125)
=58357/91125(約64%)
0748132人目の素数さん2020/05/10(日) 10:50:18.71ID:SZe05MAw
>>745
1000万回シミュレーションしてみした。

> sim <- function(){
+ lot=rep(c(1,0),c(13,45-13))
+ sum(sample(lot,3))>0
+ }
> mean(replicate(1e7,sim()))
[1] 0.6501402
0749132人目の素数さん2020/05/10(日) 10:58:57.54ID:9cYb8OlZ
半素数って
何か重要な価値、性質ってあるんですか?

2つの素数の積からなる数。
これってただの
「素因数2つからなる合成数」ってことで
めっちゃ普通の数だと思うんですが。

しかも、因数2 を使うことで、
偶数がめちゃくちゃ混ざってますよね…

・素数 … ほとんど奇数のみ ( 中身の比率は 偶数:奇数 = 1 :∞)
・半素数 … 偶数もかなり混ざっている (中身の比率は 偶数:奇数 = 沢山 : めっちゃ沢山)

半素数ってわざわざ名前を与えるほどの価値、性質があるのん?
0750132人目の素数さん2020/05/10(日) 11:11:50.62ID:4M7s5hfJ
数論はあまり分からんけど、素因数はあっても二つまでと言う状況は、普通の数に比べたら取り扱いは簡単そうだ
0751132人目の素数さん2020/05/10(日) 11:17:29.09ID:SZe05MAw
>>746
二項分布じゃなくて超幾何分布だから
> 1 - dhyper(0,13,45-13,3)
[1] 0.65045806906272041
0752132人目の素数さん2020/05/10(日) 11:21:42.70ID:MFXsv5wt
>>749
ある。具体例についてはググれ。Wikipedia でも読めば十分だろう。
一般的に重要で価値のあることを>>749個人が重要だと感じるかどうかはわからないが。
0753132人目の素数さん2020/05/10(日) 11:44:12.06ID:9cYb8OlZ
「この区間に素数が必ず存在する」
っていう予想は価値があるものとして
よく一般の新聞にも取り上げられるけどさ。

「この区間に半素数が必ず存在する」
っていう予想なんて話題にもならないじゃん。

半素数なんて、ゴキブリ並みに
どこにでも存在しているし、たいして価値はねぇよ。

逆に考えろ、ある区間に半素数が存在しないなんて起こりうるの?

「n^2 〜 (n+1)^2 の間に半素数が1つも存在しない
これを満たす n の値の最大値を求めよ…」 とか。
0754132人目の素数さん2020/05/10(日) 11:54:43.12ID:2xD3KZHD
「確率的素数」や「概素数」という言葉があるみたいだな

ほとんどの合成数は満たさないが素数であれば満たす性質を持つもの
0755132人目の素数さん2020/05/10(日) 11:58:44.47ID:4M7s5hfJ
半素数の分布はどのくらいなんだろう
素数定理からすぐ分かるのかな
0756132人目の素数さん2020/05/10(日) 12:14:34.36ID:zIWxqOun
>>743
(上)
I(k)=∫[0,∞] e^(-kx)sin(x)/x dx,

dI/dk = -∫[0,∞] e^(-kx)sin(x)dx
 =[ e^(-kx){k・sin(x)+cos(x)}/(kk+1) ](x=0,∞)]   (*)
 = - 1/(kk+1),

I(k)= arctan(1/k)→ π/2  (k→0)

(*) 高木「解析概論」改訂第三版、岩波書店 (1961)
  第3章、§35.積の積分[例3]p.115

(下)
J(k)=∫[0,∞] e^(-kx)f(sin(x))dx
 =∫[0,2π] e^(-kx)f(sin(x))dx{1 + e^(-2kπ)+ e^(-4kπ)+ ・・・・}
 =∫[0,2π] e^(-kx)f(sin(x))dx /{1 - e^(-2kπ)}
 〜 0.91973041008976/k → ∞  (k→0)
0757132人目の素数さん2020/05/10(日) 12:42:42.87ID:2xD3KZHD
>>755
概素数のwikiにそれっぽいのがあった

n以下の自然数mでその素因子(相異なる必要はない)の数が高々k個であるようなものの数π_k(n)は漸近的に
π_k(n)〜n(loglogn)^(k-1)/(k-1)!logn
となるらしい

k=2のときは
nloglogn/logn

ここからpやp^2タイプの個数(それらはn/lognや2√n/logn程度)を引いた純粋にpq形の半素数の個数も同じフォーミュラでいいと思う
0758132人目の素数さん2020/05/10(日) 13:01:42.71ID:b5TU0NTc
>>744
最大値は存在しない
なぜなら、いくらでも長い素数砂漠が存在するから
0759132人目の素数さん2020/05/10(日) 13:07:36.46ID:zIWxqOun
>>588
>>597
Hadamardの定理は

高木:「解析概論」改訂第三版、岩波書店(1961)
 第2章§26.極大極小 [例2] p.72-73


>>594
スカラー3重積は

同書 第2章§27.接線および曲率 p.75
0760132人目の素数さん2020/05/10(日) 13:09:05.11ID:9cYb8OlZ
素数の自乗の値が
24nの左隣り (24n-1) になることを教えたら
甥っ子から天才扱いされるようになりました!
0761132人目の素数さん2020/05/10(日) 13:13:08.16ID:URsi1s0v
745です。

回答してくださった皆様、ありがとうございました!
0763132人目の素数さん2020/05/10(日) 13:17:07.22ID:URsi1s0v
>>762
ちなみに、13個当選は変わらず4人応募で総応募人数50名の場合はどうなりますか?
0765132人目の素数さん2020/05/10(日) 13:22:53.15ID:2xD3KZHD
>>758
√(x+a)-√x=a/(√(x+a)+√x))に注意
x付近の素数砂漠の長さが√x程度ないと、この値は大きくなれない

>>760
右隣りだろ
あと5以上の素数
0766132人目の素数さん2020/05/10(日) 13:55:56.44ID:URsi1s0v
>>764
60%ですね!?

ありがとうございます( ノ;_ _)ノ
0767132人目の素数さん2020/05/10(日) 13:58:04.72ID:9cYb8OlZ
>>765
訂正サンクス。
+1 だから 右隣だったわ。
今はお酒を飲んでるので、右も左も上も下も前も後ろも分からん。

>>763
一般化して考えるべ。
N本 の抽選をK回行い、D人家族の誰か 1人以上が当選する確率

1 - (D人全員がK回 全部で外す確率)

D人全員がK回の全部を外す確率 p を求めると

p = Π (k=1〜K個までの積) {(N-D)-k +1 / (N-k)+1}

上の一般化した式に値を入れて代数を解く。

50本の抽選で4人家族で 13回 と固定すると

p = Π(k=1〜13までの積) {(50-4) -k +1} / 50-1+1}
 = Π {47-k/51-k}

1 - p = 0. 7132...
よって、 71.31% で当たる
0768132人目の素数さん2020/05/10(日) 14:29:27.47ID:b5TU0NTc
>>765
おっと、恥ずかしいミスをしてしまった
√(8*p2+1) - √(8*p1+1) = 8*(p2 - p1) / (√(8*p2+1) + √(8*p1+1))
だから、例えば、Andrica's conjectureが正しければ、
p2 - p1 < 2√(p1) + 1
となることから、最大値は存在するのが自然なのか
0769イナ ◆/7jUdUKiSM 2020/05/10(日) 14:43:18.86ID:Cwd/UqAb
>>560-561
>>575
8arccos(t/2)を微分して、-8arcsin(t/2)になるとして、t=4/√5になる途中式でどうやってarcsinを消したか書いてほしい。前>>738
0770イナ ◆/7jUdUKiSM 2020/05/10(日) 15:06:14.71ID:Cwd/UqAb
>>769わからんちんどもとっちめちん。arccosは勝手に出てきてなんで消えたんか説明せよ。
‖∩∩‖ □ ‖   \
((-_-)  ‖______‖
(っγυ  。‖╂─╂‖
■`(_)_)ц~ ‖╂─╂‖
\■υυ■_∩∩、\\‖
\\\\⊂(_ _ )`⌒つ)
\\\\\\\`υ、\/|
\\\\\`.,、、、\`/ |
__\\\\彡`-`ミっ/ L
 ̄|\_\\_U,~⌒ヾ /
]| ‖ ̄ ̄ ̄ ̄U~~U / /
__| ‖ □ □ ‖ |/ /
___`‖________‖/_/
 ̄ ̄ ̄ ̄ ̄ ̄ ̄ ̄ ̄‖ /
__________________‖/
0774132人目の素数さん2020/05/10(日) 15:29:14.95ID:2xD3KZHD
Andrica予想の関連から調べたらx以下の最大素数ギャップは(logx)^2(logxではダメっぽい)くらいで考えられてるみたいだな
やはり ID:k2HXTAi9 が主張してた1/2+ε限界はウソくさい気がしてきた
リーマン予想方面からの研究では1/2というのが最良というだけで実際は冪オーダー以下(つまりlog的オーダー)に下げれそう
0775132人目の素数さん2020/05/10(日) 15:32:27.40ID:URsi1s0v
>>767
ありがとうございます!
助かります!
0776イナ ◆/7jUdUKiSM 2020/05/10(日) 17:15:53.77ID:Cwd/UqAb
>>770
>>771
√(4-t^2)の微分が、
(-2t){1/2√(4-t^2)}
=-t/√(4-t^2)じゃないの?
√(4-t^2)=(4-t^2)^(1/2)だから、微分すると、
(1/2)(4-t^2)^(-1/2)・(-2t)
=-2t/2√(4-t^2)
=-t/√(4-t^2)
arccos(t/2)の項とは関係ないじゃないか。
θはtの関数だから勝手に消せないだろう。なんできれいに消してんだ?
0777イナ ◆/7jUdUKiSM 2020/05/10(日) 17:21:53.11ID:Cwd/UqAb
風が涼しくていいなぁ。
 ̄]/\_________前>>776
__/\/ zz..,,、、∩∩/|
 ̄\/ zz..彡`-`ミっ))|
 ̄|\______U,~⌒ヾ、 |_
]| ‖ ̄ ̄ ̄ ̄U~~U / /
__| ‖ □ □ ‖ |/ /
___`‖________‖/_/
 ̄ ̄ ̄ ̄ ̄ ̄ ̄ ̄ ̄‖
□  □  □  ‖ /
__________________‖/
 ̄ ̄ ̄ ̄ ̄ ̄ ̄ ̄ ̄ ̄
1.7888だかって値に文句はないんだよ。
0779132人目の素数さん2020/05/10(日) 17:50:02.52ID:2xD3KZHD
>>776
微積の教科書見れば済む話だけど
dy/dx=1/(dx/dy)を認めれば逆関数の微分は簡単にできるよ
(合成関数の微分公式を使ってもできる)

y=arccosxとおくとx=cosy

よってarccosの微分は
(arccosx)'=y'=dy/dx=1/(dx/dy)=1/(d/dy(cosy))

=1/(-siny)=1/(-√(1-(cosy)^2)=-1/√(1-x^2)

つまりarccosxは√(1-x^2)と相性がよく
実際-x√(1-x^2)+arccosxの微分は項がうまく噛み合って
-2√(1-x^2)だけが残る
0780132人目の素数さん2020/05/10(日) 18:07:01.30ID:9cYb8OlZ
最近、Youtubeで数学の動画を見てるけど
説明の英語が全然聞き慣れない言葉でワロける。

日本語の数学の教科書で学んだから当たり前だけどさ。
昔の日本人はすごい労力をかけて翻訳してくれたよな、
専門用語の単語を(カタカナに置き換えるのではなく)
すべてを漢字による日本語におきかえて教科書にしてくれた。

一方、計算機科学の教科書は…
英語をカタカナ語に置き換えただけの
くそ手抜きである。
0781132人目の素数さん2020/05/10(日) 18:37:31.79ID:h5hDKi6f
>>774
すまん。
1/2*εより下げられない記憶があったんだが自信なくなってきた。
途中で諦めたからな。
何せ結果が出てるのがSieve流であんまり日本では流行ってないんだよ。
三井先生の本は割と詳しくけど、本橋先生の方はリーマン予想との絡みでちょっと紹介されてるだけやし。
で洋書買って独習で0.525+εに挑戦したけど断念した。
教科書レベルでは載ってないので結局元論文読むしかなく、そこからさらに何本も重たい論文引いてて専門外なのでこらアカンと。
まぁ難しい。
黒川先生は≦0.5+εからRHがでると言ってるぐらいだからおそらく≦0.5+ε示せるだけでおそらくフィールズ賞級だと思う。
0782132人目の素数さん2020/05/10(日) 20:12:04.13ID:nFzazPfr
>>744 について、一応100万組の素数について、チェックした結果を報告しておきます。

仮最大値を0.1にセットします。√(8*p2+1)-√(8*p1+1) を計算し、仮最大値より大きければ更新します。
これを素数を変化させながら繰り返します。
小さい方からスタートすると、全く更新されないので、100万番目 → 1番目 という順に出力させました。

{p1の素数番号,p1,p2,√(8*p2+1)-√(8*p1+1)}

{325852, 4652353, 4652507, 0.100971}
{165326, 2238823, 2238931, 0.102076}
{157680, 2127163, 2127269, 0.102782}
{149689, 2010733, 2010881, 0.147602}
{118505, 1561919, 1562051, 0.149366}
{104071, 1357201, 1357333, 0.160235}
{49414, 604073, 604171, 0.178311}
0783132人目の素数さん2020/05/10(日) 20:12:30.38ID:nFzazPfr
{40933, 492113, 492227, 0.229806}
{31545, 370261, 370373, 0.260283}
{14357, 155921, 156007, 0.307965}
{5949, 58831, 58889, 0.33809}
{4612, 44293, 44351, 0.389612}
{4522, 43331, 43391, 0.407489}
{3644, 34061, 34123, 0.474875}
{3385, 31397, 31469, 0.57432}
{738, 5591, 5623, 0.604359}
{650, 4831, 4861, 0.609452}
{590, 4297, 4327, 0.646087}
{462, 3271, 3299, 0.690873}
{429, 2971, 2999, 0.724758}
{367, 2477, 2503, 0.73685}
{263, 1669, 1693, 0.827807}
{217, 1327, 1361, 1.31154}
{30, 113, 127, 1.80722}
{4, 7, 11, 1.88415}
0784イナ ◆/7jUdUKiSM 2020/05/10(日) 20:24:30.67ID:Cwd/UqAb
>>777考え中。
>>779もうひと越え。
t=2cosθ
t/2=cosθ
θ=arccos(t/2)
θ'=-arcsin(t/2)
t'=-2sinθ
S'=2t-2√(4-t^2)-2t(-2t)/√(4-t^2)-8(1/2)arcsin(t/2)
=2t-2√(4-t^2)+4t^2/√(4-t^2)-8(1/2)arcsin(t/2)
=2t-2√(4-t^2)+4t^2/√(4-t^2)-4arcsin(t/2)
───[境界線]────
S'=2t-4√(4-t^2)
0785132人目の素数さん2020/05/10(日) 21:57:01.47ID:zIWxqOun
>>765
仰るとおり。
 x = n! -n 〜 n! -2
 x = n! +2 〜 n! +n
の場合は長さ n-1 だから √x よりずっと短い。

長さ〜√x の素数砂漠は極めて稀で可能性はほぼ0だけど、
無いとは言えない。
0786132人目の素数さん2020/05/10(日) 22:19:00.21ID:2xD3KZHD
>>781
いずれにしても自粛が開けたら読むつもり
かなり興味湧いたので

>>782
x付近での素数ギャップはxに対していかなるxの冪根よりも小さくなっていくはず(素朴な統計学的には)だから
√xで測ると桁数を上げるほど高値更新の確率は急速に小さくなるし、値自体もゼロに向かうはず

>>784
微分の「'」は何の微分で考えてるの?
θの微分とtの微分がごっちゃになってないか
0788132人目の素数さん2020/05/10(日) 22:51:41.45ID:b5TU0NTc
>>782
>>783
興味深い
100,000,000(=1億)以下の素数について計算してみたが(π(100,000,000) = 5,761,455)、
同様の結果だった

√(8*p2 + 1) - √(8*p1 + 1) < 2 なら>>657の「悪い自然数は無い」
というのはどのようにして導かれる式なんですか?

ちなみに、仮最大値を 0.05 にセットしたらこうなった
{1319945, 20831323, 20831533, 0.06506909556992468}
{733588, 11113933, 11114087, 0.06532814471393067}
{597311, 8917523, 8917663, 0.06630086688710435}
{566214, 8421251, 8421403, 0.07407442976364109}
{491237, 7230331, 7230479, 0.0778386683196004}
{415069, 6034247, 6034393, 0.08405308937472}
{325852, 4652353, 4652507, 0.10097079811112053}
(以下>>782-783と同じ)
0789132人目の素数さん2020/05/10(日) 23:59:29.13ID:2xD3KZHD
>>787
係数の1/2が面倒だから正方形の辺を2xとする
その時のハミ出した円の4つの耳を考える
耳は円を中心からxの距離にある直線で切ったものだから
耳のさらに半分はちょうど半径1の円の方程式√(1-x^2)を(x〜1)で積分した形になってる、この面積をg(x)とおく
√(1-x^2)の原始関数をF(x)としたときg(x)=F(1)-F(x)である

問題の面積S(x)は
S(x)=正方形-(円-4耳)+4耳=正方形+8耳-円
だから
S(x)=4x^2+16g(x)-π

よって
S'(x)=8x+16g'(x)=8x+16(0-F'(x))=8x-16F'(x)=8x-16√(1-x^2)

これでどうよ!
ちなみにここで出てきたg(x)が具体的に書くと面倒な
(-x√(1-x^2)+arccosx)/2となってる
0790132人目の素数さん2020/05/11(月) 00:20:57.72ID:S5JMkReH
>>788
一億までの検証ありがとうございます。
>>657 の問題は、「連続する三角数の間には必ず素数が存在するか? もし、N-1番目の三角数とN番目の三角数の間に
一つも素数が無いなら、Nを「悪い自然数」と呼ぶ」というようなものです。

「悪い自然数Nが存在する」なら、次を満たす連続する素数p1,p2(p1<p2)があることになります。
・p1 は N-2番目の三角数とN-1番目の三角数の間にあり、
・p2 は N番目の三角数とN+1番目の三角数の間にある

p1についての条件式は、
 (N-2)(N-1)/2 < p1 < (N-1)N/2 → 1+√(8*p1+1) < 2*N < 3+√(8*p1+1)
と変形できます。2*Nを挟む、上限と下限の差は、2であり、p1を与えると、自然数Nが、唯一、かつ、必ず、定まります。
p2についても同様で
 N(N+1)/2 < p2 < (N+1)(N+2)/2 → -3+√(8*p2+1) < 2*N < -1+√(8*p2+1)
連立させると、
1+√(8*p1+1) < 2*N < -1+√(8*p2+1)
が得られます。これが、「悪い自然数Nが存在する」ための、本来の条件式です。中辺を取り除いて、不等号を逆転させた
1+√(8*p1+1) > -1+√(8*p2+1)
が、常に成立しているなら、「悪い自然数Nが存在する」ための条件式が常に成り立たないことになり、
「悪い自然数は存在しない」と言うことになります。
0791132人目の素数さん2020/05/11(月) 00:30:57.03ID:aBpWM8d5
〔ラプラスの展開公式〕
Aをn次の正方行列、a, b, ・・・・ をAの列ヴェクトルとする。
列ヴェクトルaの要素に対する余因子がなす行ヴェクトルを a~ とすれば
 a・a~ = det(A),
 b・a~ = c・a~ = ・・・・ = 0,

* 3次のときは a~ = b × c (外積)と書ける。

>>759 の概略
|a|, b, c, ・・・ を固定して|det(A)|を最大にする。
a~ も固定される。 上の第1式とコーシーから
 |det(A)|≦ |a| |a~|,
等号成立条件 a // a~ と第2式から
 b・a = c・a = ・・・・ = 0,
∴ a が b, c,・・・ と垂直のとき。  (・・・・垂直成分 a'[k] = a[k])

他の列ヴェクトル b, c, ・・・ についても同様だから、
a,b,c,・・・ は(内積の意味で)直交系をなす。
Gram{a,b,c,・・・}は対角行列で、成分は |a|^2, |b|^2, ・・・・
∴ det(A)^2 = det(A・tA)≦ det(Gram{a,b,c,・・・})=(|a| |b|・・・・)^2,
0792132人目の素数さん2020/05/11(月) 01:06:01.96ID:m1+vBmdx
>>790
2点わからないところがあります

>・p1 は N-2番目の三角数とN-1番目の三角数の間にあり、
>・p2 は N番目の三角数とN+1番目の三角数の間にある

p1をこのように仮定するとき、「悪い自然数Nが存在する」なら、
p2 はN番目の三角数より大きいとは言えますが、N+1番目の三角数より小さいと言えるのはなぜですか?

>p1についての条件式は、
> (N-2)(N-1)/2 < p1 < (N-1)N/2 → 1+√(8*p1+1) < 2*N < 3+√(8*p1+1)
>と変形できます。

この式変形がフォローできません
分母を払って展開すると、
N^2 - 3*N + 2 < 2*p1 < N^2 - N
となりますが、この後はどうすればいいですか?
0793132人目の素数さん2020/05/11(月) 01:46:01.44ID:m1+vBmdx
>>790
>>792
すみません、式変形は自己解決しました
要するに、
p1 < (N-1)N/2 ⇒ 1+√(8*p1+1) < 2*N
かつ、
N(N+1)/2 < p2 ⇒ 2*N < -1+√(8*p2+1)
から、
1+√(8*p1+1) < 2*N < -1+√(8*p2+1)
が従うが、この中辺を取り除いた不等式
1+√(8*p1+1) < -1+√(8*p2+1)
は成り立たないことが予想される。すなわち、もし常に上の不等号を逆にした不等式
1+√(8*p1+1) > -1+√(8*p2+1) ⇔ √(8*p2+1)-√(8*p1+1) < 2
が成り立つならば、「悪い自然数」 N は存在しない
ということですね
0794132人目の素数さん2020/05/11(月) 02:17:09.69ID:m1+vBmdx
なるほど、全く同様な考察で、
Andrica's conjecture: √(p2) - √(p1) < 1
から
Legendre's conjecture: n^2 < p < (n+1)^2 (平方数の間には常に素数が存在する)
が従うことがわかる

実際、もし平方数の間に素数がない区間 ((n+1)^2, (n+2)^2) が存在すれば、
ある連続した素数 p1 < p2 が存在して、
p1 < (n+1)^2 < (n+2)^2 < p2 ⇒ -1 + √(p1) < n < -2 + √(p2)
より、 -1 + √(p1) < -2 + √(p2) となるが、
Andrica's conjectureはこの逆の不等式が常に成り立つと主張しているわけだな
0795132人目の素数さん2020/05/11(月) 02:17:45.07ID:P3aDTzfm
必要なのはその不等式だけでいいから
p_iがS(N)-kのチェック区間に入ってさえいなければよくて
他のどこの三角数の間にいるかは重要じゃない
0796132人目の素数さん2020/05/11(月) 02:45:14.19ID:m1+vBmdx
>>795
そういうことですね
連続した素数 p1 < p2 に対して、
p1 < (N-1)N/2 < N(N+1)/2 < p2
となる N の存在と、常に
√(8*p2+1)-√(8*p1+1) < 2
となることは両立しないと
0797132人目の素数さん2020/05/11(月) 13:31:41.58ID:S5JMkReH
なるほど、>>796 のように表現すればすっきりしますね。勉強になりました。


ちなみに、>>657 の問題は、次のように言い換えられると思います。

2以上の任意の自然数nに対し、次を満たす素数pは存在する か?

| ((n^2-1)/(2p)) -1 | < 1/n
0799132人目の素数さん2020/05/11(月) 17:16:06.43ID:V2U0Ryfa
Aの逆行列Bが存在しないとは幾何的にはどういう意味なのか教えて下さい。
3次以下の場合、行列Aによって点Pが移った先を点Qとすると、点Qを点Pに移す行列は存在しないのでしょうか。あるいはBでない行列Cが存在して、QをPに移すのでしょうか。
0800132人目の素数さん2020/05/11(月) 17:30:21.81ID:iwedaXW5
服部「位相幾何学」を読んでいて以下の証明がわからないのでご教示ください
CW複体Xとその部分複体Aについて、Aを含むXの開集合Vで、AがVの変位レトラクトであるようなものが存在する

(Aが閉集合のときそのようなVがとれることを(X,A)がカラー空間対であると定義すると、
部分複体は閉集合なので(X,A)がカラー付き空間対であることを示してほしいです)
使えそうな定理としては
1.(X,A)がカラー付き空間対のときそれをf:A→Yで張り合わせた接着空間X∪Yについて
(X∪Y,Y)はカラー付き空間対であること
2.CW複体の対(X,A)についてX^q∪AはX^(q-1)∪AにX-Aに含まれるqセルを張り合わせたものに等しい
ただしX^qはXのq切片とする
あたりです、よろしくお願いします
0801132人目の素数さん2020/05/11(月) 21:05:38.77ID:LsbAQCSo
>>800
でけた。
Xのセル数についての帰納法
Xの極大セルSをとり
T=A∩S、Y=X\intS、B=A\intT
とおく。
BのYでの近傍Vと
G:V×I→Vを
G(v,0)=b、G(v,1)∈B、G(b,i)=b
ととる。
Aの開近傍UとGの拡張F:U×I→Uを定める。
S⊂Aである時にはU=V∪S、F(u,i)=uと定める。
S⊂Aでないとする。
O∈intSを選びU=V∪S\{O}、
F(y,i)=y (i<1/2)、
F((1-λ)O+λθ,i)
=(1-2i)((1-λ)O+λθ)+2iθ (i<1/2,θ∈∂S,λ∈(0,1))、
F(u,i)=F(F(u,1/2),2i-1) (i≧1/2)
と定めれば求める条件を満たす。
0803132人目の素数さん2020/05/11(月) 22:25:17.04ID:TMlZF2rC
係数 a,b, ... c など は複素数

xは複素数

このとき、複素数の 多項式 f(x) = ax^n + bx^(n-1) + .... + c

がf(x) = 0 を満たすxを持つことを証明せよ。
(そのときの x がどんな値かは求めなくてもよい)
0804132人目の素数さん2020/05/11(月) 22:26:36.12ID:0tRtKTKI
>>799
点Qを点Pに移す行列は存在する。行列Bは存在しない。Bが存在しないから「Bでない」という条件を満たすことがそもそも不可能なため行列Cは存在しない。
そもそも空間内の特定の2点P,Qについて、点Qを点Pに移す1次変換は無数に存在する。もしAの逆行列が存在したとしても、点Qを点Pに移す変換はAの逆行列以外にもいくらでもある。

Aの逆行列が存在するなら、QをPに戻す行列Cが「点Pに依存せず、行列Aにだけ依存して」定まる。ということ。
Aの逆行列が存在しない場合でも、点Pに依存していいのなら点Pの値ごとに個別の行列Cを与えてQをPに戻すことはできる。一意ではないが。
要するに、逆行列が存在しないということは単射でないのだから、逆に戻そうとしたらそれは写像ですらなくなるだろう。

例えば、Aの逆行列をA~、点Pの移った先をA(P)と記述するなど、どれが何に依存して定まっているのかを明確に記述することは大事。これを怠るからごちゃごちゃになる。

Aの逆行列が存在しないとき
「任意の点Pに対して P=B(A(P))」を満たす行列Bは存在しないが
任意の点Pに対して「P=B(A(P))」を満たす行列Bは存在する
ということ。
0805132人目の素数さん2020/05/11(月) 22:56:59.10ID:vF2hgW3A
>>803
Cは代数的閉体なので自明
いじょ
0806132人目の素数さん2020/05/11(月) 23:06:06.02ID:0tRtKTKI
>>803
代数学の基本定理を証明せよと来たか……

重要定理なんだからググればいくらでも解答は得られるだろうけど、
きちんと本を読むなり講義を受けるなりしてちゃんと学んだほうがええと思うで。
0807132人目の素数さん2020/05/11(月) 23:47:48.09ID:TMlZF2rC
f(0) を計算した結果を 点O
f(1000000) を計算した結果を点P

Oを中心に半径1の小さい円を描く
Oを中心に、かつ、Pを通るような半径の大きい円を描く

f(0) を中心とする円を小さい円から大きい円へ広げていくと、
必ず原点 (0+0 i) を通るような大きさの円が描ける。
よって f(x) = 0 を満たすような 複素数x は必ず存在する

だいたい、そんな漢字?
0808132人目の素数さん2020/05/11(月) 23:50:48.05ID:TMlZF2rC
幾何学的に表現すると
こんな感じでええんか

点O を中心にして半径の大きさをちょっとずつ伸ばして
円を書いていくと、 原点を通る円を必ず1つは描ける。

原点を通る円が描けるってことは、
f(x) = 0 を満たすxが存在するってことで。
0809132人目の素数さん2020/05/12(火) 00:31:16.98ID:kUbV8TN4
a,tを実数、nは自然数の定数とする。xについての方程式
(x-1)(x-2)...(x-n)+a=0
が相異なるn個の実数解x_1,x_2,...,x_nをもち、それらはi=1,2,...nに対してi-t≦x_i<iを満たすと仮定する。
このときtを動かしt→+0とすれば、それに伴いa→0となる。ではa/(t^p)が0でない定数に収束するような実数pを求めよ。
0810132人目の素数さん2020/05/12(火) 00:54:12.31ID:aKb58kvU
K/Cを有限次ガロア拡大とする。
G=Gal(K/R)の2-sylow群をHとしてHの固定体をLとする。
この時L/Rは次数が奇数の代数拡大であるが、Rは次数が奇数の真の代数拡大を持たないからL=Rであり、H=Gである。
特にGは冪零群である.
群の降鎖列G=G0⊃G1⊃‥を[Gi:G(i+1)]=2ととる。
対応する体の昇鎖列をR=K0⊂K1⊂‥Kn=Kをとる。
K1はRの二次拡大だからCである。
n≧2ならK2はCの真の二次拡大であり矛盾する。
よってn=1でありK=Cである。
0811132人目の素数さん2020/05/12(火) 00:58:24.99ID:H35eBoB9
>>801
ありがとうございます
同様に考えてセルが無限個あっても最高次のセルがある時は示せそうですね

問題はS^∞のような無限次元のCW複体の時にどうするかですが…
検討もつかないので分かる人いたら引き続きよろしくお願いします
0814132人目の素数さん2020/05/12(火) 01:58:02.89ID:g6bEKpZn
>>811
一緒じゃないの?
A=X0⊂ 1⊂‥
X=∪ Xi,X(i+1)=Xi∪Di (Diはある次元のDisk)
としてAのXiねおける開近傍UiとFi:Ui×[0,1]→Uiを
・Fi(u,0)=u
・Fi(u,1)∈U(i-1)
・Fi(a,t)=a
・U(i+1)∩Xi = Ui
>>801の方法に類するようにとる。
ただしGからFを作るときは
F((1-λ)O+λθ,t)=(1-t)((1-λ)O+λG(θ,t))+tG(θ,t)
としてFがGの拡張になるように(F(i+1)がFiの拡張になるように)作れば良い。
0815132人目の素数さん2020/05/12(火) 08:37:43.77ID:gmSQkuCI
>>803
f(x)がn次式のとき、複素平面のn重被覆となるから
f(x)=0は重複度を含めて、n個の解を持つ
0816132人目の素数さん2020/05/12(火) 08:40:45.64ID:H35eBoB9
>>814
>>801だと極大セルSを取ったことが重要で
極大でないセルを取り除いたものについては同様に構成するのは難しそうだと思ったのですが
(S^2の各次元2個ずつのセル複体を考えると、1セルを取り除いたところでの0セルの近傍をとっても
S^2全体での近傍にはうまく拡張できないですよね)

有限次元でAの近傍が拡張としてどんどん作っていけるので、その帰納極限として無限次元の近傍も構成できる的な感じでしょうか?
0817132人目の素数さん2020/05/12(火) 11:30:03.48ID:1foaz9rw
助けてください。測度論の完備性についての質問です。L^1(X)の点列{f_n}が与えられた時、
g_l(x)=|f_n_1(x)|+Σ(k=1〜l)|f_n_k+1(x)-f_n_k(x)| ∈ L^1(X)となるのは何故でしょうか?教えてください。
0818132人目の素数さん2020/05/12(火) 11:31:57.13ID:/CoSoSV5
>>816
>>800の2.を使えばいいでしょ?
コレからCW複体の列X0⊂X1⊂‥を
・X0=A、∪Xi=X
・X(i+1)はXiに単体を貼り付けたもの
となるように構成できる。
だから示すべきは
補題
(X,A)がCW複体対、Y=X∪Dをある∂D^q→Xで単体DをXに接着した空間とする。
UがAのXでの開近傍、F:U×I→UがAのUにおけるUの強変位レトラクトを与えるとき、 AのYにおける開近傍VとG:V×I→VでAのVにおけるVの強変位レトラクトを与えるものが取れる。
∵O∈intDを任意に選び
V=U∪{(1-λ)O+λθ | λ∈(0,1)、θ∈U}、
G((1-λ)O+λθ ,t)=(1-t)((1-λ)O+λF(θ,t))+tF(θ,t)
後は↑コレで構成した開近傍とレトラクトの帰納的極限を取ればよい。
0819132人目の素数さん2020/05/12(火) 11:33:39.40ID:ASeuP31U
>>805-806 >>815
確認だけど、幾何学的な説明としては
>>803 のイメージで…あっているよね?

これって証明としてアリだよね、論理に不備はないよね?
ただの整合性を確認するためのデモンストレーションじゃなくて。
0820132人目の素数さん2020/05/12(火) 11:35:53.91ID:ASeuP31U
レベルが低くてすいません…

高校生への複素数の多項式の説明として
もう1回詳しく述べます。

問い.

複素数の 多項式 f(x) = ax^n + bx^(n-1) + .... + c
がある。
f, x, 係数a, b, c... は全て複素数である。

この多項式 f(x) が f(x) = 0 を満たすxを1つ以上持つことを証明せよ。
(そのときの x がどんな値かは求めなくてもよい)

回答.
・任意の複素数 x = a + b i と
・複素数 f(x) = A + B i

変数がa,b, A,B, と4つあるので
説明のために複素平面を左と右に1枚ずつ用意する。

1. xの値を適当に決めて、左の複素平面へ点を打つ。
同時にf(x) を計算して右の複素平面に点 "x→ f(x)" を1つずつ打つ。

この作業を x=0 から x = 非常に大きい値 まで繰り返す。
例として、 f(0) 〜 f( 100 + 100i ) までに対して
右側の複素平面に x → f(x) の点を計算して書き込んでいく。

xが0 の時の f(x) である f(0) = z とする。
また、これに対応する点 (右側の複素平面上の点) を点Zとする。
右側の複素平面で
点Zを中心にし、コンパスで非常にさな円を描く (例として半径1の円)。
そこから半径を少しずつ大きくしながら、大量に円を描いていく。

そのうちに、原点 (0+0 i) を通る円が現れる。

よって、 f(x) = 0 を満たす 複素数 x が (左の複素平面) に必ず存在している。

(x の値は不明のままだが f(x) = 0 を満たす複素数が
必ず存在するのが直感的にあると分かる)
0821132人目の素数さん2020/05/12(火) 11:42:29.26ID:ASeuP31U
多項式 f(x) = ax^n + bx^(n-1) + .... + c

がf(x) = 0 を満たすxを持つことを証明せよ。

これ、複素数の多項式じゃなく、
実数の多項式の時は条件付きでしか成立しないんだよね。
(高1 の教科書レベルだけど)
xに乗る次数nが 「奇数」 であるという条件が必須。

複素数にしたら条件なしで成立するって不思議。



実数の世界の制約が厳しすぎるのか…、
複素数アリの世界は制約がゆるすぎるのか…。

複素数は何でもありやな。 ( '〜')
0822132人目の素数さん2020/05/12(火) 13:05:30.63ID:3mMO1kdL
>>820>>803
全体的に意味のないことをしているとしか思えないが、とくに
>そのうちに、原点 (0+0 i) を通る円が現れる。
>よって、 f(x) = 0 を満たす 複素数 x が (左の複素平面) に必ず存在している。
ここの論理の飛躍が意味不明すぎる。

>>805-806 あたりで十分に妥当な返答が得られているんだから、きちんと勉強しようよ。
ろくにわかっていない人物から説明されるであろう高校生とやらが不憫でならない。

そもそも、問いの時点で「fが複素数」とかいう意味不明な条件を付けくわえていたり
証明をしようとしているのに「直感的にあると分かる」とか言い始めたり
人に数学を説明できるだけの素養が身についているとは思えない。
0823132人目の素数さん2020/05/12(火) 13:20:10.61ID:ASeuP31U
>>822
すいません、大学の数学は知りません。
高校生までの知識で高校生向けに
説明しようとしたらこうなりました。

反省します、書き込みを控えます。
0824132人目の素数さん2020/05/12(火) 13:21:23.25ID:HlS2vCHp
高校で教えるならリウヴィルの定理を使った証明を紹介して、複素解析へのモチベーションにすればいい
三角関数 sin, cos は実数の範囲では有界だが、複素数に拡張すると有界にならないこともついでに紹介しておけばいい
0826132人目の素数さん2020/05/12(火) 13:33:33.95ID:HlS2vCHp
>>825
【余談】と断った上でプリントにのせて配布する形なら良いでしょう
「テストや入試とは関係ないけど、もし興味があれば読んでみてね」って言っておけばいいよ
0827132人目の素数さん2020/05/12(火) 14:07:49.62ID:ASeuP31U
>>822
すいません、
証明じゃなくて
整合性を確認する視覚的なデモンストレーションです。

ごもっともです、
「f が 複素数」 というのは訂正します
正確には、 …
関数f は複素数xを扱い、f(x)も複素数となり、
複素平面上で表される点である…と強調したかった

直感的という言葉も訂正します → 視覚的に と置き換えてください
0828132人目の素数さん2020/05/12(火) 14:19:24.66ID:HlS2vCHp
「整合性を確認する視覚的なデモンストレーション」としてもイマイチだなあ
それって結局、 n 次の多項式関数 f(x) は任意の複素数値 α をとるってことでしょ?
要するに、任意の複素数 α に対して、 n 次の多項式関数
g(x) = f(x) - α
が g(x) = 0 満たす x を1つ以上持つということを暗に使っているわけで
0829132人目の素数さん2020/05/12(火) 14:30:27.44ID:ASeuP31U
>>822

そこの論理は飛躍していません、そのままです。

やっていることは、 実数x で
多項式 f(x) = x^n + x^n-1 + ...
が 「nが奇数の場合」 f(x) = 0 をみたすxを1つ以上持つことを証明するのと同じです。

x に 0からとても離れた負の値、 とても離れた正の値の2つを代入して、
f(負の値) と f(正の値) 、この2つの点を結ぶ線を描くと
どこかで x軸を横切らざるをえない。
よって、 f(x) = 0 となるような実数x が存在する。

これを複素平面上で、
中心Zで とても小さい円 → とても大きい円でやっているだけッス。
0830132人目の素数さん2020/05/12(火) 14:38:46.80ID:ASeuP31U
あかん、ワイがアホすぎるのか。

何が分からないのかが分からない、
いちおう、大卒なのに (´・ω・`)
0831132人目の素数さん2020/05/12(火) 14:43:43.81ID:3mMO1kdL
そのデモンストレーションで整合性が確認できるとは思えない。

「代数学の基本定理の証明方法のうちの1つを取り上げて、その証明方法の整合性を視覚的なデモンストレーションで確認する」
のであれば、まず先にその証明を述べる必要があるだろう。

「代数学の基本定理そのものの整合性を確認する」のであれば、代数的な方法を用いるべきであろう。

代数学の定理を証明するために解析的な手段を用いることの整合性を確認するのか?全然そのようには見えないが。
0832132人目の素数さん2020/05/12(火) 14:49:40.41ID:bNOpG3bC
>>829
なんかの研究会でSEGの古川がDeriveで視覚的にみせていた
それをみて一松先生が頷いていた
0833132人目の素数さん2020/05/12(火) 17:10:29.09ID:gmSQkuCI
f(0)から、ε>0をとって|z|=εとなるf(ε)の像を描くと面白いかもしれない
この程度ならエクセルで図も書ける

「証明方法の整合性を視覚的なデモンストレーションで確認する」といわずに、
「ナイーブな方法で、f(z)=0となるzを見つけてみる」といえばいい
0834132人目の素数さん2020/05/12(火) 17:11:13.94ID:gmSQkuCI
訂正

f(0)から、ε>0をとって|z|=εとなるf(z)の像を描くと面白いかもしれない
この程度ならエクセルで図も書ける

「証明方法の整合性を視覚的なデモンストレーションで確認する」といわずに、
「ナイーブな方法で、f(z)=0となるzを見つけてみる」といえばいい
0835132人目の素数さん2020/05/12(火) 17:48:51.05ID:HlS2vCHp
>>829
そのような方法で「確認」するのは無理がある
多項式関数が零点を持つからたまたま上手くいくだけ
例えば、複素指数関数 exp は整関数だが、零点を持たない
しかし、
exp(x+iy) = e^x (cos(y) + i sin(y))
だから、 exp は 0 以外の任意の複素数値をとることができる
もし、 exp に対して同様の方法を使うと、
exp(z) = 0 となる複素数 z が存在することが「確認」できてしまうのでは?
0836132人目の素数さん2020/05/12(火) 19:58:50.70ID:eBJaVlVk
部屋で冷蔵庫開けっぱなしにすると、部屋の温度上がるってホントなんですか?
0837132人目の素数さん2020/05/12(火) 20:17:13.40ID:HlS2vCHp
>>831
ところで、代数学の基本定理の「純粋に」代数的な証明ってある?
俺が知っている「代数的」な証明は、
「奇数次実数係数の多項式は少なくとも1つ実数の根を持つ」
を使っているんだよね
で、↑の証明には中間値の定理が必要だから、やっぱり解析的な手段が使われているんだよね
0838132人目の素数さん2020/05/12(火) 20:31:06.07ID:gdd+7JW+
何を持って代数的と思うかだね
実数の定義がそもそも代数的じゃないように思えるし、そこが原因な気もする
0839132人目の素数さん2020/05/12(火) 20:39:52.06ID:HlS2vCHp
>>838
確かに「実数係数」の時点で連続性の概念が必要だな

じゃあ少し問題を変更して、
「有理数係数の多項式は少なくとも1つ根を持つ」
なら、純粋に代数的に証明できる?
0840132人目の素数さん2020/05/12(火) 20:44:13.11ID:LHLoA2rw
QからRの構成って代数的にできんのかな
そこで解析的な手法を使うしかないなら難しいかも
俺が知ってるのはQのコーシー列で構成するか、デデキントの截斷の二つだけど、いずれも解析的手法 or Qの順序構造を使用していて、代数的には構成できてない
0841132人目の素数さん2020/05/12(火) 20:48:21.26ID:HlS2vCHp
>>839
訂正
「有理数係数の多項式は少なくとも1つ根を持つ」

「有理数係数の多項式は少なくとも1つ複素数の根を持つ」

代数的閉包に根を持つのは自明だからね
あるいは、
「有理数体Qの代数的閉包(代数的数体)は複素数体Cに含まれる」ことを純粋に代数的に証明せよ
と言ってもいいかな
0842132人目の素数さん2020/05/12(火) 20:50:50.72ID:gdd+7JW+
>>839
有理数の範囲では解持たないことあるから微妙な問いのような
既約多項式の解として代数的数を定義するならばそれは定義だから自明なはず

むしろ代数的数たちが体をなすことを基本定理というべきな気もしてきた
0843132人目の素数さん2020/05/12(火) 20:52:25.47ID:gdd+7JW+
>>841
あ、訂正きてたか
複素数も実数から作られるから解析的範疇に入るのでは?
0844132人目の素数さん2020/05/12(火) 20:53:16.90ID:H35eBoB9
>>818
拡張として構成しているのでその帰納的極限が取れるわけですね
やっと理解できました
何回も説明していただきありがとうございます、助かりました
0845132人目の素数さん2020/05/12(火) 21:48:52.88ID:vIdFfIqo
円Cに内接する△ABCと、弧ABの中点L、弧BCの中点M、弧CAの中点Nを考える。
△ABC∽△LMNとなる条件を述べ、また特に合同となる場合の△ABCの形状を述べよ。
0846132人目の素数さん2020/05/12(火) 21:49:19.81ID:bKWeqWjB
工房です。

https://imgur.com/a/mdhyxq8

この問題の回答の仕方を教えてください!
0847132人目の素数さん2020/05/12(火) 21:50:36.13ID:Qx4sczK/
がんばってけいさんすればいいよ!
0848132人目の素数さん2020/05/12(火) 22:19:21.85ID:3mMO1kdL
>>845 弧AL=弧LB=s、弧BM=弧MC=t、弧CN=弧NA=u とする。

△ABC∽△LMN
⇔∠A=∠L かつ ∠B=∠M かつ ∠C=∠N
⇔弧BC=弧MN かつ 弧AC=弧LN かつ 弧AB=弧LM
⇔2t=t+u かつ 2u=s+u かつ 2s=s+t
⇔t=u かつ u=s かつ s=t
⇔s=t=u

したがって、△ABC∽△LMNと△ABC≡△LMNと△ABCが正三角形であることはすべて同値である。
0849132人目の素数さん2020/05/12(火) 22:39:24.28ID:+0VznqQm
複素関数です、よろしくお願いします
https://i.imgur.com/WdyhYqP.jpg
0851132人目の素数さん2020/05/12(火) 23:12:22.97ID:3mMO1kdL
>>849
等比数列の和の公式から、公比 e^(2πi/M)≠1 のときは
Σ[n=0~(M-1)]e^(2πin/M)=(e^(2πi)-1)/(e^(2πi/M)-1)
e^(2πi)=1 だから分子=0で終わり
公比1のときはM=1に限るから>>850の通り
0852132人目の素数さん2020/05/12(火) 23:19:58.31ID:8Qxout3p
>>849
a=e^(2πi/M)とする。M>1のとき
1+a+a^2+...+a^(M-1)=(1-a^M)/(1-a)であるが
a^M=1なので、求める式は0
0853132人目の素数さん2020/05/12(火) 23:24:54.94ID:HlS2vCHp
>>849
じゃあ M > 1 のときに示そうか
M で割る意味は全く分からんが

e^(2πin/M) (n = 0, 1, … , M-1) は方程式
x^M - 1 = 0
の相異なる M 個の解になっているから、 M > 1 ならば、解と係数の関係から、
それらの総和は 0 であることがわかる
0854132人目の素数さん2020/05/12(火) 23:57:31.69ID:6F2V66NY
e^(i2π/M)を掛けても(nがずれるだけで)和Sは変わらない。
 Se^(i2π/M)= S,
M>1 のとき e^(i2π/M)≠1
0855132人目の素数さん2020/05/13(水) 00:53:59.81ID:2QHQ2mTG
>>845
∠A = t, ∠B = u, ∠C = s,
とおくと
 s+t+u = 180゚

∠L =(t+u)/2 = 60゚ +(60゚-∠C)/2,
∠M =(u+s)/2 = 60゚ +(60゚-∠A)/2,
∠N =(s+t)/2 = 60゚ +(60゚-∠B)/2,

60゚に向かって(1.5倍)寄ってくるので、
正三角形以外は相似にならない。
0856132人目の素数さん2020/05/13(水) 15:46:12.76ID:TyXtfd/1
グラフ理論で閉路c6の直径、半径はどちらも3ですか?
0857132人目の素数さん2020/05/13(水) 16:37:28.87ID:kJ1xjDA9
多様体上の関数が点pで最大値をとるなら微分は0は成り立ちますか?
成り立つなら証明を教えてください
0859132人目の素数さん2020/05/13(水) 17:19:19.68ID:Ic/fO1E1
a[n]=0.999999...(小数点以下に9がn個)とする。a[n]<p<a[n+1]を満たす有理数からなる集合をS_nとする。
S_nの要素のなかで、既約分数の形で表したときの分母の桁数が最小になるもの1つとり、その分母の桁数をN[n]とする。
lim[n→∞] N[n] = +∞ は成り立つか。
0860132人目の素数さん2020/05/13(水) 17:23:45.66ID:wwtUdnQA
お話ぶった切って申し訳ないのですが以下の疑問について教えて頂ける方はいらっしゃいますでしょうか?

@自然数1からnまでの約数の個数の総和の公式または近似式について一般的に知られているものはあるのでしょうか?

A自然数1からnまでの約数の個数の逆数の総和の公式または近似式について一般的に知られているものはあるのでしょうか?

よろしくお願いします
0861132人目の素数さん2020/05/13(水) 17:26:25.58ID:wwtUdnQA
あとこれもお願いします!

約数関数の公式ですが、この関数に出てくるμやγと言った記号は何を意味しているのでしょうか?

https://i.imgur.com/FpyitS6.jpg
0862132人目の素数さん2020/05/13(水) 17:32:41.45ID:bLjdgSej
>>861
μは(νも)ただの自然数(1からnまで)だろ
γはその画像には出てきてないから知らん
0863132人目の素数さん2020/05/13(水) 18:06:49.25ID:wwtUdnQA
>>862
なるほど、γじゃなかったですね見間違いしました
0864132人目の素数さん2020/05/13(水) 20:25:50.62ID:RccedyTb
>>831-844
なんだか知らんが
>>820 から建設的な話に発展して良かったわ。
ちなみに、皆の話している内容は
まったくワイの頭に入らん。

大学レベルの解析学の知識なのかな。
0865132人目の素数さん2020/05/13(水) 22:16:19.22ID:OL2EE5gC
>>864
いわゆる代数学の基本定理。
簡単なようで難しい。
ガウスは生涯で四つだか六つだか本質的に異なる証明を与えたとか。
大筋として
・複素関数論使う(リュービルの定理使うなど)
・代数的位相幾何学系でやる
・ガロア理論使う
2番目は直感的にはすぐそりゃそうだと思えるけど、数学的に厳密化するのは結構難しい。
1番目が多分意外と1番簡単。
3番目は数学科でないと無理。
0866132人目の素数さん2020/05/13(水) 22:17:59.29ID:OmzkE7Cy
>>865
>ガウスは生涯で四つだか六つだか本質的に異なる証明を与えたとか。

平方剰余の相互法則ではなく?
0867132人目の素数さん2020/05/13(水) 22:41:13.87ID:1AYYOpy0
ガウスによる代数学の基本定理の証明は4つらしい
全部見たことはないけど
0869132人目の素数さん2020/05/13(水) 23:25:09.89ID:Zbu5x5qi
他のスレで見た問題なんですが手も足も出ないので教えて下さい

a_(n+1)=a_n - (1/a_n)
a_1=2とする
(1) a_nは収束しないことを示せ
(2) a_nは非有界であることを示せ

1は収束値をaとすると、a=a-1/aとなるaがないことから分かるのですが、2はどうやって示せばいいのでしょうか
0870132人目の素数さん2020/05/14(木) 00:22:42.93ID:w+h9h8DE
>>859
a[n]<1 は単調増加で
 lim[n→∞]a[n]= 1,
とする。 p∈S_n ならば
 a[n]< p < 1
pの分母をMとすれば
 p ≦ 1 - 1/M,
 1 - 1/M > a[n],
 M > 1/(1-a[n]),
 N[n]≧ -log(1-a[n]),   (常用対数)
 lim[n→∞]N[n]= +∞
0871132人目の素数さん2020/05/14(木) 01:19:22.85ID:gPEAv0yf
>>869
y=x-1/xのグラフでも見てみたら?
0872132人目の素数さん2020/05/14(木) 04:49:29.76ID:w+h9h8DE
>>594
>>759
桑野耕一「ラグランジュ恒等式とは何か」
  数学セミナー、連載(2006年4月号〜)
0873132人目の素数さん2020/05/14(木) 07:26:05.04ID:FpIi6hKf
745です。
最終的に4人で応募して、二人当たりました!
総応募数は、46名でした!

計算してくれた皆様ありがとうございました!
0874132人目の素数さん2020/05/14(木) 07:45:25.91ID:wJxP1lyG
>>871
x-1/xは非有界ですが、それとa_nが非有界かは別ではないでしょうか
0875132人目の素数さん2020/05/14(木) 09:47:17.29ID:44IPwDRu
>>871
確かにグラフを見れば直観的には明らかな気もするが、
数列 a_n が有界の範囲内で「ループ」する可能性が排除できなくね?
0876132人目の素数さん2020/05/14(木) 10:36:56.62ID:954J+g7L
>>873
神です。

このスレを見て、君の家族ために
当選確率をいじっておきました。
0877132人目の素数さん2020/05/14(木) 11:54:11.60ID:gPEAv0yf
>>874.875
y=xとy=x-1/xとでx=2からどう動いていくかを見る
0878132人目の素数さん2020/05/14(木) 12:02:04.18ID:gPEAv0yf
負になるところは正に折り曲げて
0879132人目の素数さん2020/05/14(木) 12:21:20.10ID:44IPwDRu
>>877
いや分かるけどね
2 → 3/2 → 5/6 → -11/30 → 779/330 → …
って感じで、 x の絶対値が 0 に近づくと次の x - (1/x) の絶対値は大きくなる

でもそれが本当に有界にならないことを示すのはまた別の難しさがあるのでは
エクセルで a_1000 まで計算してみても、絶対値は 50 を超えないみたいだし
0880132人目の素数さん2020/05/14(木) 12:55:26.28ID:zc5pFGyk
>>869

1/a_{n} = b_{n} 等として、b_{n}で挙動を見た方が扱いやすいと思われる。
b_{n}の動きを見るグラフは、y=x/(1-x^2)

0<|b_{n}|< 1 なら、|b{n+1}|>|b_{n}|
1<|b_{n}|< √2 なら、|b{n+1}|>√2
√2<|b_{n}| なら、|b{n+1}|<√2

という挙動を取る。|b_{n}|=0,√2の場合は、停滞/振動するが、そうでない場合は、
特定の周期を持つか、あるいは、(正負を考えて)六つの範囲を、行き来することが確認できる。
0882132人目の素数さん2020/05/15(金) 03:29:08.79ID:1Op9vddI
[2(vt/√5 -1)a+(vt/√5)b]×(2a+b)=0
t=4/√5×v
どなたかこの数式の途中式お願いします
0883132人目の素数さん2020/05/15(金) 04:20:26.55ID:jFGVDVfH
>>745
3人とも外れ
(32*31*30)/(45*44*43) = 0.34954193093728  >>747 >>751
1人だけ当たり
3*(32*31*13)/(45*44*43) = 0.45440451
2人が当たり
3*(32*12*13)/(45*44*43) = 0.17589852
3人とも当たり
(11*12*13)/(45*44*43) = 0.020155038

>>873
4人とも外れ
(33*32*31*30)/(46*45*44*43) = 0.250758341
1人だけ当たり
4*(33*32*31*13)/(46*45*44*43) = 0.434647792
2人が当たり
6*(33*32*12*13)/(46*45*44*43) = 0.252376137
3人が当たり
4*(33*11*12*13)/(46*45*44*43) = 0.057836198
4人とも当たり
(10*11*12*13)/(46*45*44*43) = 0.00438153
0886132人目の素数さん2020/05/15(金) 11:25:25.70ID:gSETMYHr
>>882
式写し間違えてない?
0888132人目の素数さん2020/05/15(金) 12:54:24.63ID:1Op9vddI
[2{vt/√(5) -1}i+{vt/√(5)}j](2i+j)=0

 ∴t=4/√(5)v

文面だと分かりづらいですね。
0890132人目の素数さん2020/05/15(金) 13:50:35.59ID:1Op9vddI
自己解決しました!
答えを有理化したら合いました、迷惑をかけて申し訳ないです。
0891132人目の素数さん2020/05/15(金) 14:43:56.67ID:6oHV1UgQ
>>858
f○γの微分が0になるのはなぜですか?
γによってはf○γがt=0で極値をとらないこともありませんか?
0892132人目の素数さん2020/05/15(金) 14:48:08.59ID:isLXFKGA
>>888 エスパー
[2{vt/√(5) -1}i+{vt/√(5)}j]・(2i+j)=0 ただしi・j=0, i・i=j・j=1
2{vt/√(5) -1}(i・(2i+j))+{vt/√(5)}(j・(2i+j))=0
4{vt/√(5) -1}+{vt/√(5)}=0
5vt/√(5) -4 =0

 ∴t=4/(√(5)v)
0893132人目の素数さん2020/05/15(金) 15:12:32.22ID:1Op9vddI
>>892
おぉ……ありがとうございます!
0894132人目の素数さん2020/05/15(金) 15:44:59.37ID:3dOo0xKH
>>891
>>858じゃないけど、微分の定義から明らかでは
d/dt (f(γ(t)) = lim[h→0] ( f(γ(t+h)) - f(γ(t)) ) / h
でしょ?
γ(0) = p で関数 f が最大値をとるなら、常に
f(γ(h)) ≦ f(γ(0))
だから、 h→+0 と h→-0 の極限が一致することから
d/dt (f(γ(t)) = 0 at t = 0
でなければならない
0896132人目の素数さん2020/05/15(金) 18:00:13.03ID:kQg3ZDZ7
虚数っていう、この言葉が酷い。
imaginary って幻影か何かのように言ってるけど、
ちゃんと複素平面上に点をとれるよね。

i = √-1 これを実在しないと言うなら、
ゼロや負の値なんかもっとimaginary だよ。

発明されたどの概念も
すべては観念上の物に過ぎないから
数学すべてが imaginary や。 虚学や!
0897132人目の素数さん2020/05/15(金) 20:00:08.45ID:9FhKEJR1
負の数もnegative numberと言われてて登場直後は嫌われてたろうと思う
昔の偉い数学者も1-2は0とするのが自然と考えていたようだし
けど借金の計算とかで日常生活で自然と出てきたので受け入れられた

虚数は複素平面で表されるけど、単なるR^2平面と違って足し算だけでなく掛け算構造まで入ってしまってる
日常生活でここまで複雑な情報扱うことはないから、ほとんどの人にとっては方程式の解を表すための便宜的な数で終わってしまってる
結局数学界で受け入れられたのはオイラーの定理、代数学の基本定理、複素解析で非常に綺麗な結果が出るから、数学の中では自然と思われて受け入れられた

虚数を世の中に浸透させるには高校生全員に複素平面だけでは飽き足らず、代数学の基本定理や複素解析を学ばせる必要があるだろう
0898132人目の素数さん2020/05/15(金) 20:40:26.67ID:ofoiXtbS
複式簿記の方がグロタンディーク構成と保存量の線形代数プロトタイプに思える。
0900132人目の素数さん2020/05/15(金) 22:53:33.64ID:4UfYL4F0
∫[0,a] sin(x)/x dx = ∫[b,∞] exp(-x^2) dx
を満たす正の実数a,bが満たす関係式を求めよ。
0902132人目の素数さん2020/05/16(土) 12:37:16.53ID:oii8q1/C
3+7k=2^nを満たすような自然数n,kは存在しないことを示せ
0903132人目の素数さん2020/05/16(土) 13:50:32.42ID:QWsoVfrS
n=3q+r (r=0,1,2)
2^n = 2^(3q+r) = ((7+1)^q)*(2^r) ≡ 2^r [not≡] 3 (mod.7)
0904132人目の素数さん2020/05/16(土) 13:51:27.17ID:VN/D3za9
>>902
もしそのような自然数 n, k が存在すれば、そのような k の最小値が存在するので、それを K とする
左辺が偶数となることから、 K は奇数である
したがって、ある自然数 m が存在して、 K = 2m + 1 と書ける
このとき、
3 + 7K = 2(5 + 7m)
となるが、これが偶数であるためには、 m は奇数でなければならない
したがって、ある自然数 j が存在して、 m = 2j + 1 と書ける
このとき、
5 + 7m = 2(6 + 7j)
となるが、これが偶数であるためには、 j は偶数でなければならない
したがって、ある自然数 i が存在して、 j = 2i と書ける
このとき、
6 + 7j = 2(3 + 7i)
となる
さて、 3 + 7K = 2^n であったから、以上より、
2^3 (3 + 7i) = 2^n
となるので、両辺の素因数分解を考えると、ある自然数 n_0 が存在して、 3 + 7i = 2^n_0 となる
ところが、 i < K であるので、これは K の最小性に矛盾する
0906132人目の素数さん2020/05/16(土) 15:31:18.16ID:r2A4ZBtC
n=1

(略解)
n≧2 のとき
 5^n = 5^(n-2)(3^2 + 4^2) ≧ 3^n + 4^n,
ゆえ 左辺 > 右辺。
0907132人目の素数さん2020/05/16(土) 15:56:10.47ID:VN/D3za9
>>905
n = 1 のみ

なぜなら、 mod 3 で考えれば、
2^n + 5^n ≡ 4^n (mod 3)
となるが、 n = 1, 2, 3, … に対して、
2^n ≡ 2, 1, 2, 1, … (mod 3)
5^n ≡ 2, 1, 2, 1, … (mod 3)
4^n ≡ 1, 1, 1, 1, … (mod 3)
であるので、 n ≡ 1 (mod 2)

一方、 mod 4 で考えれば、
2^n + 5^n ≡ 3^n (mod 4)
となるが、 n = 1, 2, 3, … に対して、
2^n ≡ 2, 0, 0, 0, … (mod 4)
5^n ≡ 1, 1, 1, 1, … (mod 4)
3^n ≡ 3, 1, 3, 1, … (mod 4)
であるので、 n = 1 または n ≡ 0 (mod 2)

ゆえに、 n = 1 以外のときは
2^n + 5^n = 3^n + 4^n
は成立しない
0908132人目の素数さん2020/05/16(土) 15:56:37.70ID:r2A4ZBtC
>>902
 4^2 = 16 ≡ 2 (mod 7)
∴ 2 は mod 7 での平方剰余。
∴ mod p で非剰余{3,5,6}となるような 2ベキは存在しない。

* 一般に(pp-1)/8 が偶数 ⇔ 2 が mod p での平方剰余。
(第2補充法則)
0909132人目の素数さん2020/05/16(土) 16:22:15.87ID:r2A4ZBtC
>>906

n>1 のとき
 f(x) = x^n は下に凸ゆえ、Jensen により
 (2^n+2^n+5^n)/3 >{(2+2+5)/3}^n = 3^n,
 (2^n+5^n+5^n)/3 >{(2+5+5)/3}^n = 4^n,
辺々たすと
 2^n + 5^n > 3^n + 4^n,
0910132人目の素数さん2020/05/16(土) 18:04:12.37ID:VN/D3za9
>>904
訂正

>これが偶数であるためには、

これが2のべきであるためには、
0911132人目の素数さん2020/05/16(土) 18:06:55.26ID:r2A4ZBtC
n>1 のとき
2項公式で
(5^n - 4^n)-(3^n - 2^n)
={(4+1)^n - 4^n}-{(2+1)^n - 2^n}
= Σ[k=1,n-1] C[n,k] (4^k - 2^k)
> 0,
あるいは
(5^n - 3^n)-(4^n - 2^n)
={(3+2)^n - 3^n}-{(2+2)^n - 2^n}
= Σ[k=1,n-1] C[n,k] (3^k - 2^k)・2^(n-k)
> 0,
0914132人目の素数さん2020/05/16(土) 18:40:25.56ID:5X6GadRw
h/(exp(ah)-1)=(1/a)*((ah)/(exp(ah)-1))->1/a*1 (h->0)
0916132人目の素数さん2020/05/16(土) 22:14:56.61ID:r2A4ZBtC
>>909
>>911

n >1, a > 1 のとき 2項公式より
f_n(a)=(a-1)^n - 2a^n +(a+1)^n
= 2C(n,2)a^(n-2)+ 2C(n,4)a^(n-4)+ ・・・・
 > 0,
より
 2^n - 3^n - 4^n + 5^n
 =(2^n - 2・3^n + 4^n)+(3^n - 2・4^n + 5^n)
 = f_n(3)+ f_n(4)
 > 0,
0917132人目の素数さん2020/05/17(日) 01:20:08.66ID:3mQ+mqXu
実正方行列Aが対角化できないときでも、虚数を成分に含む行列表現まで許せば、Aを対角化できますか?
0919132人目の素数さん2020/05/17(日) 07:59:11.47ID:jv4DNZp5
対角行列のm乗は、各成分をm乗したものである。
∴対角行列(≠O)はm乗しても ≠Oである。(冪零ではない。)

また、冪零性は相似変換において保存される。
(PAP^(-1))^m = P(A^m)P^(-1)= POP^(-1)= O,
0920132人目の素数さん2020/05/17(日) 08:59:32.24ID:jv4DNZp5
>>916
d_n = 2^n - 3^n - 4^n + 5^n の評価

d_1 = 0,
d_2 = 4,
d_(n+1)- 5d_n =
 ={2^(n+1)- 3^(n+1)- 4^(n+1)+ 5^(n+1)}- 5(2^n - 3^n - 4^n + 5^n)
 = -3・2^n + 2・3^n + 4^n
 = 3(3^n -2^n)+(4^n - 3^n)
 > 0,

d_n ≧ 4・5^(n-2)   (n>1)
0921132人目の素数さん2020/05/17(日) 12:36:10.77ID:8HptwUYX
レベル低い質問で申し訳ないんだけど

2(dy/dx)-2(y/x)-(y^3/x^3)logx=0

上の微分方程式みたいにlogxとかが含まれてても同次形z=y/x として計算していいのかどうかがわかりません
0923132人目の素数さん2020/05/17(日) 13:34:43.92ID:8HptwUYX
>>922
ですよね
これ院試の過去問にあったやつで、先輩方の解答例が全部同次形でやってた(logxはそのまま)ので気になって質問しました
これはベルヌーイ形とみて解くという考え方で合ってますか?
0925132人目の素数さん2020/05/17(日) 14:53:19.36ID:8HptwUYX
>>924
手打ちですみませんが

2(dy/dx)-2(y/x)-(y^3/x^3)logx=0
z=y/xとすると
dy/dx=z+x(dz/dx)
与式に代入して
2x(dz/dx)+(z^3)logx=0
-1/z^3 dz=logx/2x dx
両辺積分して・・・

って感じでlogxそのまま残して変数分離形に持ち込んでます
0926132人目の素数さん2020/05/17(日) 15:05:19.02ID:4Wd753iR
なるほど
左辺の符号が違う気がするが、変数分離形にできる場合もあるのか
そうして求まった解 y = f(x) が元の微分方程式を満たしていることが確認できればよさそう
0927132人目の素数さん2020/05/17(日) 15:19:13.90ID:T90WB8A4
数学掲示板群 ttp://x0000.net/forum.aspx?id=1

学術の巨大掲示板群 - アルファ・ラボ ttp://x0000.net
数学 物理学 化学 生物学 天文学 地理地学
IT 電子 工学 言語学 国語 方言 など

PS 連続と離散を統一した!
ttp://x0000.net/topic.aspx?id=3709-0
0928132人目の素数さん2020/05/17(日) 16:03:54.81ID:8HptwUYX
>>925
>>926
与式の(y^3/x^3)logxの前の符号が+でした、すみません!
0929132人目の素数さん2020/05/17(日) 16:44:54.89ID:pbs0E7GR
結局z=y/xとおけば同次形のときはうまく行くけど必ずしも同次形出なくともうまくいく時もあるくらいに思っておくしかないような。
どんな微分方程式でも現代数学で解けてるやつ全部覚えてしまうなんておそらく専門家でも無理な気がする。
0930132人目の素数さん2020/05/17(日) 21:35:53.57ID:jVCw+yKb
>>929
昔は公式集・数表、今は計算機でマセマティカみたいな統合的なシステムとか構築するのにコスト掛けてる。
0931132人目の素数さん2020/05/17(日) 22:52:48.55ID:XSX55d+G
>>869
「もし有界なら収束部分列がとれるが、その極限aはa=a-(1/a)を満たさねばならず矛盾」
でOKな気がするが安直過ぎてどこかに考え落としがあるかも
0932132人目の素数さん2020/05/17(日) 23:10:43.44ID:ma6kDNrV
>>931
それはあかん。

a1=1, a(n+1)=-1/an

有界とすると収束部分裂が取れる。
その極限をaとするとa=-1/aを満たさなければいけないが、そのような実数はないので矛盾‥‥ではない。
0933132人目の素数さん2020/05/17(日) 23:16:39.50ID:idOy6qD3
>>931
収束部分列が定義の漸化式満たすとは限らない
0934132人目の素数さん2020/05/17(日) 23:18:47.69ID:XSX55d+G
>>856
yes
定義から明らかでは?
0935132人目の素数さん2020/05/17(日) 23:19:20.94ID:MOJwpJ9P
>>931
a_[n+1]=f(a_n)のとき部分列(b_n)も同じ関係式b_[n+1]=f(b_n)を満たすか?と言われたら殆ど満たさないでしょう
0937132人目の素数さん2020/05/17(日) 23:22:53.85ID:XSX55d+G
>>932
>>933
あーその通りですね ぼんやりしていた
そんなに甘いはずが無いか
0938132人目の素数さん2020/05/17(日) 23:35:26.69ID:4Wd753iR
本当に非有界なのかな
a_n は常に有理数だし、ループする可能性のほうが高くね?
0941132人目の素数さん2020/05/18(月) 11:58:27.28ID:5tleQtKo
友人に聞かれた問題なんですけど恥ずかしながら全く解けない...
http://imepic.jp/20200518/429930
0942132人目の素数さん2020/05/18(月) 12:30:06.32ID:b04VT8qc
>>941
問題ですらない
∇ と a と ∇ × a の定義を確認して、定義に従って計算するだけ
0943132人目の素数さん2020/05/18(月) 12:54:58.50ID:0gqdjMjt
エディントンのε(レヴィ・チヴィタの記号)は
e_(ijk)= 1  (ijk)が(123)の偶置換
   = -1 (ijk)が(123)の奇置換
   = 0  otherwise
だから
(∂2 a3 - ∂3 a2)e1 +(∂3 a1 - ∂1 a3)e2 +(∂1 a2 - ∂2 a1)e3,
ここで
 (∂1, ∂2, ∂3)=(∂/∂x, ∂/∂y, ∂/∂z)= ∇
0945132人目の素数さん2020/05/18(月) 14:11:44.31ID:hA1IIleM
>>938

aとbが互いに素なら、

a/b - b/a = (a^2-b^2)/(ab) = (a+b)(a-b)/(ab)

これは、既約分数。
従って、初項が有理数なら、この数列の分母は、どんどん大きくなり、ループすることはない。
0946132人目の素数さん2020/05/18(月) 14:19:23.09ID:zWuoSzdr
すいません物凄い馬鹿なんですが
vが2.0の時のUmaxの値を比例配分で求めたいのですがどうやってやればいいのでしょうか
教えていただけませんか?

https://i.imgur.com/5X1UazS.jpg
0947132人目の素数さん2020/05/18(月) 14:26:21.04ID:b04VT8qc
>>945
なるほど、ループはしないのね
でも非有界になるもっともらしい理由はわからない
y = x - (1/x) のグラフから考えれば、数列 a_n が非有界であるということは、
例えば、 x > 2 から減少して降りてきたときに、 x が 0 (または 1 )にいくらでも近くなり得るということを意味するけど、
どうしてそうなるんだろうか?
0948132人目の素数さん2020/05/18(月) 14:37:09.29ID:9c+q2gj1
数学掲示板群 ttp://x0000.net/forum.aspx?id=1

学術の巨大掲示板群 - アルファ・ラボ ttp://x0000.net
数学 物理学 化学 生物学 天文学 地理地学
IT 電子 工学 言語学 国語 方言 など

PS 連続と離散を統一した!
ttp://x0000.net/topic.aspx?id=3709-0
0949132人目の素数さん2020/05/18(月) 14:51:17.34ID:0gqdjMjt
>>939
y' = 1/z(y),
d/dt =(y ')(d/dy)=(1/z)(d/dy),
y'' =(1/z)(1/z) ',
これらを与式に入れてzを掛けると
(1/z) ' + (y-1)z + √z = 0,
0950132人目の素数さん2020/05/18(月) 14:57:30.01ID:OTNg8CTC
0 4 5 6 8
上記の5個の数字と計算式を使って
83
を導く、もしくは近づけたいです
0952132人目の素数さん2020/05/18(月) 15:21:13.02ID:hA1IIleM
>>947

 >>880 で書いた
>> 0<|b_{n}|< 1 なら、|b{n+1}|>|b_{n}|
と >>945 で書いた
>> 初項が有理数なら、この数列の分母は、どんどん大きくなり、
が恐らく回答になります。
0953132人目の素数さん2020/05/18(月) 15:33:42.93ID:b04VT8qc
>>952
すみません、全然わかりません

>> 0<|b_{n}|< 1 なら、|b{n+1}|>|b_{n}|

これは、数列 a_n が上の条件と同値な条件
|a_n| > 1
を満たすときに、 |a_(n+1)| < |a_n| が成り立つということですが、
そのことと、

>> 初項が有理数なら、この数列の分母は、どんどん大きくなり、

はどのような関係にあるのでしょうか?
0954132人目の素数さん2020/05/18(月) 15:44:07.02ID:zCuhpDVw
「10^10^10^10、テトラログの場合、(1×10^10000000000)+1桁の数となる。」

これって合ってますか?
0956132人目の素数さん2020/05/18(月) 16:03:35.39ID:hA1IIleM
>>953
大部分は、1<|a_{n+1}| < |a_n|  に従った「微減」ですが、
たまに、微減が過ぎ、|a_{n+1}| < 1 になることがあります。
そうなると、ちょっとの間、ダイナミックな動きが起こりますが、また、
|a_{n+1}| < |a_n| に従う「平穏」な動きに戻ります。

一連の平穏な動きを、第k次微減列と呼ぶこととすると、
「第k次微減列の中での 1との距離の最小値」 というものを考えることができます。
それ(=|1-a_x|)を、{c_k}とすると、c_k の分母は、どんどん大きくなります。
微減の度合いは、1に近くなればなるほど緻密になるため、
c_k より、c_{k+1}のほうが小さいことが期待されます。
これが、無限に繰り返されるので、c_kはいくらでも、小さくなるという論理です。
0958132人目の素数さん2020/05/18(月) 16:31:21.02ID:b04VT8qc
>>956
わかるような、わからないような…

>微減の度合いは、1に近くなればなるほど緻密になるため、

むしろ、 1 に近くなればなるほどダイナミックになるんじゃないですか?
例えば、 1/100 による微減と、 1/10 による微減では後者のほうが変動幅が大きいと思います

分母が大きくなることの有効性がわかりません
値は分子との比によって決まるので、
分子の大きさがわからないと何とも言えない気がします
0959132人目の素数さん2020/05/18(月) 17:02:04.05ID:1IQa2o4y
>>941
友人じゃなくて、
「おれが友人だと思っている男」に訂正しろ。

ちゃんと証明されてないからね。
0964132人目の素数さん2020/05/18(月) 18:59:27.84ID:hb1pLHLA
集合X={a,b,c}について、関係R = {(a, b), (b, a)} の反射律、推移律、対称律について考えています。反射律は成り立たない。対称律は成り立つ。ここまではわかるのですが推移律で悩んでます。
0966132人目の素数さん2020/05/18(月) 19:13:07.20ID:b04VT8qc
推移律の定義をよく読んで、成り立つか成り立たないか考えればいい
0967132人目の素数さん2020/05/18(月) 19:32:31.60ID:0gqdjMjt
(a,b) (b,c) (c,a) の
・すべて成立     (例:≠)
・1つだけ成立
・すべて不成立
の3とおりが考えられる。

反射律  a〜a
推移律  a〜b ∧ b〜c ⇒ a〜c
対称律  a〜b ⇔ b〜a
0970132人目の素数さん2020/05/18(月) 19:43:30.11ID:hb1pLHLA
>>965
回答ありがとうございます。教科書に書いてあった推移の定義が「a〜bかつb〜cのとき(ry」だったので「a〜bかつb〜a」の問題で悩んでました。cがaでも同じように考えられるということですか?
0971132人目の素数さん2020/05/18(月) 19:44:35.13ID:hb1pLHLA
>>966
すみません。教科書を読んでもしっくりこなかったもので。
0972132人目の素数さん2020/05/18(月) 19:47:49.20ID:b04VT8qc
その教科書にどのように書かれているかわからんが、普通は推移律と言えば、
a, b, c は集合の任意の元なので、当然 c = a でも可
0973132人目の素数さん2020/05/18(月) 19:51:07.85ID:hb1pLHLA
>>972
そうなんですね。分かりました。ありがとうございました。
0974132人目の素数さん2020/05/18(月) 20:00:42.16ID:0gqdjMjt
・a〜b、b〜c、c〜a のうち1つでも成立すれば >>965 により
(推) 不成立

・すべて不成立 ⇔ (推)成立
0975132人目の素数さん2020/05/18(月) 20:14:58.15ID:hA1IIleM
>>958

私は、b_n=1/a_n を定義し、b_n を通してa_nを考えていました。
動きを見るグラフは、y=x/(1-x^2) です。
原点付近から、漸近線x=±1に徐々に向かっていくパターンの説明だったのですが、
緻密な動きをしているのは、原点近辺。漸近線に近づくにつれて加速してますね。
間違ってました。ごめんなさい。

では、こう考えましょう。あるところまでの、{|a_n|}の最大値がNだったとします。
N<|a_k|<2N なるkは存在するか?
これは、b_kにおいては、1/2N < |b_k| < 1/N があるかという問題になります。

関数、y=x/(1-x^2) において、値域が [1/2N,1/N]となるのは、定義域がどのような時か?
より原点に近い範囲と、絶対値の大きなところの二つの範囲が必ず候補となります。
(グラフ y=x/(1-x^2) の性質)
原点に近い方は、求めるものでは無いので、絶対値の大きな方が求める範囲となります。
そして、そこが、値域となるような、定義域はどこか?...
これを繰り返していけば、いずれは、現実路線と繋がる。というのではどうでしょう?
0976132人目の素数さん2020/05/18(月) 20:58:06.76ID:b04VT8qc
>>975
そうですね
例えば、 N = 3 としましょう
このとき、
関数 y = x / (1 - x^2) において、値域が [1 / 6, 1 / 3] となるのは、
絶対値の大きなところでは、定義域が(大雑把に見て) -6 〜 -4 のあたりになります
そして、値域が -6 〜 -4 のあたりになるような定義域は、
|x| = 1 の近くになります
これを数列 b_n に翻訳すると、要するに |b_n| が 1 に近づくということであり、
数列 a_n に翻訳すれば、結局は |a_n| が 1 に近づくということになります
値域が [-1 / 3, -1 / 6] の場合も同様です

うーん…
0977132人目の素数さん2020/05/18(月) 22:13:21.09ID:Yjebi02/
>>970で躓いてるってことは、文脈で意味が違うのに同じ文字だから同じはずだって言ってるも同然だから
これからもあちこちで同じ間違いで詰まる可能性高すぎるな……
0978132人目の素数さん2020/05/18(月) 22:26:27.42ID:SRN4oWZK
>>917
ジョルダン
0979132人目の素数さん2020/05/18(月) 22:28:36.12ID:b04VT8qc
>>977
まあ、初学者あるあるだな
違う文字が使われていると違うものだと思ったり、
∀ と ∃ の違いがわからなかったり、
A ⇒ B という命題は A が偽のときは必ず真になることがわからなかったり、…

上でも ∇ × a を計算するだけの問題があったが、
「定義をちゃんと読む」ということが初学者にとって、いかに難しいことなのかがよくわかる
0980132人目の素数さん2020/05/18(月) 22:55:01.35ID:Yjebi02/
>>979
こういうのってどう応えてあげるのがいいのか毎回悩むよ
思う存分自分で悩んで納得するまで考えるのが個人的には良さそうなんだけど
そういう方向で回答すると上から目線のマウントだとか嫌がらせだとか
割とボロカス返されることが多々ある
逆に「これが正しい推論だからこれを覚えろ」みたいな方向で答えると
きっちり曲解される、変なスローガンに頼り始める、数学は暗記だみたいに捉える
みたいな悲惨な結果もよくある話に……
0981132人目の素数さん2020/05/18(月) 23:01:01.67ID:KTzWRLMz
クソ問は放置が正解
0982132人目の素数さん2020/05/18(月) 23:05:33.52ID:b04VT8qc
>>980
個人的には、先に答えを教えちゃうと本人のためにならないと思うんだよね
だからやっぱり時間をかけて自分で納得できるように誘導するべきなんだと思う

一方で、「数学は暗記だ!」と主張しながら教職を取りつつ、国立大の博士課程に進学した人を知っているので、
記憶力が良い人は先に覚えてしまうのも一つの手なのかなとも思う
0983132人目の素数さん2020/05/18(月) 23:10:21.71ID:gENZkXy3
ここは分からない問題を書くスレです
0984132人目の素数さん2020/05/18(月) 23:16:33.70ID:b04VT8qc
すみません…
つい解答方法の是非について語ってしまいました
0985132人目の素数さん2020/05/18(月) 23:23:27.67ID:hA1IIleM
>>976
チョット伝わらなかったのかもしれませんが、ミソは候補となる区間が二つ現れると言うことです。
中には、探る価値か意味が無い区間も含まれるかもしれませんが、調べてみる価値がある区間も含まれています。

今回は、答えがわかっているからやれることですが、b[15]=-0.2779... となり、これが、求めるものです。
b_nは、{0.50,0.66,1.2,-2.7,0.42,0.51,0.703,1.394,-1.47,1.255,-2.17,0.58,0.878,3.85,-0.2779,...}

1/6 ≦x/(1-x^2)≦ 1/3 → x∈[-3-√10,(1/2)(-3-√13)]∪[√10-3,(1/2)(√13-3)]
b[14]=3.85 これを含む区間はありません。ただし、-3.85を含む区間ならあるので、以後符号を反転します。

-3-√10 < x/(1-x^2) < (1/2)(-3-√13) → x∈[-0.922148...,-0.860006...]∪[1.08443...,1.16278...]
-0.878 を含む方の区間[-0.922148...,-0.860006...]を採用し、
-0.922148 < x/(1-x^2) < -0.860006... を解き、-0.58を含む方の区間 [-0.595326,-0.575335] を選ぶ。
...と繰り返していけば、今回は、必ず、-b[1]を含む区間が得られます。

一般の場合は、候補区間数が倍→四倍→八倍→...に増えていきますが、そのどれかに、前提としてよい値を含む区間に
到達するであろうと考えられます。
0988132人目の素数さん2020/05/18(月) 23:38:00.95ID:hxVG1u3/
>>869を少し変形した
a(n+1)={a(n)-1/a(n)}/2は一般項を求めることができて
a(n)=tan((b+π/2)*2^(n-1)-π/2)
ただしbはtan(b)=2を満たす数
ところがこれですらa(n)の非有界性を示すのはかなり難しくb/πの正規性を示すという未解決問題クラスの難しさ
一般項すら出すの難しいこっちはさらに厳しいんじゃねえかという気がする
0989132人目の素数さん2020/05/18(月) 23:39:38.43ID:Twrrq+J8
>>987
それならスレの趣旨に沿っているので全く問題ありません
0990132人目の素数さん2020/05/18(月) 23:40:27.50ID:DRtned0W
ここの健全な運びを討論する自治スレが別にあるわけじゃなきゃここで語る内容だろ
お前も歳いってるだろうに何でそんな壊れたロボット判断するんだよ
末長くやってくにゃ現役回答者で此のスレの運び方を話し合って、やり方を詰めてくべきだろ

そんな最初から完璧なスレ運び出来るほど人生やってないだろ?関孝和だって最初から完璧は無理_b
0991132人目の素数さん2020/05/18(月) 23:46:05.23ID:b04VT8qc
>>985
ええと、つまり、
b_n → b_(n+1) というよりは、むしろ
b_(n-1) ← b_n という方向で考えるとわかりやすいってことですかね?

|b_k| < 1/N なる k が存在すると「仮定」すれば、数列 b_n を「遡る」ことによって、
候補となる区間に属するような b_(k-1), b_(k-2), … を拾い出すことができると
で、その下降列はやがて b_1 に到達するだろうと

そんな感じですかね?

きちんと証明するなら、トートロジーにならないように気を付ける必要がありそうですが…
0992132人目の素数さん2020/05/18(月) 23:47:14.40ID:13WCrwJX
俺は先に全ての答えを教えてためにならないとは思わないな。
車輪の再発明をする意味が良くわからない。
0993132人目の素数さん2020/05/18(月) 23:58:28.84ID:13WCrwJX
答えを完全に出して「考え方」そのものを暗記すれば良い。
どうせ「考える力」みたいなものがあったとしても、自力で「過去数学を発展させてきた者たち」に考える力で勝利し続けるのは不可能だ。
そんな者たちがどう考えたのかを回答を見て学ぶほうが効率的だろうね。
0994132人目の素数さん2020/05/19(火) 00:07:05.17ID:fmb4zX+n
こと数学に限って言えば、理屈のつながりといううわべだけ残すのが数学の解答で
「考え方」は解答にはほぼ残らないので、まず解答を書くというのはむしろ能率悪いよ
0995132人目の素数さん2020/05/19(火) 00:15:43.52ID:HnyGTIPw
>>993
有名な手法レベルの解法で、それを知らないだけなら、それでもいいと思うけどね
(帰納法の応用とか、鳩ノ巣原理とか、無限降下法とか、…)
ただし、問題のパターン化には限界があるし、
重要なのは「見たこともない問題」や「解けるかどうかわからない問題」に出会ったときに、どう対処するか?
あと、明らかに定義が理解できていないだけの「問題」には、答えを先出しするのはいかがなものかと思う

まあ俺自身、偉そうに語るだけの知識があるわけじゃないけど
ポリアの本でも読もうかな
最近だと、テレンス・タオの本もいい感じらしいって聞いた
0996132人目の素数さん2020/05/19(火) 00:31:02.61ID:VFcp0MVp
純粋な論理を重ねていくだけで
解ける類の物なら暗記でいいと思う。

想像力が多少なりとも必要な問題だとキツいな。
数学オリンピックの問題とか…
過去の解法を暗記しても解けるかどうか。
0997132人目の素数さん2020/05/19(火) 09:05:33.05ID:t7nozl3h
>>995
そもそも自分で考えれば未知の問題を解く力もつく
ということに疑問がある
何故か常識化しているが、そんなデータは見たことがない
0998132人目の素数さん2020/05/19(火) 09:06:50.32ID:t7nozl3h
一方で分からない問題は回答を全部見て納得する、を繰り返せば、効率的に多くの問題を理解できるので、
「全くもって未知の問題」に遭遇する確率そのものを下げることができる
09999502020/05/19(火) 10:58:33.99ID:by4XN031
>>963
ありがとうございます!
すっきりしました。
10011001Over 1000Thread
このスレッドは1000を超えました。
新しいスレッドを立ててください。
おみくじ集計(特殊)
【男の娘】 1
【腐女子】 0
【髪】 0
【神】 0
【姫君】 0
【女神】 0
【尊師】 0

life time: 50日 11時間 48分 6秒
10021002Over 1000Thread
5ちゃんねるの運営はプレミアム会員の皆さまに支えられています。
運営にご協力お願いいたします。


───────────────────
《プレミアム会員の主な特典》
★ 5ちゃんねる専用ブラウザからの広告除去
★ 5ちゃんねるの過去ログを取得
★ 書き込み規制の緩和
───────────────────

会員登録には個人情報は一切必要ありません。
月300円から匿名でご購入いただけます。

▼ プレミアム会員登録はこちら ▼
https://premium.5ch.net/

▼ 浪人ログインはこちら ▼
https://login.5ch.net/login.php
レス数が1000を超えています。これ以上書き込みはできません。

ニューススポーツなんでも実況